[obm-l] Lament�vel mal entendido

2007-09-21 Por tôpico Carlos Eddy Esaguy Nehab

Oi, Aline,

Peço desculpas se minha brincadeira não foi clara o suficiente e a 
induziu a pensar em algo inadequado.


Sou um antigo participante da Lista da OBM (e retornei há pouco 
tempo) e certamente tenho idade de ser, possivelmente, quase seu avô.


Na verdade comentários como o que fiz só poderiam ser entendidos por 
quem efetivamente acompanha a lista há mais tempo e, portanto, foi 
inadequado para uma Lista (qualquer que fosse).


Mas gostaria de esclarecer o contexto de minha brincadeira...:  há 
uma colega sua que brincou uma vez na Lista nos chamando de meninos 
(pois a maioria dos participantes é de jovens, efetivamente) e isto 
rendeu uma deliciosa brincadeira, pois muitos participantes da lista 
tem mais do que 50 anos (eu por exemplo tenho 61) e há até 
participantes com quase 80 anos!


Assim, meu comentário (infeliz, repito) foi no sentido de que eu 
deixaria o prazer de responder a sua pergunta aos meninos 
(propriamente ditos - ou seja, os jovens) uma vez que de fato é raro 
(infelizmente) meninas na Lista.


Espero que este esclarecimento encerre este lamentável mal entendido 
causado, sem dúvida, pela inadequação de minha brincadeira.


Abraços,
Nehab

=
Instruções para entrar na lista, sair da lista e usar a lista em
http://www.mat.puc-rio.br/~nicolau/olimp/obm-l.html
=


Re: [obm-l] Re: [obm-l] 1 Equa�ao 2 incognitas - o menor n�o existe...

2007-09-19 Por tôpico Carlos Eddy Esaguy Nehab

Bem,...

Se realmente o enunciado não fala em inteiros e a gente quiser ir às 
últimas conseqüências (ainda com trema), a resposta também não é 
x=115/43 nem nada... A questão é impossível...  :-)  pois o 
'menor' não existe...


Nehab

At 02:31 19/9/2007, you wrote:
Oi Dirichlet, a questão não diz se x e y são naturais...e que letra 
você marcaria?

:-)

-- Johann Peter Gustav Lejeune Dirichlet ---
 Bem, na questão não diz nada sobre x e y serem naturais... A minha resposta
 seria x=115/43,y=1 por exemplo...


...
  
  
Dada a igualdade 1935x = 5175y , onde x0 e y0, o menor valor que a
   variável x pode assumir e o menor valor que a variável y pode 
assumir, de

   modo que seja verdadeira a igualdade, têm soma igual a:
  
   a)   155
   b)   156
   c)   157
   d)   158
   e)   159
  
 
 


 --
 Ideas are bulletproof.

 V



=
Instruções para entrar na lista, sair da lista e usar a lista em
http://www.mat.puc-rio.br/~nicolau/olimp/obm-l.html
=


Re: [obm-l] Medida e Forma em Geometria (Elon Lages Lima)

2007-09-19 Por tôpico Carlos Eddy Esaguy Nehab

Caramba !

Eu ia responder quando percebi que temos mais uma menina para fazer 
companhia à Bruna (que anda sumida, né)...


Portanto, deixarei este privilégio para os milhões de meninos da 
lista..., apenas deixando uma pergunta à Aline: você sabe o que é homotetia?


Nehab

At 11:11 19/9/2007, you wrote:




Olá Colegas...

Estou com dúvida e agradeceria muito se alguém pudesse me ajudar.

Trace no plano as semi-retas OX, OY, OZ com a mesma origem O, de 
modo que OZ esteja no interior do ângulo XOY. Por cada ponto P em 
OZ, sejam Q o pé da perpendicular baixada de P sobre OX e S a 
interseção com OY da paralela a OX passando por P. Prove que a razão 
PQ/PS não depende do ponto P tomando OZ.




Obrigada pela atenção, abraços a todos...


--
MSN Hotmail, o maior webmail do Brasil. 
http://g.msn.com/8HMBBR/2737??PS=47575Faça o seu agora. 
= 
Instruções para entrar na lista, sair da lista e usar a lista em 
http://www.mat.puc-rio.br/~nicolau/olimp/obm-l.html 
=


Re: [obm-l] Medida e Forma em Geometria (Elon Lages Lima) - ??!!

2007-09-19 Por tôpico Carlos Eddy Esaguy Nehab

Delon (e Aline),

Eu não devia responder, mas em respeito a você e à Aline...

Talvez você não tenha acompanhado em passado recente os comentários 
divertidos da Bruna, única menina participante da lista até então (a 
menos de pseudônimos), nos chamando de meninos (e alguns de nós com 
idade para ser tio ou avô dela - o meu caso, por exemplo).Acabou 
criando uma bonita cumplicidade com todos nós: moços ou não..


Assim, você interpretou de forma totalmente equivocada a minha 
brincadeira.   Mas me perdoe se deixei margem para esta interpretação boba.


Quanto a sua explicação, infelizmente parece conter outro equívoco: 
apenas triângulos com mesmos ângulos internos são semelhantes.  Isto 
não vale para outros polígonos, como parece ser o argumento que você 
usou para os retângulos da figura...


Atenciosamente,
Carlos Nehab

At 13:43 19/9/2007, you wrote:
Comentário estranho este... eu iria, mas desisti ao ver... sei 
lá,.. Imagino que não seja o caso, mas deixa margem para um 
interpretação preconceituosa.
Aline, se você fizer a representação das semi-retas e  seguir as 
demais orientações, verá que  tomando um determinado ponto P 
formará  um  quadrilátero (OSPQ).  Se este ponto P for alterado para 
um P', outro quadrilátero será formado (OS'P'Q'),  porém será 
semelhante ao primeiro, pois  todos os ângulos serão mantidos, 
afinal,  PS é  paralela a OX, então sempre formará com OY (ou OS) um 
mesmo ângulo. Da mesma forma, a perpendicular sempre gerará um mesmo 
ângulo, e o ângulo SOQ (ou YOX) é definido (o quarto ângulo será 
automaticamente congruente).
Então, tente fazer o quadrilátero OSPQ e outro OS'P'Q'. Verá a 
semelhança, e pelas propriedades já conhecidas de semelhança terá 
que PQ/PS = P'Q'/P'S', portanto, indiferente do ponto P.


Espero que ajude, ...

Delon

On 9/19/07, Carlos Eddy Esaguy Nehab 
mailto:[EMAIL PROTECTED][EMAIL PROTECTED] wrote:

Caramba !

Eu ia responder quando percebi que temos mais uma menina para fazer 
companhia à Bruna (que anda sumida, né)...


Portanto, deixarei este privilégio para os milhões de meninos da 
lista..., apenas deixando uma pergunta à Aline: você sabe o que é homotetia?


Nehab


At 11:11 19/9/2007, you wrote:




Olá Colegas...

Estou com dúvida e agradeceria muito se alguém pudesse me ajudar.

Trace no plano as semi-retas OX, OY, OZ com a mesma origem O, de 
modo que OZ esteja no interior do ângulo XOY. Por cada ponto P em 
OZ, sejam Q o pé da perpendicular baixada de P sobre OX e S a 
interseção com OY da paralela a OX passando por P. Prove que a 
razão PQ/PS não depende do ponto P tomando OZ.




Obrigada pela atenção, abraços a todos...


--
MSN Hotmail, o maior webmail do Brasil. 
http://g.msn.com/8HMBBR/2737??PS=47575Faça o seu agora. 
= 
Instruções para entrar na lista, sair da lista e usar a lista em 
http://www.mat.puc-rio.br/%7Enicolau/olimp/obm-l.htmlhttp://www.mat.puc-rio.br/~nicolau/olimp/obm-l.html 
=




Re: [obm-l] EQUA��O

2007-09-17 Por tôpico Carlos Eddy Esaguy Nehab

Oi, Arkon

Ou então veja que

3^(x/2) = [2^(x/2) - 1 ] x [2^(x/2) + 1 ]

Como a diferença entre os dois fatores da direita vale 2, um deles 
não pode ser divisor de 3 ou seja

2^(x/2) -1 = 1, isto é, x = 2.

Nehab

At 13:47 17/9/2007, you wrote:
Faça um gráfico e note que só há uma solução. E veja que x = 2 é 
solução. Logo, o conjunto solução é {2}.


Abraço
Bruno


2007/9/17, arkon mailto:[EMAIL PROTECTED][EMAIL PROTECTED]:

Alguém pode resolver, por favor, esta:

3x/2 + 1 = 2x

DESDE JÁ MUITO OBRIGADO




--
Bruno França dos Reis
email: bfreis - http://gmail.comgmail.com

e^(pi*i)+1=0


Re: [obm-l] Duro de Matar Geometrico

2007-09-17 Por tôpico Carlos Eddy Esaguy Nehab

Bem, Paulo

.a comunidade matemática, inclusive tenho logo que pensar 
num mestrado.


Acho melhor parar de pensar e começar a correr atrás...  Palavras de 
um coroa que sabe que a hora passa muito rápido...


Espero um dia poder tomar um café com vocês em algum colóquio no 
IMPA (se eu tomar vergonha e me inscrever).


Francamente, Paulo  O dia que você quizer ir ao IMPA me telefone 
(9949.1442) e eu o cicironeio  Vou de vez em quando lá ver livros 
e matar as saudades (além disso meu filho mais novo é ligado à turma 
de computaçao gráfica de lá - incluindo um homônimo seu, o PC de lá).


Abraços,
Nehab 

Re: [obm-l] Duro de Matar Geometrico - off topic

2007-09-17 Por tôpico Carlos Eddy Esaguy Nehab
 de outro craque (que nos deu aula de 
geometria descritiva): o Frank Schaeffer !
Artista plastico e muito distinto (era um verdadeiro lorde), o 
Schaeffer foi um dos poucos catedraticos do IME. Na ultima vez em 
que conversei com ele , frente `a sua lucidez, perguntei-lhe a 
idade: algo em torno de 90 anos...


Sera' que a gente chega la' ?

Abracao,
Rogerio Ponce


Carlos Eddy Esaguy Nehab [EMAIL PROTECTED] escreveu:
Oi, Palmerim,

Acho que preciso corrigir uma imagem equivocada de minha 
competência   Os craques que eu conheci da geometria (e também 
de desenho geométrico e de geometria descritiva e projetiva, de meu 
tempo de aluno) foram Célio Pinto de Almeida, Virgílio de Athayde 
Pinheiro e Luiz Oswaldo (esqueci o sobrenome - do IME e da UFF) e, 
20 anos depois, o colega Wagner desta lista (citado por 
você).  Perdoe minha ignorância (e memória) se não lembro de outros.


Embora apaixonadíssimo pela Geometria, ela nunca foi meu forte e 
talvez por isto mesmo a estude tanto...Mas por favor, fique 
muito melhor do que eu em Geometria, senão você correrá o risco de 
entrar pelo cano(êta expressãozinha antiga!).


Minha paixão pela Matemática consegue ser menor que minha paixão 
pelo Ensino da Matemática e talvez por isto você tenha me confundido 
como geômetra.  Quem dera!


E com relação a tem professor IME/ITA aposentado nesta 
lista,  infelizmente, ainda trabalho prá caramba e por esta razão 
minha presença aqui é tão descontínua no tempo (às vezes sobra, 
quase sempre falta...).


Um grande abraço,
Nehab

PS: Um conselho de um coroa que admira seu tesão pelo estudo: não 
pendure livros na Internet.  Sugira apenas sua compra, caso 
contrário de que adiantaria (eu, você, etc) escrever o livro que 
você sugeriu :-)  ?   Lembre-se que professores, matemáticos e 
geômetras também vivem de seu trabalho e um direitozinho autoral é 
uma ajuda fundamental no leite das crianças (mesmo que sejam netos...).




Flickr agora em português. Você clica, todo mundo vê. 
http://br.rd.yahoo.com/mail/taglines/flickr/*http://www.flickr.com.br/Saiba 
mais.


Re: [obm-l] EQUA��O

2007-09-17 Por tôpico Carlos Eddy Esaguy Nehab

Oi,

Obrigado pela correção.  Você tem toda razão...  Imaginei, 
equivocadamente, que x era inteiro.


Tentarei uma solução sem derivadas...

Abraços,
Nehab

At 23:50 17/9/2007, you wrote:
\neste último você está considerando que n é natural, e isto não 
está escrito em lugar nenhum...


Se temos 2^x-1=3^(x/2), seria interessante provar que o lado 
esquerdo cresce mais rápido que o direito.
Usando algo como derivadas, sairia logo. MAs tem que ver se x0 não 
dá outra solução.



Em 17/09/07, Carlos Eddy Esaguy Nehab 
mailto:[EMAIL PROTECTED][EMAIL PROTECTED] escreveu:

Oi, Arkon

Ou então veja que

3^(x/2) = [2^(x/2) - 1 ] x [2^(x/2) + 1 ]

Como a diferença entre os dois fatores da direita vale 2, um deles 
não pode ser divisor de 3 ou seja

2^(x/2) -1 = 1, isto é, x = 2.

Nehab


At 13:47 17/9/2007, you wrote:
Faça um gráfico e note que só há uma solução. E veja que x = 2 é 
solução. Logo, o conjunto solução é {2}.


Abraço
Bruno


2007/9/17, arkon mailto:[EMAIL PROTECTED][EMAIL PROTECTED]

:
Alguém pode resolver, por favor, esta:
3x/2 + 1 = 2x
DESDE JÁ MUITO OBRIGADO



--
Bruno França dos Reis
email: bfreis - http://gmail.comgmail.com

e^(pi*i)+1=0




--
Ideas are bulletproof.

V


Re: [obm-l] Duro de Matar Geometrico - off topic

2007-09-14 Por tôpico Carlos Eddy Esaguy Nehab

Oi, Palmerim,

Acho que preciso corrigir uma imagem equivocada de minha 
competência   Os craques que eu conheci da geometria (e também 
de desenho geométrico e de geometria descritiva e projetiva, de meu 
tempo de aluno) foram Célio Pinto de Almeida, Virgílio de Athayde 
Pinheiro e Luiz Oswaldo (esqueci o sobrenome - do IME e da UFF) e, 20 
anos depois, o colega Wagner desta lista (citado por você).  Perdoe 
minha ignorância (e memória) se não lembro de outros.


Embora apaixonadíssimo pela Geometria, ela nunca foi meu forte e 
talvez por isto mesmo a estude tanto...Mas por favor, fique muito 
melhor do que eu em Geometria, senão você correrá o risco de entrar 
pelo cano(êta expressãozinha antiga!).


Minha paixão pela Matemática consegue ser menor que minha paixão pelo 
Ensino da Matemática e talvez por isto você tenha me confundido como 
geômetra.  Quem dera!


E com relação a tem professor IME/ITA aposentado nesta 
lista,  infelizmente, ainda trabalho prá caramba e por esta razão 
minha presença aqui é tão descontínua no tempo (às vezes sobra, quase 
sempre falta...).


Um grande abraço,
Nehab

PS: Um conselho de um coroa que admira seu tesão pelo estudo: não 
pendure livros na Internet.  Sugira apenas sua compra, caso 
contrário de que adiantaria (eu, você, etc) escrever o livro que você 
sugeriu :-)  ?   Lembre-se que professores, matemáticos e geômetras 
também vivem de seu trabalho e um direitozinho autoral é uma ajuda 
fundamental no leite das crianças (mesmo que sejam netos...).


At 11:02 14/9/2007, you wrote:

Ola Nehab e Paulo,

ja comecei a ESTUDAR o trabalho da Silvana (agora vou ficar bom 
nisso, que nem o Nehab), e tambem vi que ela fala sobre inversão. 
Conheco tambem um bom artigo sobre inversao para simplificar 
problemas complexos de geometria, que pode ser baixado no link:

http://www.4shared.com/file/24240164/f2daf78/InversaoProblemasGeometricos.htmlhttp://www.4shared.com/file/24240164/f2daf78/InversaoProblemasGeometricos.html

Acho que ja esta na hora de termos em lingua portuguesa um livro do 
tipo 
http://www.4shared.com/file/24241815/800655c3/Geometry_Revisited__HSM_Coxeter__SL_Greitzer_1967_.htmlGeometry 
Revisited com assuntos e problemas mais avancados como o Teorema de 
Pompeiu e aqueles de que trata a Silvana. O livro dos mestres Wagner 
e Morgado (tive a honra de ser aluno deles no Colegio Princesa 
Isabel, em botafogo) deu um bom inicio, mas agora merecemos mais. 
Tem professor IME/ITA aposentado nesta lista que poderia fazer muito 
bem esse trabalho. Tenho certesa que o livro seria um verdadeiro 
sucesso, Nehab...


P.S.Tambem sou do Rio, Paulo Cesar.

Abracos,
Palmerim



Em 13/09/07, Carlos Eddy Esaguy Nehab 
mailto:[EMAIL PROTECTED][EMAIL PROTECTED] escreveu:

Oi, Palmerim,

O problema abaixo é exatamente um dos dois problemas que eu 
mencionei na resposta a você.   Portanto, não deixe de ler a tese 
da Silvana...

...
E ja que estamos falando em diversidade de solucoes, existe um 
problema muito famoso, conhecido de todos aqui da lista, do mesmo 
tipo do que apresentei aqui, mas que possui oito excelentes 
solucoes que foram colecionadas pelo professor Tom Rike, que faz 
parte do Berkeley Math Circle (sei lah o que eh isso), e que vale a 
pena dar uma boa olhada e estuda-las (pelo menos para mim...). Esta 
no link abaixo:
http://www.4shared.com/file/24161313/99643ed9/An_Intriguing_Geometry_Problem.htmlhttp://www.4shared.com/file/24161313/99643ed9/An_Intriguing_Geometry_Problem.html 




Abraços,
Nehab



Re: [obm-l] Duro de Matar Geometrico - UFA !

2007-09-13 Por tôpico Carlos Eddy Esaguy Nehab

Oi, Palmerim,

O problema abaixo é exatamente um dos dois problemas que eu mencionei 
na resposta a você.   Portanto, não deixe de ler a tese da Silvana...

...
E ja que estamos falando em diversidade de solucoes, existe um 
problema muito famoso, conhecido de todos aqui da lista, do mesmo 
tipo do que apresentei aqui, mas que possui oito excelentes solucoes 
que foram colecionadas pelo professor Tom Rike, que faz parte do 
Berkeley Math Circle (sei lah o que eh isso), e que vale a pena dar 
uma boa olhada e estuda-las (pelo menos para mim...). Esta no link abaixo:

http://www.4shared.com/file/24161313/99643ed9/An_Intriguing_Geometry_Problem.htmlhttp://www.4shared.com/file/24161313/99643ed9/An_Intriguing_Geometry_Problem.html



Abraços,
Nehab 

Re: [obm-l] Duro de Matar Geometrico

2007-09-13 Por tôpico Carlos Eddy Esaguy Nehab

Oi, Paulo Cesar,
...
Mas enfim, assim que vi o problema com aquele triângulo 100º, 40º, 
40º, desconfiei que fosse alguma variante daquela questão famosa do 
ângulo adventício de 30º (aprendi esse nome com o Nicolau).
Por que será que esses problemas tão interessantes envolvem sempre 
os mesmos ângulos e triângulos?
Conheço duas outras questões famosas (já postadas aqui) onde os 
mesmos triângulos de 100º, 40º, 40º e 20º, 80º, 80º aparecem, 
envolvendo congruências e paralelismos.


Se você der uma olhada na referência que sugeri da tese Silvana (vai 
gostar, com certeza) encontrará parte de sua resposta,  estudando um 
pouco sobre Complexidade na Geometria...


Uma explicação simplista, mas já uma primeira motivação para você ler 
algo sobre isto, é que tais ângulos recaem em problemas do terceiro 
grau, e não como os problemas menos complexos, que admitem solução 
do segundo grau.


A propósito, sou do Rio, e você?

Abraços,
Nehab


Re: [obm-l] Duro de Matar Geometrico

2007-09-12 Por tôpico Carlos Eddy Esaguy Nehab

Oi, Palmerim,

Pois é, o Ponce diz que solução trigonométrica é feia só para 
implicar comigo.   Mas então eu vou implicar com ele, também, dando a 
solução geométrica.


A inspiração da solução está no mesmo contexto que dois problemas 
clássicos que habitam esta lista com alguma regularidade:
1) Triângulo Isósceles de 20/80/80 onde  A = 20; traçam-se por B e C 
cevianas fazendo 50 e 60 graus com a base BC, de tal forma que D está 
em AC, E em AB, BCE = 50, DBC = 60 e se deseja o angulo EDB;
2) Triângulo Isósceles de 40/40/100, onde A = 100.  marca-se D no 
prolongamento de AB de tal forma que AD = BC.  Pergunta-se o angulo BCD.


O problema que você propôs e mais estes dois estão relacionados com a 
geometria do polígonos regulares eneágono (9 lados) e octadecágono 
(18 lados) e suas diagonais.


Veja a tese de mestrado da Silvana (já mencionada nesta lista pelo 
Nicolau)  cujo capítuko 2 é inteiramente dedicado aos dois problemas 
clássicos acima (1 e 2), mas não se intimide, pois este capítulo é simples.


Copie este link e baixe o pdf:
http://www.google.com.br/url?sa=tct=rescd=2url=http%3A%2F%2Fwww.mat.puc-rio.br%2F~hjbortol%2Fcomplexidade%2Fcomplexidade-em-geometria.pdfei=mfPnRoDCGZyMevvb5dcGusg=AFQjCNG8D4jKXKF_v_qKbopL9H4-74cE6Qsig2=6o2sWVI6_6h-4iAotETV2g

Bem, agora vamos à solução do problema que você propôs:

Lema (que está demonstrado no texto mencionado):
Um octadecágono regular possui 4 diagonais (diferentes de 
'diâmetros') que passam por um mesmo ponto em um diâmetro do seu 
círculo circunscrito.


Ai, a partir de sua figura (no link que você enviou) basta observar que:
a) B é o centro do octadecágono e BC o raio;
b) ED e AD são as diagonais mencionadas no lema e D seu ponto de 
interseção no diâmetro cujo reta suporte é BC;


Dai, é trivial que o ângulo desejado vale 30 graus por ser um ângulo 
inscrito que subentende um ângulo central de 60 graus.


Abraços,
Nehab

PS: Aproveitando a citação da Silvana, dê uma olhada em um outro 
lindo problema clássico chamado de Teorema de Napoleão (o próprio) 
...  que ela desenvolve de uma forma muito elegante e 
interessante.   Para os interessados neste problema, especialmente, 
veja o 
link  http://www.cut-the-knot.org/proofs/napoleon_intro.shtml  do 
maravilho site www.cut-the-knot.com.  Nenhum amante da geometria pode 
desconhecer este site...


At 10:20 12/9/2007, you wrote:

Maravilha!!!

Obrigado Rogerio, ja era previsivel mesmo uma solucao sua. E nao eh 
feia, mas linda. Se algum outro colega da lista conseguir a solucao 
puramente geometrica, agradeço.


Abracos,
Palmerim

Em 12/09/07, Rogerio Ponce 
mailto:[EMAIL PROTECTED][EMAIL PROTECTED] escreveu:

Ola' Palmerim e colegas da lista,
vou dar uma solucao feiosa mesmo, isto e', por trigonometria...

Sem perda de generalidade, vamos atribuir um comprimento unitario a 
cada lado de ABC.
Agora, trace a vertical que passa pelo vertice E, encontrando o 
prolongamento ( 'a esquerda) de BD no ponto F.


Vamos usar o triangulo DEF para calcular a tangente do angulo DEF = 
x +10 graus.

Assim,
tg (x+10) = FD / EF = (FB + 1 - DC) / EF

Como EB=1, e o angulo BEF=10 graus , temos que:
EF=cos10
FB=sin10

Aplicando lei dos senos no triangulo ADC, vemos que
DC= sin20 / sin100

Substituindo os valores no calculo da tangente, obtemos
tg (x+10) = ( sin10 + 1 - sin20 / sin100 ) / cos10

Substituindo sin20 por  2 * sin10 * cos10 , assim como sin100 por cos10, vem:
tg (x+10) = ( 1 - sin10 ) / cos10

Entao, lembrando da velha formuleta
 (1 - sinA) / cosA  = tg ( 45 - A/2 ) ,

finalmente podemos escrever:
tg (x+10) = tg ( 45 - 10/2 ) = tg 40

Que nos da' x=30 graus.

[]'s
Rogerio Ponce

Palmerim Soares mailto:[EMAIL PROTECTED] 
[EMAIL PROTECTED] escreveu:

 Ola pessoal

Esta aqui eh para os grandes mestres Nehab, Ponce e outros geometras 
da lista. Poderiam dar uma solucao puramente geometrica (a moda 
agora e essa...). Mas gostaria tambem de ver a solucao 
trigonometrica, se possivel. Figura no link abaixo:

http://imageshock.eu/img/Triangulo.jpghttp://imageshock.eu/img/Triangulo.jpg

Obrigado,

Palmerim



Flickr agora em português. Você clica, todo mundo vê. 
http://br.rd.yahoo.com/mail/taglines/flickr/*http://www.flickr.com.br/Saiba 
mais .




Re: [obm-l] numera�

2007-09-12 Por tôpico Carlos Eddy Esaguy Nehab

Oi, Kleber,

Se eu entendí o que você perguntou você quer saber a quantidade total 
de símbolos usados para escrever de 1 até 10^n -1 , é isto?


Por exemplo (se eu entendi...), para escrever de 1 a 99 no nosso 
sistema decimal usual, é necessario usar 9 símbolos para os números 
de (1 a 9), mais 180 símbolos para escrever de 10 a 99 (que 
corresponde a 2 simbolos para cada um dos 90 números), etc:


Se minha interpretação esta correta, vamos lá:

-  Decimal usual
Qde = (10 - 1) + 2* (100 - 10) + 3 * (1000 - 100) + ... + n [10^n - 10^(n-1) ]
Qde = n.10^n - [ 1 + 10 + 100 + 1000 + ...+ 10^(n-1) ](note a PG)
Qde = n.10^n - (10^n - 1)/9

Egípcio
Suponha, por exemplo, que você escreveu os números de 1 a 99 e que 
usou até ai X símbolos.  Para escrever os números de 101 a 999 
precisamos unir  as escritas destes 99 números números com:

- 1 símbolo de 'centena' para escrever os números de 101 a 199 (falta o 100);
- 2 símbolos de 'centena' para escrever os números de 201 a 299 (falta o 200);
- 3 símbolos de 'centena' para escrever os números de 301 a 399 
(falta o 300)...

...
- 9 símbolos de 'centena' para escrever os números de 901 a 999;

Mas ficaram faltando o 100, 200, ... e o 900, o que exige um total 
(1+2+...+9) símbolos de centena, ou seja, 45 símbolos a mais.


Então, no total, para escrevermos os números de 100 a 999 usamos  9X 
+ 45 símbolos !


De uma maneira geral (perceba que o raciocínio anterior se mantém), 
chamando de Q(n) a quantidade de símbolos para escrever os numeros de 
1 a 10^n - 1 usaremos 9 vezes esta quantidade para completar a 
escrita de 10^n até  10^(n+1) -1 acrescidos de 1+2+3+...+9 = 45 símbolos.

Assim:
Q(n+1) = Q(n) + 9 Q(n) + 45 , ou seja, Q(n+1) = 10.Q(n) + 45 e 
obviamente Q(1) =  45  (1 pauzinho mais 2 pauzinhos, etc)


Esta recorrência é clássica (uma progressão 
aritmética-geométrica).  Logo,  Q(n) = 5(10^n - 1)


Espero não ter me distraído...

Abraços,
Nehab

PS: Possivelmente para o caso egípcio n é no maximo 6 pois não 
conheço símbolo maior que 1 milhão neste sistema de numeração.


At 12:00 12/9/2007, you wrote:
Considere o sistema de numeração por agrupamento aditivo egípicio e 
o sistema de numeração decimal indo-arábico.


Para cada n pertencente aos naturais, determine expressões para as 
quantidades de símbolos necessários para expressar todos os números 
naturais menores que 10^n em cada um dos sistemas.


--
Kleber B. Bastos


Re: [obm-l] SOMA DAS RA�ZES

2007-09-12 Por tôpico Carlos Eddy Esaguy Nehab

Oi, Arkon.

Acho que você tá precisando colocar um formulariozinho de trigono em 
sua mesinha de cabeceira :-)


Use:

sen3x = 3.sen x - 4.(senx)^3   e
sen2x = 2.sen x . cos x
(sen x)^2 = 1 - (cos x)^2

e tente fazer o exercício.

Abraços,
Nehab

At 16:14 12/9/2007, you wrote:

Alguém pode resolver, por favor, esta:?xml:namespace prefix = o ns 
= urn:schemas-microsoft-com:office:office /




?xml:namespace prefix = v ns = urn:schemas-microsoft-com:vml 
/(AFA-97) A soma das raízes da equação sen 3x + sen 2x = 0 para 0 
= x  = pi é :




a) 6.pi/5.  b) 9.pi/5. c) 
11.pi/5. d) 13.pi/5.




DESDE JÁ MUITO OBRIGADO


Re: [obm-l] Duro de Matar Geometrico

2007-09-12 Por tôpico Carlos Eddy Esaguy Nehab

Oi, Paulo Cesar,

Gostei muito da solução.  Eu bem que tentei mas não havia conseguido 
uma solução tão geometrica e bonita.  Mas quanto a fortalecer 
simulados, tu é mau pra caramba, hein...  :-)


Sua solução também inspira outra solução para a propriedade que 
mencionei das diagonais do octadecágono.


E confesso que não conhecia o problema.   Será que o Palmerim sabe a 
origem deste problema ?  Não vale dizes que caiu em prova, pois vou 
achar que foi na sua...


Abraços,
Nehab

At 22:32 12/9/2007, you wrote:


Olá Palmerim.

Caso ainda reste alguma curiosidade sobre o problema, aí vai mais 
uma solução. Tal solução, como disse o Nehab, é mesmo referente aos 
polígonos eneágono e octadecágono, mas a abordagem é um pouco mais 
independente das construções desses polígonos. Aí vai:


Trace o segmento CE, cortando AB em F. Trace agora o segmento DF.
O triângulo CBE é isósceles, logo BCE = BEC = 40º.
Note que o quadrilátero AFDC será um trapézio isósceles (observe os 
ângulos de 20º e 40º que as diagonais formam com os lados oblíquos). 
Logo, temos que FD é paralelo a AC e, portanto, BFD = 60º.

Concluímos então que o triângulo BFD é equilátero com BF = FD = BD.
Observe agora o triângulo BEF. O mesmo é isósceles (40º, 40º, 100º), 
daí BF = FE.
Finalizando, veja então que o triângulo DEF é isósceles (FD = EF, 
10º 10º e 160º). Como BED = x temos

x = 40º - 10º = 30º.

Espero ter ajudado.
Agradeço pela questão. Serviu para fortalecer um dos simulados do 
curso onde trabalho.


[]'s

PC


[obm-l] R = 2r por geometria...

2007-09-05 Por tôpico Carlos Eddy Esaguy Nehab

Ao colega 'perdido',

Não consegui localizar quem havia solicitado, há pouco tempo, uma 
demonstração 'puramente geomérica' que o raio do círculo circunscrito 
a um triângulo é maior ou igual ao diâmetro do circulo inscrito (na 
verdade a iguladade só vale no equilátero).


Confesso que havia pensado no problema mas, ai vai a historinha...:

- Primeiro pensei como seria em um triângulo retângulo: trivial, pois 
o triângulo retângulo está contido na 'metade' de seu círculo 
circunscrito e, então, seu círculo inscrito está inteiramente contido 
no semicírculo circunscrito.  Logo, R = 2r e note que o mesmo 
raciocínio também vale para um triângulo obtusângulo.


- Então fiquei ralando' nos triângulos acutângulos...  Pensei: uma 
boa idéia seria tentar dobrar o triângulo original (suponha-o ABC), 
pois ai eu teria um triângulo com raio inscrito igual a 2r.  Traçando 
paralelas aos lados pelos vértices ABC seria uma boa forma de 
'dobrá-lo'  (chame tal triângulo de A'B'C').Ai, olhei, olhei e 
não vi.  Faltou o pulo do gato e 'encostei o problema'


Esta semana, absolutamente por acaso, folheando o Lidski, lá estava o 
maldito problema (359) e o pulo do gato também.
Para quem não tem  Lidski ai vai o pulo do gato (apenas): construa um 
triângulo paralelo a A'B'C' com lados tangentes ao círculo 
circunscrito do triângulo original ABC (chame-o de ABC) e olhe, 
olhe, e olhe, que você vê que o A'B'C' está contido em A'BCe então 
seu círculo circunscrito é menor ou igual a R, que por sua vez, é 
menor ou igual a 2r...


Abraços,
Nehab

=
Instruções para entrar na lista, sair da lista e usar a lista em
http://www.mat.puc-rio.br/~nicolau/olimp/obm-l.html
=


Re: [obm-l] SOMA

2007-08-31 Por tôpico Carlos Eddy Esaguy Nehab

Oi, Arkon.

Escreva sua expressão assim, dividind os dois 
polinômios...:  (n+31)/(n-4) = 1 - 35/(n-4).


Agora continue...

Abraços,
Nehab

At 12:29 31/8/2007, you wrote:




Pessoal alguém pode resolver , por favor, esa:

?xml:namespace prefix = o ns = urn:schemas-microsoft-com:office:office /

Existem quatro números naturais que o quociente n + 31/n – 4 é um 
número natural. A soma desses quatro números é igual a:


a) 64.   b) 68.   c) 72.   d) 78.   e) 89.



DESDE JÁ MUITO OBRIGADO


Re: [obm-l] SOMA

2007-08-31 Por tôpico Carlos Eddy Esaguy Nehab

Obviamente errei as contas e o Saulo respondeu corretamente,
Abraços,
Nehab

At 18:27 31/8/2007, you wrote:

Oi, Arkon.

Escreva sua expressão assim, dividind os dois 
polinômios...:  (n+31)/(n-4) = 1 - 35/(n-4).


Agora continue...

Abraços,
Nehab

At 12:29 31/8/2007, you wrote:




Pessoal alguém pode resolver , por favor, esa:

?xml:namespace prefix = o ns = urn:schemas-microsoft-com:office:office /

Existem quatro números naturais que o quociente n + 31/n – 4 é um 
número natural. A soma desses quatro números é igual a:


a) 64.   b) 68.   c) 72.   d) 78.   e) 89.



DESDE JÁ MUITO OBRIGADO


Re: [obm-l] Ge�metras de Plant

2007-08-28 Por tôpico Carlos Eddy Esaguy Nehab

Oi, Geo,

Antes de mais nada, não esqueça de assinar algum nome.  Para mim, 
pelo menos, não é instigante escrever sem saber o nome de meu 
destinatário.  Na boa :-)


Bem, longe de ser geômetra, apenas uma dica básica para os problemas 
que você listou...


Assim como a razão entre as áreas de duas figuras planas semelhante é 
o quadrado da razão de semelhança, a razão entre os volumes de duas 
figuras semelhantes (ou homotéticas se preferir)  é o cubo da razão 
(linear, é claro) de sua semelhança.


Exemplos:
Se um triângulo é o triplo de um outro (no sentido usual de razão 
entre segmentos correpondentes), sua área é 3^2  = 9 vezes a área do outro;


Da mesma forma se um cubo, uma pirâmide, ou seja lá o que for for o 
quádruplo de outra figura (no sentido da razão linear usual de 
semelhança) seu volume é  4^3  = 64 vezes o volume da outra


Abraços,
Nehab


1-)Um cone tem 6cm de altura. A que distância do vértice deve estar uma
seção transversal para que o volume do cone destacado seja 8/27 do volume do
cone dado ?



2-)Um tanque cônico de diâmetro de base D, inicialmente cheio, despeja água
através de um dreno de diâmetro a dentro de um tanque cilíndrico
inicialmente vazio, que também tem diâmetro D. O tanque cilíndrico tem um
dreno de diâmetro b em seu fundo. Determine o valor mínimo de b para que o
tanque cilíndrico não transborde, sabendo que o volume
total do tanque cônico é o dobro do volume do tanque cilíndrico. Considere
que o tanque cilíndrico dispõe de uma válvula que permanece fechada até seu
nível máximo de água
ser atingido, após o que é aberta.

Obs.: Existe alguma relação entre o volume e a altura entre o cone e o
tronco de cone ?

Pessoal muito obrigado, um abraço a todos.


RE: [obm-l] CONE SUL 1996

2007-08-27 Por tôpico Carlos Eddy Esaguy Nehab

Oi, José,

Caros colegas, se possivel, gostaria que me ajudassem a resolver 
este problema de matematica!


O triangulo ABC, retangulo em Â, e tal que A^BC  A^CB. Abissetriz 
interna de  intercepta o lado BC em D. Seja HD perpendicular a BC 
(H entre A e C). Nestas condiçoes podemos afirmar que o angulo H^BD 
mede, em graus:


Propriedade: Uma bissetriz divide o lado oposto em segmentos 
proporcionais aos dois outros lados.


Assim, dividir a proporcionalmente a b e c  determina em a os 
segmentos  a x b/(b+c) e  a x c/(b+c), ok?  Ou seja:


BD = ac/(b+c)  e  DC = ab/(b+c).  (1)

Mas os triângulos HDB e BAC são ambos retângulos e têm um ângulo em 
comum (C).  Logo, são semelhantes.


Então  HD/DC = c/b.  (2)

Logo, substituindo (1) em (2) obtemos  HD = ac/(b+c).

Logo, HD = BD e seu ângulo HBD vale 45o.

Abraços,
Nehab  

Re: [obm-l] IME - ITA - Provas

2007-08-26 Por tôpico Carlos Eddy Esaguy Nehab

Luiz,

Acho melhor você ficar antenado nesta lista e dar uma paquerada nos 
seguintes sites, que já enviei como dica para outro colega.


Em primeiro lugar, site da OBM. Artigos da revista Eureka e a revista 
inteira:

http://www.obm.org.br/frameset-eureka.htm

O site Majorando, de dois olímpicos, também é muitíssimo interessante.
http://majorando.com/
(vá em artigos: http://majorando.com/?page_id=12)

No site do grupo Teorema, muito bom:
http://www.grupoteorema.mat.br/

O grupo rumoaoita também oferece alguns textos interessantes em seu 
site (embora seja de uma instituição com fins lucrativos)

www.rumoaoita.com/matematica.php

Mas vá com calma.  A sensação de mula, no fundo no fundo é 
estimulante para os neurônios...  :-)


Abraços,
Nehab

At 12:55 25/8/2007, you wrote:

Olá Carlos!!!
Tudo bem???
Gostei muito deste link c/ as questões do IME...
Mas também acabei me sentindo uma mula...
Você, ou alguém do grupo, poderia me indicar uma bibliografia 
decente para encarar problemas como esses???

Abração para todos.
Luiz.


On 8/24/07, Carlos Eddy Esaguy Nehab 
mailto:[EMAIL PROTECTED][EMAIL PROTECTED] wrote:

Oi, filhote de Anne Rice (rsrsrs)

Beleza 

Vá em 
http://www.lps.ufrj.br/profs/sergioln/ccount11/click.php?id=20http://www.lps.ufrj.br/profs/sergioln/ccount11/click.php?id=20 
para as provas de matemática do IME.


Ug,
Nehab


At 14:49 24/8/2007, you wrote:

Oi Pesssoal...
beleza?
...

Tipo  que eu estava querendo as provas do ime ( se possível com 
todas as matérias - mais significamente mat, fis e qui de 99 pra 
cá) resolvidas...

Alguém sabe onde posso achar ou pode me passar se tiver?
Ah... se tiver as antigonas pode mandar que também quero!!!

Até! Desde já agradeço!




Re: [obm-l] Rela�ao Metrica

2007-08-26 Por tôpico Carlos Eddy Esaguy Nehab

Oi, Rita:

Uma soluçao simples é chamar os catetos de b e c e observar que você 
recai num sisteminha simples, pois:


b^2+c^2 = a^2  = 100   (Pitágoras)
bc = ah = 48  (Propriedade simples que se obtém por semelhança)

Lembrando que b^2 + c^2  - 2bc = (b - c)^2   e   b^2 + c^2 + 2bc = (b 
+ c)^2, fica simples fazer as contas...

(b - c)^2 = 100 - 96 = 4 --- b - c = 2
(b + c)^2 = 100 + 96 = 196 ---  b + c = 14

Logo os catetos são os manjados 6 e 8 (no sentido que é um triângulo 
semelhante ao 3, 4, 5)


Abraços,
Nehab


At 13:56 26/8/2007, you wrote:

Caros colegas,

Nao consegui resolver esta questão, talves eu nao esteja cosegundo 
observar todos os angulos da questão, quem pode me ajudar.


A altura relativa a hipotenusa de um triangulo retangulo mede 4,8 e 
a hipotenusa 10. Quais os valores dos catetos?


Rita Gomes


Re: [obm-l] IME - ITA - Provas

2007-08-24 Por tôpico Carlos Eddy Esaguy Nehab

Oi, filhote de Anne Rice (rsrsrs)

Beleza 

Vá em 
http://www.lps.ufrj.br/profs/sergioln/ccount11/click.php?id=20  para 
as provas de matemática do IME.


Ug,
Nehab

At 14:49 24/8/2007, you wrote:

Oi Pesssoal...
beleza?
...

Tipo  que eu estava querendo as provas do ime ( se possível com 
todas as matérias - mais significamente mat, fis e qui de 99 pra cá) 
resolvidas...

Alguém sabe onde posso achar ou pode me passar se tiver?
Ah... se tiver as antigonas pode mandar que também quero!!!

Até! Desde já agradeço!


Re: [obm-l] Desigualdade II

2007-08-23 Por tôpico Carlos Eddy Esaguy Nehab

Oi, Bruna,

Em geral a gente é tentado a desenvolver  (x+y+z)^2 , para resolver 
esta questão, mas não obtemos sucesso, pois as parcelas x^2, y^2 e 
z^2, possuem coeficiente 1, e as parcelas xy, xz e yz têm coeficientes 2.


Então temos que encontrar uma forma de empatar os coeficientes, ou 
seja, gostaríamos de ter  2.x^2, 2.y^2 e 2.z^2.Esta é a motivação 
para perceber que o que deve funcionar (para resolver o problema) é o 
desenvolvimento de


(x - y)^2 + (x - y)^2 + (y - z)^2 que, como é soma de quadrados, é sempre = 0

Abraços,
Nehab

At 04:08 23/8/2007, you wrote:

Olá meninos voltei. rs

Mais uma de desigualdade

x^2 + y^2 + z^2 = xy + xz + yz.

--
Bjos,
Bruna


Re: [obm-l] RESTO

2007-08-23 Por tôpico Carlos Eddy Esaguy Nehab

Oi, Arkon,

Note que se você divide um polinômio P(x) por um polinômo de segundo 
grau, o resto é no máximo do primeiro grau, certo?

P(x) = (x^2 - 1).Q(x) + (Ax+B)

Faça x = 1 e x = -1 nesta igualdade, pois tais valores anulam x^2 - 
1  e pronto, você descobre A e B.


Nehab


At 13:00 23/8/2007, you wrote:


Alguém pode resolver esta:



O resto da divisão de 1 + x + x2 + ... + x100 por x2 – 1 
é:?xml:namespace prefix = o ns = urn:schemas-microsoft-com:office:office /




a) 0.   b) x + 1. c) 50x + 50.  d) 50x + 
51.e) 51x + 50.




DESDE JÁ MUITO OBRIGADO


Re: [obm-l] Desigualdade II

2007-08-23 Por tôpico Carlos Eddy Esaguy Nehab

Johann Peter Gustav Lejeune Dirichlet (ufa)
e Artur e Bruna também...

Como não sei qual série a Bruna cursa, minha sugestão foi no sentido 
de não usar nada além do básico, da mesma forma que sua segunda 
solução e da solução que o Artur sugeriu.


Mas já que o você, Iórran Pêter Lejêne Dirixileti, mencionou a 
solução de rearranjo, não sei se você conhece um ótimo texto do 
Marcio Cohen e do Rodrigo Villard sobre desigualdades homogênas (que 
também é o caso do exercício da Bruna) disponível em 
http://majorando.com/?page_id=12  na seção artigos avançados (não 
confundir com o texto em intermediário) sob o título 'Desigualdades.


Certamente, se você não conhecer o artigo e/ou o tema, vai 
adorar.   E releve a brincadeira do nome, pois eu nunca sei como 
chamá-lo... dá um trabalhão...  Ainda bem que tem o copiar/colar...


Abração
Nehab

At 16:11 23/8/2007, you wrote:

Dá pra usar rearranjo:
Se
A=B=C e a=b=c
Então
Aa+Bb+Cc=Ab+Bc+Ca
Se fizermos A=a, B=b, C=c, acabou!

Outro modo é usar Médias mesmo: x^2+y^2=2xy, escreve para os outros 
pares de variáveis, soma tudo e fim!


Em 23/08/07, Carlos Eddy Esaguy Nehab 
mailto:[EMAIL PROTECTED][EMAIL PROTECTED] escreveu:

Oi, Bruna,

Em geral a gente é tentado a desenvolver  (x+y+z)^2 , para resolver 
esta questão, mas não obtemos sucesso, pois as parcelas x^2, y^2 e 
z^2, possuem coeficiente 1, e as parcelas xy, xz e yz têm coeficientes 2.


Então temos que encontrar uma forma de empatar os coeficientes, ou 
seja, gostaríamos de ter  2.x^2, 2.y^2 e 2.z^2.Esta é a 
motivação para perceber que o que deve funcionar (para resolver o 
problema) é o desenvolvimento de


(x - y)^2 + (x - y)^2 + (y - z)^2 que, como é soma de quadrados, é sempre = 0

Abraços,
Nehab


At 04:08 23/8/2007, you wrote:

Olá meninos voltei. rs

Mais uma de desigualdade

x^2 + y^2 + z^2 = xy + xz + yz.

--
Bjos,
Bruna





--
Ideas are bulletproof.

V


Re: [obm-l] Imagem da uni�o de dois conjuntos

2007-08-23 Por tôpico Carlos Eddy Esaguy Nehab

Oi, Renan,
(ia responder em off, mas acho que este assunto é de interesse geral)

Você está certo.  Na verdade você provou apenas que  f(A inter B) = 
f(A) inter f(B)   (está contido)


Eis o que você escreveu:


{f(x): x pertence (A inter B)} - {f(x): x pertence A e x pertence B}.


Aqui, você está no contexto do se e somente se.  Está 
perfeito.  Mas quando você coloca se x pertence a A, etc

você não está mais no se e somente se...

Veja seu argumento (recheado com um pouco mais de detalhes):

Quero provar que   f(A inter B) = f(A) inter f(B).   Então provemos que:
a)  f(A inter B) = f(A) inter f(B)
b)  f(A) inter f(B) = f(A inter B)

Para provar a)  vejamos (foi o que você fez):

Seja y em f(A inter B); então  há x em (A inter B) tal que f(x) = y
Mas se x está em (A inter B) podemos afirmar que  x está em A e x está em B;
Logo, y = f(x) está em f(A)  e  y = f(x)  está em f(B); logo este 
mesmo y está em na interseção, ou seja, f(A) inter f(B).   Isto você 
provou, como está escrito abaixo  (mas acho que começando um pouco 
pelo meio, pois se você quer provar que M = N forçosamente comece 
assim: seja z em M até chegar a z em N; e você começou assumindo 
um x em A, que não está diretamente ligado ao que você quer provar - 
deu para entender?):


se x pertence a A, f(x)  pertence a f(A) e se x pertence a B, f(x) 
pertence a f(B), dessa forma

f(x) pertence a f(A) e a f(B) - f(A inter B) = f(A) inter f(B)


Mas em nenhum momento você provou que dado um y em f(A) inter 
f(B)  tal y está em f(A inter B), pois, veja (item b) :


Seja y em  f(A) inter f(B); logo, y está em f(A)  e em  f(B)  (pois 
está na interseção dos conjuntos, logo está em cada um deles);  mas 
se y está em f(A)  há algum sujeito em A, chamemo-lo de x1 tal 
que  f(x1) = y;  e se y está em f(B)  há  algum cara em B (chamemo-lo 
de x2)  tal que f(x2) = y.Então você tem um cara x1 em A e um 
cara x2 em B e pronto (e mais nada).  Mas se x1 e x2 forem o mesmo 
cara, então de fato você teria um x=x1=x2  em A inter B e então 
haveria um x em A inter B tal que f(x) = y ou seja,  tal y estaria 
(como desejamos) em f(A inter B).


E é fácil ver que se  f for injetora, de fato x1 e x2 seriam iguais e 
isto fecharia sua demonstração )ou seja a seginda inclusão para 
justificar a igualdade dos conjuntos).


Esta dificuldade que você assinalou, que é comum, me lembrou quando 
fui aluno do Prof. Barbosa no IME (ih em 1969 e 70) e ele nos 
enlouquecia  com milhares de exercícios deste tipo e muitos na época 
achavam um saco.Ledo engano.  Foram estes exercícios que 
certamente nos deram a clareza que hoje minha tchurma tem no que 
poderíamos chamar de prática de lógica e de teoria dos conjuntos básicos).


Abração
Nehab

PS: Conseguiu dar uma paquerada nos livros que sugeri ?

At 21:31 23/8/2007, you wrote:

Olá a todos!

Estou iniciando o estudo de análise real pelo livro do A.J. White
(Análise Real, uma introdução) e Kolmogorov  Fomin (Introductory Real
Analysis, é a terceira edição da tradução do R. Silverman).

Resolvendo os primeiros exercícios do A.J. White encontrei dificuldade em:

f( A inter B) = f(A) inter f(B) sse f é injetora.

Onde f(X) denota o conjunto dos f(x) tal que x pertence a X.

Parece razoavel a premissa de que f é injetora, mas, na demonstração,
não encontro essa condição. Além disso, na página 6 do Kolmogorov há
uma prova que não necessita que a função seja injetora NO CASO DE f(A
união B).

Procedi da seguinte forma na prova: {f(x): x pertence (A inter B)} -
{f(x): x pertence A e x pertence B}. Mas se x pertence a A, f(x)
pertence a f(A) e se x pertence a B, f(x) pertence a f(B), dessa forma
f(x) pertence a f(A) e a f(B) - f(A inter B) = f(A) inter f(B)

Essa prova não é válida, já que encontrei contra-exemplos, mas não
consigo encontrar o erro (já que existem casos que A inter B = vazio e
f(A) inter f(B) não é vazio, casos em que f não é injetora). Uma coisa
me ocorreu enquanto escrevia, o problema foi não ter provado que f(A)
inter f(B) está contido em f(A inter B) ?


Agradeço qualquer ajuda,
Abraços,
J. Renan

=
Instruções para entrar na lista, sair da lista e usar a lista em
http://www.mat.puc-rio.br/~nicolau/olimp/obm-l.html
=


Re: [obm-l] Integral Gaussiana

2007-08-22 Por tôpico Carlos Eddy Esaguy Nehab

Oi, Shine,

Você conhece alguma demonstração que não utilize este artifício 
clássico?   Já procurei no passado outros caminhos, inclusive 
utilizando séries, mas não fui bem sucedido.


Abraços,
Nehab

At 10:56 22/8/2007, you wrote:

Oi Henrique,

Você pode consultar a Wikipedia, em
  http://en.wikipedia.org/wiki/Gaussian_integral
para uma solução (ligeiramente) mais detalhada.

De qualquer forma, você tem que estudar coordenadas
polares (em especial, por que dx dy = r dr dtheta)
para entender essa solução em particular.

[]'s
Shine

--- Henrique Rennó [EMAIL PROTECTED] wrote:

 Olá!

 Encontrei em um livro uma integral que o autor chama
 de integral Gaussiana.
 Não achei a solução muito clara. Alguém poderia me
 explicar com ela foi
 obtida?

 Mostrar que:

 int_-inf_inf {e^[(-a/2)*x^2]} dx = [(2*pi)/a]^(1/2)

 A solução do livro é:

 Primeiro ele chama a integral de I e eleva ao
 quadrado ambos os lados:

 I^2 = int_-inf_inf int_-inf_inf {e^[(-a/2)*x^2 +
 (-a/2)*y^2] dx.dy
 I^2 = int_-inf_inf int_0_2*pi {e^[(-a/2)*r^2]}
 r.dr.dtheta
 I^2 = pi * int_0_inf {e^[(-a/2)*u]} du
 I^2 = (2*pi)/a
 I = [(2*pi)/a]^(1/2)

 Ele considera x = r.cos(theta), y = r.sen(theta) e u
 = r^2

 Em livros de cálculo, qual seria a parte de
 integrais que eu deveria estudar
 para obter o conhecimento utilizado nessa solução?

 Obrigado!

 --
 Henrique






Park yourself in front of a world of choices in alternative 
vehicles. Visit the Yahoo! Auto Green Center.

http://autos.yahoo.com/green_center/
=
Instruções para entrar na lista, sair da lista e usar a lista em
http://www.mat.puc-rio.br/~nicolau/olimp/obm-l.html
=


Re: [obm-l] Integral Gaussiana

2007-08-22 Por tôpico Carlos Eddy Esaguy Nehab

Oi, Nicolau,

Adorei,

Obrigado,
Nehb

At 15:28 22/8/2007, you wrote:

On Wed, Aug 22, 2007 at 12:34:39PM -0300, Carlos Eddy Esaguy Nehab wrote:
 Oi, Shine,

 Você conhece alguma demonstração que não utilize este artifício
 clássico?   Já procurei no passado outros caminhos, inclusive
 utilizando séries, mas não fui bem sucedido.

Eu não sou o Shine, mas vou responder.

Calcular esta integral é equivalente a calcular
(-1/2)! = Gamma(1/2) = sqrt(pi)
onde Gamma é a função Gamma de Euler, ou seja, definimos

a! = int_0^infty t^a e^(-t) dt

De fato, fazendo a substituição s^2 = t temos

int_0^infty e^(-s^2) ds = (1/2) int_0^infty t^(-1/2) e^(-t) dt = (1/2)!

Para provar que (-1/2)! = sqrt(pi) podemos usar o seguinte limite:

a! = lim_(n - infty) n^a * n!/(a+1)(a+2)...(a+n)

Este limite é conseqüência da convexidade de log(Gamma(x)).

Assim,

(-1/2)! = lim_(n - infty) n!/(sqrt(n)*(1/2)*(3/2)*...*((2n-1)/2))
= lim_(n - infty) 2^(2n)*(n!)^2/(sqrt(n)*(2n)!)

Agora usamos Stirling:

n! ~= n^n e^(-n) sqrt(2 pi n)

para obter

(-1/2)! = lim_(n - infty)
2^(2n)*n^(2n)*e^(-2n)*2*pi*n/sqrt(n)*(2n)^(2n)*e^(-2n)*sqrt(2*pi*n)
= sqrt(pi)

Bem, a outra solução ainda é mais simples...

[]s, N.

=
Instruções para entrar na lista, sair da lista e usar a lista em
http://www.mat.puc-rio.br/~nicolau/olimp/obm-l.html
=


Re: [obm-l] Desigualdade, meninos e meninas... quase-off

2007-08-21 Por tôpico Carlos Eddy Esaguy Nehab

Não resisti:

Pois então menina :-), sua apostila está errada...

Abraços,
Nehab, um menino, há muito e muito tempo...

At 04:43 21/8/2007, you wrote:

Olá meninos, na minha apostila só fala que a e b são reais não nulos.

--
Bjos,
Bruna


Re: [obm-l] Desigualdade, meninos e meninas... quase-off

2007-08-21 Por tôpico Carlos Eddy Esaguy Nehab

Puxa, Bruna,

O que tem menino esperto nesta lista...  Você nem imagina!  Mas só o 
oninem espelhado encerrou o assunto...


Abraços,
Nehab

At 14:29 21/8/2007, you wrote:
a desigualdade é válida para todo a e b real não nulo desde que 
tenham o nesmo sinal, podendo portanto serem ambos negativos tambem.

os menimos não viram isto ?

Ojesed.
- Original Message -
From: mailto:[EMAIL PROTECTED]Artur Costa Steiner
To: mailto:obm-l@mat.puc-rio.brobm-l@mat.puc-rio.br
Sent: Tuesday, August 21, 2007 11:18 AM
Subject: RES: [obm-l] Desigualdade, meninos e meninas... quase-off

Eh, mas se puderem ser negativos a desigualdade nao eh valida. Os 
meninos aqui, incluinodo este aqui, menino do inicio dos anos 60, viram isso

Artur
-Mensagem original-
De: mailto:[EMAIL PROTECTED][EMAIL PROTECTED] 
[mailto:[EMAIL PROTECTED] nome de Carlos Eddy Esaguy Nehab

Enviada em: terça-feira, 21 de agosto de 2007 08:04
Para: obm-l@mat.puc-rio.br
Assunto: Re: [obm-l] Desigualdade, meninos e meninas... quase-off

Não resisti:

Pois então menina :-), sua apostila está errada...

Abraços,
Nehab, um menino, há muito e muito tempo...

At 04:43 21/8/2007, you wrote:

Olá meninos, na minha apostila só fala que a e b são reais não nulos.

--
Bjos,
Bruna



--
No virus found in this incoming message.
Checked by AVG Free Edition.
Version: 7.5.484 / Virus Database: 269.12.1/963 - Release Date: 
20/8/2007 17:44


Re: [obm-l] algebra linear

2007-08-20 Por tôpico Carlos Eddy Esaguy Nehab

Oi, Klaus,

Pense no plano, por exemplo:  X_y =  X_0 + y(X_1 - 
X_0)emas   X1 - X_0  é um vetor paralelo à reta que une os 
pontos X_0 e X_1.


Este X_y  é a equação da reta que une os pontos X_0 e X_1.  Ou 
seja, variando y em Reais você cobre a reta...


Se y estiver entre 0 e 1,  o X_y é a expressão de qualquer ponto 
interno ao segmento que une os dois pontos.  Por exemplo, se y = 1/2 
que você tera o ponto médio, certo?


Esta é a motivação de escolher tal X_y:  a reta 

Abraços,
Nehab

At 09:27 20/8/2007, you wrote:


Sejam A uma matriz mxn e B uma matriz mx1. Se o sistema linear AX = B

possui duas soluções distintas X_0 X_1, então ele tem infinitas soluções.

Esse é um teorema que tem em qualquer livro de álgebra linear. Tenho 
um livro aqui que a demonstração é a seguinte:
Seja X_y=(1-y)X_0 + yX_1 , vamos mostrar que X_y é solução do 
sistema AX=B para qualquer y pertencente a R. Para isto vamos 
mostrar que AX_y=B.


Minha dúvida é de onde saiu Seja X_y=(1-y)X_0 + yX_1 ?

Grato.

Flickr agora em português. Você clica, todo mundo vê. 
http://br.rd.yahoo.com/mail/taglines/flickr/*http://www.flickr.com.br/Saiba 
mais.


Re: [obm-l] ajuda em complexo

2007-08-20 Por tôpico Carlos Eddy Esaguy Nehab

Oi, Rivaldo.

Agora que pude ler o enunciado...

De fato; mas o Saulo não disse que serve qualquer  alfa  3/4  (pois 
aí estaria de fato errado).  Mas a questão é de múltipla escolha e 
então, veja o que o Saulo na verdade disse:  se o enunciado do 
problema vale ENTÃO, dentre as opções de resposta, EU (ele) JURO que 
MEU alfa satisfaz a  opção A), pois MEUS alfas valem  +0,707 ou - 0,707...


Tenho duas observações:
1) de fato fato faltou explicitar que para os alfa encontrados há 
realmente 4 soluções para z;

2) a opção C também é válida e ai eu acho que a questão melou...

Abraços,
Nehab

At 16:58 20/8/2007, you wrote:


Se fosse alfa  3/4 , então poderiamos tomar alfa = 0 por exemplo, mas
para alfa = 0 a equação não admite 4 raizes distintas, tem alguma coisa
errada.
Abs.

Rivaldo





sabendo que zb=conjugado de z
 z*zb=modz^2
 entao temos
 (z/modz)^2=a*(1+i)

 z/modz=cosc+isenc
 cos2c+isen2c=a(1+i)
 cos2c=sen2 c=a
 -1=a=1
 c=pí/8+npi
 a=+-rq2/2
 a melhor resposta e a letra a, a3/4 e diferente de 1/2.

 On 8/16/07, [EMAIL PROTECTED] [EMAIL PROTECTED] wrote:

 Desculpe prof Nehab e  galera.Quem puder ajudar eu agradeço,
 Considere  Z^2 = alfa* Z(1+i)* z(conjugado de z),onde alfa eh um numero
 real.Determine alfa de modo que a equacao tenha 4 raizes distintas.
 alternativas
 a)alfa 3/4,alfa diferente 1/2
 b) alfa  4/5
 c) alfa diferente 1/2
 d) alfa = -1,5 ou alfa  =1,5
 e) ALFA =2
 galera não tenho gabarito,o q vcs puderem ajudar eu agradeco


 Atenciosamente

 Wellington Silva
  --
 *Check Out the new free AIM(R)
 
Mail*http://pr.atwola.com/promoclk/100122638x1081283466x1074645346/aol?redir=http%3A%2F%2Fwww%2Eaim%2Ecom%2Ffun%2Fmail%2F--

 Unlimited storage and industry-leading spam and email virus protection.





=
Instruções para entrar na lista, sair da lista e usar a lista em
http://www.mat.puc-rio.br/~nicolau/olimp/obm-l.html
=


Re: [obm-l] Re: [obm-l] Combinat�ria e Probabilidade

2007-08-18 Por tôpico Carlos Eddy Esaguy Nehab

Oi, Pedro e Salhab,

Para minimizar f(n), Pedro, basta olhar o valor de f(n+1)/f(n)  e ver 
quando esta joça é maior que 1.   Ai você vê o crescimento de f(n):

f(n+1) / f(n) = 3n/(60-n)  1  ---  n  15 que é o que você queria, eu acho.

Mas eu não entendi o problema ou o que vocês estão falando é 
simplesmente a distribuição (de probabilidade) chamada de 
binomial.   O valor esperado da distribuição binomial é np, ou 
seja,  60 . 1/4 = 15.


Vejamos: a probabilidade de você acertar k questões é, como Pedro já 
escreveu, P(k) = C(n,k) p^k (1-p)^(n-k).


Desejamos então a média desta distribuição, ou seja,  o somatório [ k 
. P(k) ], k de 0 a n


Mas este somatório é simples (veja em 
http://en.wikipedia.org/wiki/Binomial_distribution): usa apenas 
macetezinho bobo de combinações... para se provar que vale np.


Abraços,
Nehab


At 02:22 18/8/2007, you wrote:
Salhab, primeiro obrigado por tentar resolver o problema. Segundo, 
vou procurar te mostrar até onde cheguei, para ver se você consegue, 
porque conhece muito mais do que eu, solucionar de vez a questão.


A chance de se acertar n questões - P(n) - é igual a (1/4)^n * 
(3/4)^(60-n) * C(60,n). Esse fator
C(60,n) entra porque não foi estabelecida nenhuma ordem de acerto. 
Reescrevendo, separando o que varia do que é constante, temos:


P(n) = (1/4)^n * (3/4)^(60-n) * C(60,n) = (1* 3^60 * 60!) / [4^60 * 
3^n * n! * (60-n)!]
Veja que, dessa forma, o numerador é constante e somente uma parte 
do denominador é variável.


P(n) = (1* 3^60 * 60!) / [4^60 * f(n)], onde f(n) = 3^n * n! * (60-n)!

O problema passa a ser minimizar f(n), n variando de 0 a 60. Para 
a+b = 60, ab, C(60,a) = C(60,b), mas 3^a  3^b. Fica bem óbvio, 
então (embora isso já fosse algo intuitivo), que só temos de testar 
os valores até n = 30. Para n = 31, por exemplo, f(29)  f(31) = P(29)P(31).


Sobre intuitivmente acertarmos 1 questão a cada quatro... Vamos 
supor uma prova composta de 4 questões, cada uma com quatro 
alternativas. Nesse caso, f(n) = 3^n * n! * (4-n)!, e só precisamos 
testar até n = 2.


Testando n=1... f(1) = (3 * 1! * 3!) = 18
Testando n=2... f(2) = (3^2 * 2! * 2!) = 36.

De fato, acertar uma questão é o mais provável. Acertar 15 de 60 
também seria portanto o resultado mais provável para a UERJ. Acho, 
aliás, que eu poderia supor ser essa prova de 60 questões a junção 
de 15 provas de 4 questões. E, testando alguns valores, lembrando 
que f(n) tem de ser minimizado, temos:


f(14) = 3^14 * 14! * 46!
f(15) = 3^15 * 15! * 45! = f(14)*45/46
f(16) = 3^16 * 16! * 44! = f(14)*48/46

f(15)f(14)f(16), o que faz sentido. A chance deve crescer de 1 até 
15 e descrescer de 15 até 60.


Mas eu ainda queria saber como minimizar f(n) = 3^n * n! * (60-n)!

Grato,

Pedro Lazéra Cardoso

_
Mande torpedos SMS do seu messenger para o celular dos seus amigos 
http://mobile.msn.com/


=
Instruções para entrar na lista, sair da lista e usar a lista em
http://www.mat.puc-rio.br/~nicolau/olimp/obm-l.html
=


Re: [obm-l] Re: [obm-l] Combinat�ria e Probabilidade

2007-08-18 Por tôpico Carlos Eddy Esaguy Nehab

Oi,

Corrigindo (me atrapalhei nas contas - fui fazer mentalmente e errei):
f(n+1) / f(n) = 3(n+1)/(60-n)  1  ---  n  14,25  ---  n = 15
Nehab

At 08:33 18/8/2007, you wrote:

Oi, Pedro e Salhab,

Para minimizar f(n), Pedro, basta olhar o valor de f(n+1)/f(n)  e 
ver quando esta joça é maior que 1.   Ai você vê o crescimento de f(n):
f(n+1) / f(n) = 3n/(60-n)  1  ---  n  15 que é o que você queria, 
eu acho.


Mas eu não entendi o problema ou o que você estão falando é 
simplesmente a distribuição (de probabilidade) chamada de 
binomial.   O valor esperado da distribuição binomial é np, ou 
seja,  60 . 1/4 = 15.


Vejamos: a probabilidade de você acertar k questões é, como Pedro já 
escreveu, P(k) = C(n,k) p^k (1-p)^(n-k).


Desejamos então a média desta distribuição, ou seja,  o somatório [ 
k . P(k) ], k de 0 a n


Mas este somatório é simples (veja em 
http://en.wikipedia.org/wiki/Binomial_distribution): usa apenas 
macetezinho bobo de combinações... para se provar que vale np.


Abraços,
Nehab


At 02:22 18/8/2007, you wrote:
Salhab, primeiro obrigado por tentar resolver o problema. Segundo, 
vou procurar te mostrar até onde cheguei, para ver se você 
consegue, porque conhece muito mais do que eu, solucionar de vez a questão.


A chance de se acertar n questões - P(n) - é igual a (1/4)^n * 
(3/4)^(60-n) * C(60,n). Esse fator
C(60,n) entra porque não foi estabelecida nenhuma ordem de acerto. 
Reescrevendo, separando o que varia do que é constante, temos:


P(n) = (1/4)^n * (3/4)^(60-n) * C(60,n) = (1* 3^60 * 60!) / [4^60 * 
3^n * n! * (60-n)!]
Veja que, dessa forma, o numerador é constante e somente uma parte 
do denominador é variável.


P(n) = (1* 3^60 * 60!) / [4^60 * f(n)], onde f(n) = 3^n * n! * (60-n)!

O problema passa a ser minimizar f(n), n variando de 0 a 60. Para 
a+b = 60, ab, C(60,a) = C(60,b), mas 3^a  3^b. Fica bem óbvio, 
então (embora isso já fosse algo intuitivo), que só temos de testar 
os valores até n = 30. Para n = 31, por exemplo, f(29)  f(31) = P(29)P(31).


Sobre intuitivmente acertarmos 1 questão a cada quatro... Vamos 
supor uma prova composta de 4 questões, cada uma com quatro 
alternativas. Nesse caso, f(n) = 3^n * n! * (4-n)!, e só precisamos 
testar até n = 2.


Testando n=1... f(1) = (3 * 1! * 3!) = 18
Testando n=2... f(2) = (3^2 * 2! * 2!) = 36.

De fato, acertar uma questão é o mais provável. Acertar 15 de 60 
também seria portanto o resultado mais provável para a UERJ. Acho, 
aliás, que eu poderia supor ser essa prova de 60 questões a junção 
de 15 provas de 4 questões. E, testando alguns valores, lembrando 
que f(n) tem de ser minimizado, temos:


f(14) = 3^14 * 14! * 46!
f(15) = 3^15 * 15! * 45! = f(14)*45/46
f(16) = 3^16 * 16! * 44! = f(14)*48/46

f(15)f(14)f(16), o que faz sentido. A chance deve crescer de 1 
até 15 e descrescer de 15 até 60.


Mas eu ainda queria saber como minimizar f(n) = 3^n * n! * (60-n)!

Grato,

Pedro Lazéra Cardoso

_
Mande torpedos SMS do seu messenger para o celular dos seus amigos 
http://mobile.msn.com/


=
Instruções para entrar na lista, sair da lista e usar a lista em
http://www.mat.puc-rio.br/~nicolau/olimp/obm-l.html
=


Re: [obm-l] revistas e publica��es on line - onde encontrar

2007-08-18 Por tôpico Carlos Eddy Esaguy Nehab

Oi, Johnson

Há varios sites onde você encontrará bons textos e/ou dicas em 
português.  Divirta-se:


Em primeiro lugar, no site da OBM. Artigos da revista Eureka e a 
revista inteira:

http://www.obm.org.br/frameset-eureka.htm

O site Majorando, de dois olímpicos, também é muito interessante.
http://majorando.com/
(vá em artigos: http://majorando.com/?page_id=12)

No site do grupo Teorema, muito bom:
http://www.grupoteorema.mat.br/

Um site que admiro mas que não conheço pessoalmente o autor, embora 
quase que só com coisas muito básicas (um trabalho sério, honesto e 
legal) - mas aparentemente pararam de atualizar.

http://pessoal.sercomtel.com.br/matematica/

O grupo rumoaoita também oferece alguns textos interessantes em seu 
site (embora seja de uma instituição com fins lucrativos)

www.rumoaoita.com/matematica.php

Bem, certamente estou esquecendo outras iniciativas importantes, mas 
os demais colegas poderão completar minha pequena lista.


Abraços,
Nehab


At 11:26 18/8/2007, you wrote:

Ola amigos !
Gostaria de pedir informações de voçes onde encontrar revistas e 
blublicações de artigos on line de matematica.
de preferencia em portugues por favor sou pessimo em ingles, mais se 
nao tiver jeito que seja em ingles mesmo.
Eu vi o pessoal falando de publicações dos responsaveis da lista so 
que nao sei onde encontra- las então gostaria do auxilio de voçes.

muito obrigado ;)


Re: [obm-l] ajuda em complexo

2007-08-16 Por tôpico Carlos Eddy Esaguy Nehab

OI,
Edite sua pergunta de outra forma.  Olhe o que eu recebi...
Nehab

At 17:14 15/8/2007, you wrote:

ei galera quem puder ajudar eu agradeço,
Considere a equação Z^2 = alfa*Z(1+i)*z(lê-se (conjugado de 
zê),onde alfa é um número real.Determine alfa de modo que a 
equação tenha 4 raízes distintas.

alternativas
a)alfa 3/4,alfa diferente 1/2
b) alfa  4/5
c) alfa diferente 1/2
d) alfa = -1,5 ou alfa  =1,5
e) ALFA =2
galera não tenho gabarito,o q vcs puderem ajudar eu agradeço

Atenciosamente

Wellington Silva

--
http://pr.atwola.com/promoclk/100122638x1081283466x1074645346/aol?redir=http%3A%2F%2Fwww%2Eaim%2Ecom%2Ffun%2Fmail%2FCheck 
Out the new free AIM(R) Mail -- Unlimited storage and 
industry-leading spam and email virus protection.


Re: RES: [obm-l] divisibilidade II

2007-08-16 Por tôpico Carlos Eddy Esaguy Nehab

Oi, Artur

Seja N o número que, na base 10, tem representação, a_n 
a_(n-1).a_0 e seja P o polinomio dadao por P(x) = a_n x^n + 
a_(n-1)x^(n-1)...+ a_0. Temos, entao, que N = P(10).

Ok
Sendo 0  k  10 um inteiro, então o teorema de Taylor, 
particularizado para polinômios, nos mostra que k|N se, e somente 
se, k | P(10 - k).
Você se refere à série de Taylor?  Não entendi o porque da série de 
Taylor justificar k | N sss k | P(10-k)  (se for óbvio, não tô vendo...:-)
Que isto é verdade eu concordo, pois a diferença entre P(10) e 
P(10-k)  é uma combinação linear inteira de expressões 10^p - 
(10-k)^p  que obviamente são divisíveis por k, pois a^p - b^p tem 
fator a-b = k).
No caso, k = 7 e nosso polinômio tem os 99 primeiros coeficientes 
iguais a 9 e o das unidades igual a 6. Como este numero e 3^100 - 4, ...

Você quis dizer  10^100 - 4, certamente.
que vimos ser divisivel por 7, segue-se que P(3) é divisível por 7. 
E temos que P(3) = 9 *3^99 + 9 * 3^98.+ 9 * 3 + 6 =o 9 (3^100 - 
3)/2 + 6 =   (9 *3^100 - 27 + 12)/2 =   (9 *3^100 - 15)/2 = (3(3^101 
- 5)/2. Logo, este número é divisível por 7. Pode ser cultura um 
tanto inútil, mas achei isso legal.


Também achei legal.  Apenas realmente não entendi como você enxergou 
sua afirmativa pensando na série de Taylor.


Obrigado pelas dicas

Abração,
Nehab


Artur



-Mensagem original-
De: [EMAIL PROTECTED] [mailto:[EMAIL PROTECTED] 
nome de Carlos Eddy Esaguy Nehab

Enviada em: quarta-feira, 15 de agosto de 2007 22:28
Para: obm-l@mat.puc-rio.br
Assunto: Re: [obm-l] divisibilidade II

Oi, Arthur,

De fato  3^101 - 5  é divisível por 7 mas não consegui enxergar a 
relação deste fato com a dica que eu havia dado para o 
Francisco?  Pode me explicar melhor ?


Só consegui ver que  7  divide  3^101 - 5  usando aritmética 
modular.   Acho que você sacou alguma coisa que eu não ví...


Abração,
Nehab

PS:
O que fiz:  3^6  = 729 = 1 (mod 7)  ---  3^96 = 1^16 = 1 (mod 7); 
mas 3^5 = 243 = 5 (mod 7); então  3^101 = 5  (mod 7).


At 18:03 15/8/2007, you wrote:
E como decorrencia disto, segue-se que (3 (3^101 - 5))/2 eh 
divisivel por 7. Certo?

Artur



 -Mensagem original-
De: [EMAIL PROTECTED] [ 
mailto:[EMAIL PROTECTED] nome de Carlos Eddy Esaguy Nehab

Enviada em: quarta-feira, 15 de agosto de 2007 17:14
Para: obm-l@mat.puc-rio.br
Assunto: Re: [obm-l] divisibilidade

Oi, Francisco,
O correto é 10^100 - 4  e não 10^100 - 6.
Tipicamente estes exercícios devem ser resolvidos usando 
módulo.   Mas este, em especial, dá pra fazer até diretamente...

Solução 1)
Note que o 10^100 - 4 é um monte de noves (ou seja, 99 noves) 
terminando com um 6, correto?
Mas cada grupo de seis noves (99) é divisível por 7 dando 
142857.   Após os 96 primeiros algarimos (do dividendo) você terá 
obtido no quociente 16 vezes a seqüência 142857 e sobrariam os 
algarismos 9996 para terminar a divisão.

Mas 9996 é divisível por 7 dando 1428.
Solução 2)
Note a seguinte propriedade (pode prová-la: é um exercício simples 
e elegante):
Seja N = (Mr), ou seja, os algarismos iniciais de N compõem o 
número M e seu último algarismo (de N) é r.

Então N é divisívível por 7 sss  M - 2r é divisível por  7.
Usando esta propriedade também dá para resolver seu problema (tente).
Abraços,
Nehab
PS: Deixo a solução por módulo para os demais colegas.
Abraços,
Nehab


At 15:39 15/8/2007, you wrote:

Como mostro que 7 | (10^100 - 6)  ?

Grato.


--
Receba GRÁTIS as mensagens do Messenger no seu celular quando você 
estiver offline. Conheça o MSN Mobile! 
http://mobile.live.com/signup/signup2.aspx?lc=pt-brCadastre-se já!


Re: [obm-l] ajuda em complexo

2007-08-16 Por tôpico Carlos Eddy Esaguy Nehab

Oi, Jones,

Ainda não entendi sua expressão, pois acho que é z^2 = alfa. 
z'.(1+i), onde z'é o conjugado de z, mas neste caso não há reposta certa...


Seja o que for sua expressão ai vai o caminho das pedras... (se você 
conhecer o tal do cis teta = cos teta + i sen teta...)


1) Seja z = r cis teta;
2) Então é imediato que
z^2 = r^2 cis (2.teta),
z'= r cis (-teta);
3) Como 1+ i = raiz(2) cis pi/4,
substituindo e simplificando, temos:
r^2 cis(3teta-pi/4) = r. raiz(2) .alfa.

Como alfa é real, é necessário que cis (3teta-pi/4) seja real (ou r = 
0), ou seja: sen (3teta - pi/4) =0.
Ai, seu z = 0 ou  z = r cos (3teta-pi/4) para sen (3teta-pi/4) = 0 
seriam as possíveis raízes.
Como você deseja 4 soluções ao todo, seria necessário que houvesse 3 
valores diferentes de cos (3teta-pi/4)
tal que sen (3teta-pi/4) = 0 o que não dá pois cos teta  só pode 
valer (neste caso) +1 ou -1


Se eu não dei bobeira...

Abraços,
Nehab


At 16:31 16/8/2007, you wrote:

Quem é o conjugado de quem?

como pode dizer que z é conjugado de z?

além disso, se for z* é o conjugado de z, porque ele conjuga alfa* e 
logo depois diz que ele é um número real?


Sugestão: Reedite a sua fórmula.
t+
Jones



On 8/16/07, mailto:[EMAIL PROTECTED][EMAIL PROTECTED] 
mailto:[EMAIL PROTECTED][EMAIL PROTECTED]  wrote:

Desculpe prof Nehab e  galera.Quem puder ajudar eu agradeço,
Considere  Z^2 = alfa* Z(1+i)* z(conjugado de z),onde alfa eh um 
numero real.Determine alfa de modo que a equacao tenha 4 raizes distintas.

alternativas
a)alfa 3/4,alfa diferente 1/2
b) alfa  4/5
c) alfa diferente 1/2
d) alfa = -1,5 ou alfa  =1,5
e) ALFA =2
galera não tenho gabarito,o q vcs puderem ajudar eu agradeco

Atenciosamente

Wellington Silva

--
http://pr.atwola.com/promoclk/100122638x1081283466x1074645346/aol?redir=http%3A%2F%2Fwww%2Eaim%2Ecom%2Ffun%2Fmail%2FCheck 
Out the new free AIM(R) Mail -- Unlimited storage and 
industry-leading spam and email virus protection.




Re: [obm-l] ajuda em complexo

2007-08-16 Por tôpico Carlos Eddy Esaguy Nehab

Perdão, na última linha, leia-se .. não dá pois cos (3teta-pi/4) ..
Nehab


At 18:44 16/8/2007, you wrote:

Oi, Jones,

Ainda não entendi sua expressão, pois acho que é z^2 = alfa. 
z'.(1+i), onde z'é o conjugado de z, mas neste caso não há reposta certa...


Seja o que for sua expressão ai vai o caminho das pedras... (se você 
conhecer o tal do cis teta = cos teta + i sen teta...)


1) Seja z = r cis teta;
2) Então é imediato que
z^2 = r^2 cis (2.teta),
z'= r cis (-teta);
3) Como 1+ i = raiz(2) cis pi/4,
substituindo e simplificando, temos:
r^2 cis(3teta-pi/4) = r. raiz(2) .alfa.

Como alfa é real, é necessário que cis (3teta-pi/4) seja real (ou r 
= 0), ou seja: sen (3teta - pi/4) =0.
Ai, seu z = 0 ou  z = r cos (3teta-pi/4) para sen (3teta-pi/4) = 0 
seriam as possíveis raízes.
Como você deseja 4 soluções ao todo, seria necessário que houvesse 3 
valores diferentes de cos (3teta-pi/4)
tal que sen (3teta-pi/4) = 0 o que não dá pois cos teta  só pode 
valer (neste caso) +1 ou -1


Se eu não dei bobeira...

Abraços,
Nehab


At 16:31 16/8/2007, you wrote:

Quem é o conjugado de quem?

como pode dizer que z é conjugado de z?

além disso, se for z* é o conjugado de z, porque ele conjuga alfa* 
e logo depois diz que ele é um número real?


Sugestão: Reedite a sua fórmula.
t+
Jones



On 8/16/07, mailto:[EMAIL PROTECTED][EMAIL PROTECTED] 
mailto:[EMAIL PROTECTED][EMAIL PROTECTED]  wrote:

Desculpe prof Nehab e  galera.Quem puder ajudar eu agradeço,
Considere  Z^2 = alfa* Z(1+i)* z(conjugado de z),onde alfa eh um 
numero real.Determine alfa de modo que a equacao tenha 4 raizes distintas.

alternativas
a)alfa 3/4,alfa diferente 1/2
b) alfa  4/5
c) alfa diferente 1/2
d) alfa = -1,5 ou alfa  =1,5
e) ALFA =2
galera não tenho gabarito,o q vcs puderem ajudar eu agradeco
Atenciosamente
Wellington Silva
--
http://pr.atwola.com/promoclk/100122638x1081283466x1074645346/aol?redir=http%3A%2F%2Fwww%2Eaim%2Ecom%2Ffun%2Fmail%2FCheck 
Out the new free AIM(R) Mail -- Unlimited storage and 
industry-leading spam and email virus protection.


Re: [obm-l] divisibilidade

2007-08-15 Por tôpico Carlos Eddy Esaguy Nehab

Oi, Francisco,

O correto é 10^100 - 4  e não 10^100 - 6.

Tipicamente estes exercícios devem ser resolvidos usando 
módulo.   Mas este, em especial, dá pra fazer até diretamente...


Solução 1)
Note que o 10^100 - 4 é um monte de noves (ou seja, 99 noves) 
terminando com um 6, correto?


Mas cada grupo de seis noves (99) é divisível por 7 dando 
142857.   Após os 96 primeiros algarimos (do dividendo) você terá 
obtido no quociente 16 vezes a seqüência 142857 e sobrariam os 
algarismos 9996 para terminar a divisão.

Mas 9996 é divisível por 7 dando 1428.

Solução 2)
Note a seguinte propriedade (pode prová-la: é um exercício simples e elegante):

Seja N = (Mr), ou seja, os algarismos iniciais de N compõem o número 
M e seu último algarismo (de N) é r.

Então N é divisívível por 7 sss  M - 2r é divisível por  7.

Usando esta propriedade também dá para resolver seu problema (tente).

Abraços,
Nehab

PS: Deixo a solução por módulo para os demais colegas.

Abraços,
Nehab



At 15:39 15/8/2007, you wrote:

Como mostro que 7 | (10^100 - 6)  ?

Grato.


--
Receba GRÁTIS as mensagens do Messenger no seu celular quando você 
estiver offline. Conheça o MSN Mobile! 
http://mobile.live.com/signup/signup2.aspx?lc=pt-brCadastre-se já!


Re: [obm-l] divisibilidade II

2007-08-15 Por tôpico Carlos Eddy Esaguy Nehab

Oi, Arthur,

De fato  3^101 - 5  é divisível por 7 mas não consegui enxergar a 
relação deste fato com a dica que eu havia dado para o 
Francisco?  Pode me explicar melhor ?


Só consegui ver que  7  divide  3^101 - 5  usando aritmética 
modular.   Acho que você sacou alguma coisa que eu não ví...


Abração,
Nehab

PS:
O que fiz:  3^6  = 729 = 1 (mod 7)  ---  3^96 = 1^16 = 1 (mod 7); 
mas 3^5 = 243 = 5 (mod 7); então  3^101 = 5  (mod 7).


At 18:03 15/8/2007, you wrote:
E como decorrencia disto, segue-se que (3 (3^101 - 5))/2 eh 
divisivel por 7. Certo?

Artur



 -Mensagem original-
De: [EMAIL PROTECTED] [mailto:[EMAIL PROTECTED] 
nome de Carlos Eddy Esaguy Nehab

Enviada em: quarta-feira, 15 de agosto de 2007 17:14
Para: obm-l@mat.puc-rio.br
Assunto: Re: [obm-l] divisibilidade

Oi, Francisco,

O correto é 10^100 - 4  e não 10^100 - 6.

Tipicamente estes exercícios devem ser resolvidos usando 
módulo.   Mas este, em especial, dá pra fazer até diretamente...


Solução 1)
Note que o 10^100 - 4 é um monte de noves (ou seja, 99 noves) 
terminando com um 6, correto?


Mas cada grupo de seis noves (99) é divisível por 7 dando 
142857.   Após os 96 primeiros algarimos (do dividendo) você terá 
obtido no quociente 16 vezes a seqüência 142857 e sobrariam os 
algarismos 9996 para terminar a divisão.

Mas 9996 é divisível por 7 dando 1428.

Solução 2)
Note a seguinte propriedade (pode prová-la: é um exercício simples e 
elegante):


Seja N = (Mr), ou seja, os algarismos iniciais de N compõem o número 
M e seu último algarismo (de N) é r.

Então N é divisívível por 7 sss  M - 2r é divisível por  7.

Usando esta propriedade também dá para resolver seu problema (tente).

Abraços,
Nehab

PS: Deixo a solução por módulo para os demais colegas.

Abraços,
Nehab



At 15:39 15/8/2007, you wrote:

Como mostro que 7 | (10^100 - 6)  ?

Grato.


--
Receba GRÁTIS as mensagens do Messenger no seu celular quando você 
estiver offline. Conheça o MSN Mobile! 
http://mobile.live.com/signup/signup2.aspx?lc=pt-brCadastre-se já!


Re: [obm-l] �lgebra Linear

2007-08-14 Por tôpico Carlos Eddy Esaguy Nehab

Poxa, André

Ninguém deu bola pra você 

Tentarei o primeiro...

Exercício 1
Sejam X e Y espaços vetoriais com a mesma dimensão finita. Suponha 
que, para as aplicações lineares T:X--Y e S:Y--X, seja verdadeiro 
ST = I, a identidade em X. Mostre que S = T^-1 .


Estratégia: A questão na verdade se resume a provar que T é bijetora, 
pois se uma função T bijetora (independente de ser função linear ou 
não) é tal que existe S com ST= I,  tal S é necessariamente sua 
inversa (ou seja, sua inversa à esquerda S é também sua inversa à direita).


a) T é injetora: dados x1 e x2 de X, devemos mostrar T(x1) = T(x2) 
implica x1 = x2.  Mas como S é função, S(T(x1)) = S(T(x2)); como a 
composta é a identidade, S(T(x1))  = x1 e S(T(x2)) = x2; logo,  x1 = x2.


b) T é sobrejetora
Seja u e v vetores linearmente independentes em X.  Então, se 
mostrarmos que T(u) e T(v) são linearmente independentes em Y será 
imediato que a imagem de uma base em X é uma base em Y (pois T é 
injetora e as dimensões de X e Y são iguais e finitas).
Seja T(u) = u'  e T(v) = v'. Para mostrar que  u'e v' são 
linearmente independentes basta mostrar que se  a.u'+ b.v'= 0  (onde 
a e b são escalares e 0 o vetor nulo)  necessariamente a=b=0.  Mas 
a.u'+b.v' = aT(u) +b T(v) = T(au+bv) (pois T é linear).  Mas se 
T(au+bv) = 0 = T(0), como T é injetora, au+bv=0 e como u e v são li,  a=b=0.


Agora imagine  u1,u2,...uk uma base de X; como T(uk) é base em Y 
podemos expressar qualquer y de Y como combinação linear de T(uk): 
a1T(u1)+a2T(u2)+...+anT(un), por exemplo.  Mas é imediato perceber 
que a imagem do vetor a1.u1+a2.u2+...an.un é exatamente y...

.
Deixo como exercício para você mostrar a afirmativa contida na 
estrategia, pois a parte que depende de álgebra linear está feita...


Quanto ao exercício 2 mais tarde darei uma pensada, mas espero que 
alguém poste logo solução ... pois ainda ando enferrujado...


Abraços,
Nehab

PS: espero não haver deslizes no texto acima...:-)


At 13:05 11/8/2007, you wrote:
Olá pessoal, dêem uma ajuda nesses problemas abaixo. O primeiro 
parece óbvio demais, mas o que usar para demonstrar este resultado 
simples? O segundo já é de dificuldade um pouco maior.


Abraços,


1 - Sejam X e Y espaços vetoriais com a mesma dimensão finita. 
Suponha que, para as aplicações lineares T:X--Y e S:Y--X, seja 
verdadeiro ST = I, a identidade em X. Mostre que S = T^-1 .


---

2 - Sejam X um espaço vetorial real de dimensão finita e B uma base 
de X. Seja também T:XxX--R uma forma bilinear. Mostre que existe 
uma matriz A tal que


T(h, k) = [k]_B^t A [h]_B

Se X for um espaço com produto interno, mostre que existe uma 
aplicação linear S:X--X tal que A é a representação se S^t na base 
ortongonal B. Mostre que B é simétrica se, e somente se, A for simétrica.


---


--
André Rodrigues da Cruz


Re: [obm-l] Re: [obm-l] BETONEIRA e a m�dia harm�nica.... off-topic

2007-08-13 Por tôpico Carlos Eddy Esaguy Nehab

Caro Fernando,

E eu que achava que tinha alguma chance de deter o título de quase o 
mais antigo da lista...   Nem com o quase.  Tola pretensão...


Gostaria de fazer apenas um comentário (além de naturalmente ter me 
sentido lisongeado pela citação à navalha de Occan, esperando que os 
mais jovens não pensem se tratar de um concorrente da Gillete...).


Como esta lista é aberta, é natural que diversos temas mais 
especializados não possam ser abordados de forma a que todos o 
comprendam.  Eu mesmo, embora tenha tido formação em Matemática (além 
da Engenharia Elétrica) ando de fato enferrujado.  Já houve época (há 
uns 35/40 anos, quando eu frequentava o IMPA) em que eu dominava para 
valer assuntos como Análise Real, Topologia, Álgebra Linear (de 
verdade...), Álgebra (estruturas, não a álgebra colegial), etc.


Mas, a verdade é que voltei a estudar, depois do meu último retorno à 
esta lista e isto foi ótimo.Espero voltar a dominar pelo menos 
uns 50% do que eu dominava...  Mas não tenha dúvidas, esta lista é de 
longe a melhor lista da qual participo e se você (não vou chamá-lo de 
senhor!) desejar, poderemos conversar fora da Lista, e não será por 
falta de problemas que ficará triste... :-)


Grande Abraço
Nehab

PS:
Nicolau e Santa Rita já responderam a sua indagação, com  enfoques 
diferentes, enriquecendo a todos.  Eu de minha parte, não consegui 
compreender se você usa o termo aleatório para significar sem lei 
de formação aparente (e foram por ai as respostas de ambos) ou em 
algum outro sentido.


At 09:53 13/8/2007, you wrote:

Caro Ojesed
Concordo com você quando diz que  Muitas vezes eu vejo assuntos 
sendo tratados aqui, que é uma lista aberta, com uma linguagem 
despreocupada em atingir o máximo de pessoas possível.   De certa 
maneira esta atitude inibe o  debate.
A mim parece que a lista engloba o que há de melhor na matemática 
brasileira.  Mas também concordo com o juizo que faz do Nehab, e 
enfatizo  sua capacidade de transmitir  conhecimento, de 
solucionar  problemas, com o mínimo de palavras e o máximo de 
clareza.  Como se usasse a navalha de Occan duas vezes, a primeira 
para barbear e a segunda para escanhoar.
Não sou matemático, sou um engenheiro aposentado, tenho 82 anos, mas 
gosto de garimpar pela lista, procurando problemas que estejam ao 
meu alcance, que , (devo admitir), são poucos.
Uma vez manisfestei estranheza  quanto a ausência de qualquer 
referência aos númeroos aleatários, em clássicos de análise  que 
tratam da teoria do número real.  Não obtive resposta. No entanto, 
aparentemente, esses números são os principais atores que justificam 
a não enumerabilidade dos números reais. Isso porque os inteiros, os 
racionais e os algébricos são enumerãveis como também 
os  transcendentes que requerem um algoritmo finito para sua 
descrição. Pergunto, o que resta são os números aleatórios? Existe 
uma abordagem para esses números fora do  cálculo das probablidades?

Sds
Fernando A Candeias



Em 10/08/07, Ojesed Mirror 
mailto:[EMAIL PROTECTED][EMAIL PROTECTED]  escreveu:
Olá Nehab, você é um educador como poucos pois consegue tratar 
assuntos desde o mais elementar, como este, até os que estão na 
fronteira do conhecimento com uma clareza invejável.


Muitas vezes eu vejo assuntos sendo tratados aqui, que é uma lista 
aberta, com uma linguagem despreocupada em atingir o máximo de 
pessoas possível.


Eu fico na dúvida se estou diante de pessoas que estão somente 
exibindo conhecimento ou se os assuntos estão sendo tratados da 
forma mais palatável possível e eu é que estou muito defasado da 
turma. Geralmente fico com a segunda opção.


O fato é que uma minoria de pessoas geniais participam ativamente 
das discussões e a maioria observa.
Se isto não fere o objetivo maior da existência desta lista, 
desculpem minha preocupação.


Sds, Ojesed
- Original Message -
From: mailto:[EMAIL PROTECTED]Carlos Eddy Esaguy Nehab
To: mailto:obm-l@mat.puc-rio.brobm-l@mat.puc-rio.br
Sent: Friday, August 10, 2007 9:00 AM
Subject: Re: [obm-l] BETONEIRA e a média harmônica


Oi, Ojesed ,

At 02:04 10/8/2007, you wrote:
Seria correto dizer que se as massas não fossem iguais a resposta 
seria a média harmônica ponderada, com as massas sendo os ponderadores ?


Sim, vale...  Veja:

A média harmônica das densidades, ponderadas pelas massas é, por 
definição: o inverso da média aritmética ponderada (pesos m1 e m2) 
dos inversos das densidades d1 e d2.  Ou seja:


É o inverso de  [ m1 x (1/d1) + m2 x (1/d2) ]  /  (m1 + m2)
que vale (m1+m2) / [ (m1/d1 + m2/d2) ]

Mas esta expressão é exatamente a densidade média, pois é a massa 
total (m1+m2) dividida pelo volume total  (m1/d1 + m2/d2).


Abraços,
Nehab


- Original Message -
From: mailto:[EMAIL PROTECTED]Carlos Eddy Esaguy Nehab
To: mailto:obm-l@mat.puc-rio.brobm-l@mat.puc-rio.br
Sent: Thursday, August 09, 2007 2:17 AM
Subject: Re: [obm-l] BETONEIRA
Oi, Arkon, Ponce e Desejo...
Já que o Desejo (Ojesed Mirror) deu esta

Re: [obm-l] EQUA��O POLINOMIAL - AJUDA

2007-08-12 Por tôpico Carlos Eddy Esaguy Nehab

Oi,

Equações do terceiro grau são em geral cretinas... E em geral (a 
menos de ter que encarar o caso geral, sem nenhuma posibilidade de 
usar de malandragem), se a solução for simples, deve ser cretina...


E de fato, uma solução cretina é observar, por inspeção, que 2 é 
raiz...   e ai acabou, pois você poderia dividir o polinômo por x - 2 
e obteria as duas raízes restantes.


Mas admitindo que você não tivesse percebido isto, outro caminho 
(também cretino) seria perceber que a expressão P(x) = x^3 - 14x^2 + 
49x - 50, é curiosa...  A parte x^3 - 14x^2 + 49x  tá muito de 
bandeja, pois lembra (x - 7)^2, né...  Logo, a equação é x(x-7)^2 = 50...
Mais uma vez testando a possibilidade de x ser inteiroobtemos x = 
2 ou 5 ou 25 ou 50... e aí, obtemos outra vez o x = 2... e acabou-se.


Se definitivamente você acreditar que M+raiz(N) sendo raiz, seria 
ótimo se M-raiz(N) também fosse... supondo que a terceria raiz é P o 
produto seria  (M^2 - N).P = 50...   Mas uma vez tentando valores 
inteiros... P = 2 é uma...  (que eu acho que é o que o examinador 
deve ter pensado)


Finalmente, se nada de bom acontecesse, teríamos que encarar o caso 
geral que é clássico, irritante mas... para quem adora cúbicas como 
eu, não perco a oportunidade de sugerir este belo artigo que analisa 
de forma muito elegante a geometria das cúbicas:

http://www.google.com.br/url?sa=tct=rescd=1url=http%3A%2F%2Fwww.m-a.org.uk%2Fdocs%2Flibrary%2F2059.pdfei=Qzq_Rv2xBZ6aeYaM5YkLusg=AFQjCNEQR-gED20XTxcQRKth3xLmjazzSgsig2=w5mVP_Sa2qQAgunsJO_vzQ

Abraços,
Nehab

At 09:28 12/8/2007, you wrote:

EXISTE UMA RAIZ DA FORMA: M + RAIZ QUADRADA DE N, ACHE-A.

x^3 - 14x^2 + 49x - 50 = 0



=
Instruções para entrar na lista, sair da lista e usar a lista em
http://www.mat.puc-rio.br/~nicolau/olimp/obm-l.html
=


Re: [obm-l] BETONEIRA e a m�dia harm�nica....

2007-08-10 Por tôpico Carlos Eddy Esaguy Nehab

Oi, Ojesed ,

At 02:04 10/8/2007, you wrote:
Seria correto dizer que se as massas não fossem iguais a resposta 
seria a média harmônica ponderada, com as massas sendo os ponderadores ?


Sim, vale...  Veja:

A média harmônica das densidades, ponderadas pelas massas é, por 
definição: o inverso da média aritmética ponderada (pesos m1 e m2) 
dos inversos das densidades d1 e d2.  Ou seja:


É o inverso de  [ m1 x (1/d1) + m2 x (1/d2) ]  /  (m1 + m2)
que vale (m1+m2) / [ (m1/d1 + m2/d2) ]

Mas esta expressão é exatamente a densidade média, pois é a massa 
total (m1+m2) dividida pelo volume total  (m1/d1 + m2/d2).


Abraços,
Nehab


- Original Message -
From: mailto:[EMAIL PROTECTED]Carlos Eddy Esaguy Nehab
To: mailto:obm-l@mat.puc-rio.brobm-l@mat.puc-rio.br
Sent: Thursday, August 09, 2007 2:17 AM
Subject: Re: [obm-l] BETONEIRA

Oi, Arkon, Ponce e Desejo...

Já que o Desejo (Ojesed Mirror) deu esta ótima resposta, fica aqui 
uma dica, pois problemas desta natureza já apareceram diversas vezes 
por aqui...


Quando se introduz o conceito de médias (mesmo na 6 ou 7 séries) é 
extremamente oportuno sugerir contextos onde elas ocorrem (para não 
parecer um negócio artificial) e a resposta do Ojesed mostra que ele 
já adquiriu a malicia que eu acho legal.


Na Física a média harmônica ocorre com freqüência, pois ela é usual 
em todas as situações onde a grandeza da qual se deseja calcular a 
média é o quociente entre duas variáveis  e 
vejamos:  Velocidade é distância / tempo...  Densidade é massa / 
volume, resistência = voltagem/ corrente ...   Logo, se desejamos 
calcular velocidade média, densidade média, resistência equivalente, 
fatalmente a média harmônica entra na jogada (caso os valores das 
distâncias, volumes ou voltagens sejam iguais e isto ocorre na 
ligação em paralelo - como as voltagens não se somam, a resistência 
equivalente é o dobro da média harmônica...), posto que velocidade 
média = dist total / tempo total; densidade  final = massa total / 
volume total... e resistencia = mesma voltagem / corrente total


Vejamos um exemplinho  clássico (o outro é o do Arkon, posto que se 
misturam iguais quantidades de MASSA...)


Você vai a 60 km por hora num trecho de estrada e no mesmo trecho 
volta a 90 km/h.  Qual sua velocidade média?


Ora, você está querendo medir velocidade média, mas a variável 
chave, que é o tempo, está no denominador das velocidades e as 
duas distâncias, de ida e de volta são iguais ...


Logo a velocidade média (vm) é a média harmônica.

Veja: vm =  distância total / tempo total = (d1 + d2) / (t1 + t2)   (A)

Ocorre que t1 =  d1 /v1  e  t2 = d2/v2

Levando estas expressoes em (A)  voce obtem
dist total / tempo total = (x + x) /  [x/v1 + 
x/v2]  =  2v1.v2  (v1+v2)   que é a média harmônica...


Abraços,
Nehab


At 22:39 8/8/2007, you wrote:

A densidade total é a média harmônica das densidades parciais.
- Original Message -
From: mailto:[EMAIL PROTECTED]arkon
To: mailto:obm-l@mat.puc-rio.brobm-l
Sent: Wednesday, August 08, 2007 10:49 AM
Subject: [obm-l] BETONEIRA

Alguém pode resolver esta, por favor:


Uma betoneira está sendo preparada para produzir concreto. Receberá 
iguais quantidades, em massa, de areia, cimento e brita de 
densidades iguais a 1,5; 1,3 e 2,4, respectivamente. Calcular a 
densidade do concreto que vai ser produzido pela betoneira. 
Multiplique o resultado por 10 e despreze a parte fracionária, caso exista.



DESDE JÁ MUITO OBRIGADO
--
No virus found in this incoming message.
Checked by AVG Free Edition.
Version: 7.5.476 / Virus Database: 269.11.8/941 - Release Date: 
7/8/2007 16:06



--
No virus found in this incoming message.
Checked by AVG Free Edition.
Version: 7.5.476 / Virus Database: 269.11.8/941 - Release Date: 7/8/2007 16:06


Re: [obm-l] Re: [obm-l] BETONEIRA e a m�dia harm�nica....

2007-08-10 Por tôpico Carlos Eddy Esaguy Nehab

Oi, Ojesed

Fico muito orgulhoso (de verdade) pelo comentário, pois meu grande 
barato é de fato a sala de aula  e o resolver problemas está em 
segundo lugar (papai do céu é esperto, pois me deu muito mais aptidão 
para ensinar do que para resolver problemas).


Mas ainda bem que você e outros suprem minhas dificuldades aqui e 
acolá, pois o que eu não consigo fazer vocês da Lista conseguem.


Um grande abraço,
Nehab


At 17:06 10/8/2007, you wrote:
Olá Nehab, você é um educador como poucos pois consegue tratar 
assuntos desde o mais elementar, como este, até os que estão na 
fronteira do conhecimento com uma clareza invejável.


Muitas vezes eu vejo assuntos sendo tratados aqui, que é uma lista 
aberta, com uma linguagem despreocupada em atingir o máximo de 
pessoas possível.


Eu fico na dúvida se estou diante de pessoas que estão somente 
exibindo conhecimento ou se os assuntos estão sendo tratados da 
forma mais palatável possível e eu é que estou muito defasado da 
turma. Geralmente fico com a segunda opção.


O fato é que uma minoria de pessoas geniais participam ativamente 
das discussões e a maioria observa.
Se isto não fere o objetivo maior da existência desta lista, 
desculpem minha preocupação.


Sds, Ojesed
- Original Message -
From: mailto:[EMAIL PROTECTED]Carlos Eddy Esaguy Nehab
To: mailto:obm-l@mat.puc-rio.brobm-l@mat.puc-rio.br
Sent: Friday, August 10, 2007 9:00 AM
Subject: Re: [obm-l] BETONEIRA e a média harmônica

Oi, Ojesed ,

At 02:04 10/8/2007, you wrote:
Seria correto dizer que se as massas não fossem iguais a resposta 
seria a média harmônica ponderada, com as massas sendo os ponderadores ?


Sim, vale...  Veja:

A média harmônica das densidades, ponderadas pelas massas é, por 
definição: o inverso da média aritmética ponderada (pesos m1 e m2) 
dos inversos das densidades d1 e d2.  Ou seja:


É o inverso de  [ m1 x (1/d1) + m2 x (1/d2) ]  /  (m1 + m2)
que vale (m1+m2) / [ (m1/d1 + m2/d2) ]

Mas esta expressão é exatamente a densidade média, pois é a massa 
total (m1+m2) dividida pelo volume total  (m1/d1 + m2/d2).


Abraços,
Nehab


- Original Message -
From: mailto:[EMAIL PROTECTED]Carlos Eddy Esaguy Nehab
To: mailto:obm-l@mat.puc-rio.brobm-l@mat.puc-rio.br
Sent: Thursday, August 09, 2007 2:17 AM
Subject: Re: [obm-l] BETONEIRA
Oi, Arkon, Ponce e Desejo...
Já que o Desejo (Ojesed Mirror) deu esta ótima resposta, fica aqui 
uma dica, pois problemas desta natureza já apareceram diversas 
vezes por aqui...
Quando se introduz o conceito de médias (mesmo na 6 ou 7 séries) é 
extremamente oportuno sugerir contextos onde elas ocorrem (para não 
parecer um negócio artificial) e a resposta do Ojesed mostra que 
ele já adquiriu a malicia que eu acho legal.
Na Física a média harmônica ocorre com freqüência, pois ela é usual 
em todas as situações onde a grandeza da qual se deseja calcular a 
média é o quociente entre duas variáveis  e 
vejamos:  Velocidade é distância / tempo...  Densidade é massa / 
volume, resistência = voltagem/ corrente ...   Logo, se desejamos 
calcular velocidade média, densidade média, resistência 
equivalente, fatalmente a média harmônica entra na jogada (caso os 
valores das distâncias, volumes ou voltagens sejam iguais e isto 
ocorre na ligação em paralelo - como as voltagens não se somam, a 
resistência equivalente é o dobro da média harmônica...), posto que 
velocidade média = dist total / tempo total; densidade  final = 
massa total / volume total... e resistencia = mesma voltagem / 
corrente total
Vejamos um exemplinho  clássico (o outro é o do Arkon, posto que se 
misturam iguais quantidades de MASSA...)


Você vai a 60 km por hora num trecho de estrada e no mesmo trecho 
volta a 90 km/h.  Qual sua velocidade média?
Ora, você está querendo medir velocidade média, mas a variável 
chave, que é o tempo, está no denominador das velocidades e as 
duas distâncias, de ida e de volta são iguais ...

Logo a velocidade média (vm) é a média harmônica.
Veja: vm =  distância total / tempo total = (d1 + d2) / (t1 + t2)   (A)
Ocorre que t1 =  d1 /v1  e  t2 = d2/v2
Levando estas expressoes em (A)  voce obtem
dist total / tempo total = (x + x) /  [x/v1 + 
x/v2]  =  2v1.v2  (v1+v2)   que é a média harmônica...

Abraços,
Nehab

At 22:39 8/8/2007, you wrote:

A densidade total é a média harmônica das densidades parciais.
- Original Message -
From: mailto:[EMAIL PROTECTED]arkon
To: mailto:obm-l@mat.puc-rio.brobm-l
Sent: Wednesday, August 08, 2007 10:49 AM
Subject: [obm-l] BETONEIRA

Alguém pode resolver esta, por favor:

Uma betoneira está sendo preparada para produzir concreto. 
Receberá iguais quantidades, em massa, de areia, cimento e brita 
de densidades iguais a 1,5; 1,3 e 2,4, respectivamente. Calcular a 
densidade do concreto que vai ser produzido pela betoneira. 
Multiplique o resultado por 10 e despreze a parte fracionária, caso exista.


DESDE JÁ MUITO OBRIGADO
--
No virus found in this incoming message.
Checked by AVG

Re: [obm-l] Qual a modo correto de lidarmos com conceitos que fazem se ntido mas s�o contradit�rios?

2007-08-08 Por tôpico Carlos Eddy Esaguy Nehab
(obs: tive problemas com meu provedor gringo e cadastrei outro 
email; reenvio esta mensagem com o novo email cadastrado na Lista)


Caro Artur,

Você aborda uma questão extremamente interessante e que raramente 
ocorre com a freqüência merecida em nossa Lista.


Alguns colegas já deram respostas interessantes, mas acho que posso 
contribuir também.


Há milhões de entradas na Internete livros interesssantes, mas acho 
que o pequeno texto em

http://plato.stanford.edu/entries/settheory-early/

ajuda a dar a partida sobre a turma que se envolveu com estas 
questões. É bem compacto e mostra o ninho de vespas (maravilhoso) em 
que você está se metendo...


Apenas lembrando que seu ponto de partida intuitivo de que o 
conjunto de todos os conjuntos faz sentido é o grande bandido da 
novela que se arrastou por uns 50 anos no minimo (leia o link 
sugerido).  Apenas lembrando outro paradoxo clássico (não 
semânticos): pense no conjunto de todos os conjuntos que não 
pertencem a si próprios.  Este cara pertence a si próprio?


Há poucos dias também outro colega mencionou completude e 
incompletude e todos estes temas (além de sua observação) são muitos 
caros aos lógicos, e em passado recente,  também se tornaram vitais 
para a turma de Ciência de Computação (nos quais eu me incluo) como 
os conceitos de linguagens formais, computabilidade, recursão, 
máquinas abstratas (a de Turing uma delas) e outros conceitos afins.


O que eu modestamente sugeriria é que você trilhasse um dos três 
seguintes caminhos...:


1) se preferir algo gostoso de ler, que aborda de forma amigável mas 
genial os sistemas formais, completude e incompletude, e muitos 
outros temas, leia o livro Goedel, Escher e Bach, uma obra prima 
escrita por Douglas Hofstadter que ganhou o prêmio Pulizer há sei lá 
quantos anos (talvez nos anos 80); e se você gostar de toda a trama 
linguistica e lógica do Alice no País das Maravilhas (que até pode 
ser relido), vai amar este livro.   A propósito (da Alice) saiu uma 
edição maravilhosa em português em 2002 pela Zahar editores (edição comentada).


2) pegue um bom livro de Lógica (por exemplo o Theory of Sets do 
Bourbaki) para se aprofundar (com mais de 50 páginas só de notas 
históricas);  no passado (anos 70) cometi a maluquice de adotá-lo no 
ciclo básico do IME quando dei aula por lá... Os que sobreviveram não 
se queixam mais...  Aprenderam quase tudo que tinham (e que não 
tinham) que aprender sobre lógica...


3) se preferir caminhar pela Computação, há dezenas de livros 
interessantes, mas o de Roberto Lins de Carvalho (Máquinas, Programas 
e Algoritmos) é excelente.


Um abraço,
Nehab


At 18:43 6/8/2007, you wrote:
Como, por exemplo, o conjunto de todos os conjuntos.  Não deixa de 
fazer sentido, mas leva a contradição. O conjunto de todos os 
conjuntos tem como elementos todas as suas partes, logo tem 
cardinalidade maior do que o conjunto de suas partes. Mas isto 
contraria o famos teorema de Cantor.


Artur





Re: [obm-l] Qual a modo correto de lidarmos com conceitos que fazem se ntido mas s�o contradit�rios?

2007-08-08 Por tôpico Carlos Eddy Esaguy Nehab
Cá para nós: minha frasezinha raramente ocorre com freqüência na 
lista é de completa estupidez, né... Leia-se:   pouco aparece na 
lista, não obstante o tema ser extremamente importante  Ufa !!!


Nehab

At 08:35 8/8/2007, you wrote:
(obs: tive problemas com meu provedor gringo e cadastrei outro 
email; reenvio esta mensagem com o novo email cadastrado na Lista)


Caro Artur,

Você aborda uma questão extremamente interessante e que raramente 
ocorre com a freqüência merecida em nossa Lista.


Alguns colegas já deram respostas interessantes, mas acho que posso 
contribuir também.


Há milhões de entradas na Internete livros interesssantes, mas acho 
que o pequeno texto em

http://plato.stanford.edu/entries/settheory-early/

ajuda a dar a partida sobre a turma que se envolveu com estas 
questões. É bem compacto e mostra o ninho de vespas (maravilhoso) em 
que você está se metendo...


Apenas lembrando que seu ponto de partida intuitivo de que o 
conjunto de todos os conjuntos faz sentido é o grande bandido da 
novela que se arrastou por uns 50 anos no minimo (leia o link 
sugerido).  Apenas lembrando outro paradoxo clássico (não 
semânticos): pense no conjunto de todos os conjuntos que não 
pertencem a si próprios.  Este cara pertence a si próprio?


Há poucos dias também outro colega mencionou completude e 
incompletude e todos estes temas (além de sua observação) são 
muitos caros aos lógicos, e em passado recente,  também se tornaram 
vitais para a turma de Ciência de Computação (nos quais eu me 
incluo) como os conceitos de linguagens formais, computabilidade, 
recursão, máquinas abstratas (a de Turing uma delas) e outros conceitos afins.


O que eu modestamente sugeriria é que você trilhasse um dos três 
seguintes caminhos...:


1) se preferir algo gostoso de ler, que aborda de forma amigável mas 
genial os sistemas formais, completude e incompletude, e muitos 
outros temas, leia o livro Goedel, Escher e Bach, uma obra prima 
escrita por Douglas Hofstadter que ganhou o prêmio Pulizer há sei lá 
quantos anos (talvez nos anos 80); e se você gostar de toda a trama 
linguistica e lógica do Alice no País das Maravilhas (que até pode 
ser relido), vai amar este livro.   A propósito (da Alice) saiu uma 
edição maravilhosa em português em 2002 pela Zahar editores (edição comentada).


2) pegue um bom livro de Lógica (por exemplo o Theory of Sets do 
Bourbaki) para se aprofundar (com mais de 50 páginas só de notas 
históricas);  no passado (anos 70) cometi a maluquice de adotá-lo no 
ciclo básico do IME quando dei aula por lá... Os que sobreviveram 
não se queixam mais...  Aprenderam quase tudo que tinham (e que não 
tinham) que aprender sobre lógica...


3) se preferir caminhar pela Computação, há dezenas de livros 
interessantes, mas o de Roberto Lins de Carvalho (Máquinas, 
Programas e Algoritmos) é excelente.


Um abraço,
Nehab


At 18:43 6/8/2007, you wrote:
Como, por exemplo, o conjunto de todos os conjuntos.  Não deixa de 
fazer sentido, mas leva a contradição. O conjunto de todos os 
conjuntos tem como elementos todas as suas partes, logo tem 
cardinalidade maior do que o conjunto de suas partes. Mas isto 
contraria o famos teorema de Cantor.


Artur




Re: [obm-l] BETONEIRA

2007-08-08 Por tôpico Carlos Eddy Esaguy Nehab

Oi, Arkon, Ponce e Desejo...

Já que o Desejo (Ojesed Mirror) deu esta ótima resposta, fica aqui 
uma dica, pois problemas desta natureza já apareceram diversas vezes 
por aqui...


Quando se introduz o conceito de médias (mesmo na 6 ou 7 séries) é 
extremamente oportuno sugerir contextos onde elas ocorrem (para não 
parecer um negócio artificial) e a resposta do Ojesed mostra que ele 
já adquiriu a malicia que eu acho legal.


Na Física a média harmônica ocorre com freqüência, pois ela é usual 
em todas as situações onde a grandeza da qual se deseja calcular a 
média é o quociente entre duas variáveis  e 
vejamos:  Velocidade é distância / tempo...  Densidade é massa / 
volume, resistência = voltagem/ corrente ...   Logo, se desejamos 
calcular velocidade média, densidade média, resistência equivalente, 
fatalmente a média harmônica entra na jogada (caso os valores das 
distâncias, volumes ou voltagens sejam iguais e isto ocorre na 
ligação em paralelo - como as voltagens não se somam, a resistência 
equivalente é o dobro da média harmônica...), posto que velocidade 
média = dist total / tempo total; densidade  final = massa total / 
volume total... e resistencia = mesma voltagem / corrente total


Vejamos um exemplinho  clássico (o outro é o do Arkon, posto que se 
misturam iguais quantidades de MASSA...)


Você vai a 60 km por hora num trecho de estrada e no mesmo trecho 
volta a 90 km/h.  Qual sua velocidade média?


Ora, você está querendo medir velocidade média, mas a variável 
chave, que é o tempo, está no denominador das velocidades e as duas 
distâncias, de ida e de volta são iguais ...


Logo a velocidade média (vm) é a média harmônica.

Veja: vm =  distância total / tempo total = (d1 + d2) / (t1 + t2)   (A)

Ocorre que t1 =  d1 /v1  e  t2 = d2/v2

Levando estas expressoes em (A)  voce obtem
dist total / tempo total = (x + x) /  [x/v1 + 
x/v2]  =  2v1.v2  (v1+v2)   que é a média harmônica...


Abraços,
Nehab


At 22:39 8/8/2007, you wrote:

A densidade total é a média harmônica das densidades parciais.
- Original Message -
From: mailto:[EMAIL PROTECTED]arkon
To: mailto:obm-l@mat.puc-rio.brobm-l
Sent: Wednesday, August 08, 2007 10:49 AM
Subject: [obm-l] BETONEIRA

Alguém pode resolver esta, por favor:



Uma betoneira está sendo preparada para produzir concreto. Receberá 
iguais quantidades, em massa, de areia, cimento e brita de 
densidades iguais a 1,5; 1,3 e 2,4, respectivamente. Calcular a 
densidade do concreto que vai ser produzido pela betoneira. 
Multiplique o resultado por 10 e despreze a parte fracionária, caso exista.




DESDE JÁ MUITO OBRIGADO


--
No virus found in this incoming message.
Checked by AVG Free Edition.
Version: 7.5.476 / Virus Database: 269.11.8/941 - Release Date: 7/8/2007 16:06


Re: [obm-l] (OFF) Nome de Alguns dos Grandes Professores do Pa

2007-08-06 Por tôpico Carlos Eddy Esaguy Nehab

I João!  Caramba  !!!

Passo a bola pro Nicolau, pois certamente ele poderá fornecer dicas 
de Instituições relacionadas à OBM que desenvolvem este tipo de 
trabalho sem interesse comercial...  Sei que há uma série de 
atividades ligadas à olimpiadas, mas  esperemos o Nicolau responder...   l


Grande abraço,
Nehab

At 16:21 3/8/2007, you wrote:


Nehab, Ponce:

Quais são todos professores residentes nos grandes centros que preparam
jovens brasileiros para as Olimpíadas de Matemática, e que sejam também
partícipes desta bela lista?
Desejo-lhes fazer ousada e nobre solicitação.

Fraternalmente, João


=
Instruções para entrar na lista, sair da lista e usar a lista em
http://www.mat.puc-rio.br/~nicolau/olimp/obm-l.html
=


Re: [obm-l] Um numero N com n algarismos....

2007-07-31 Por tôpico Carlos Eddy Esaguy Nehab

Oi, Vitorio,

Semelhante a esta (acho que foi a original...) caiu na Olimpíada de 
Maio de 2001:

A solução é armar a conta e fazê-la, mesmo

Sara escreveu no quadro negro um número inteiro de menos de 30 
algarismos e que termina em 2.  Célia apaga o 2 do fim e escreve-o no 
início. O número que fica é igual ao dobro do número que tinha 
escrito Sara. Qual o número que Sara escreveu?


Solução
Do enunciado, temos:
?  g   f   e  d   c   b  a  2
   x  2
2 ...  h   g   f  e  d   c   b  a

a = 4; colocando o 4 no lugar do a na parcela de cima  e continuando 
a multiplicação, obtemos, b = 8 (2 x 4);  assim, continuando o mesmo 
mecanismo, temos, sucessivamente, c = 6 (2 x 8 = 16, e vai um); d = 
3; e = 7; f = 4...
Continuando o processo até que ocorra o algarismo 2 pela primeira 
vez, obtemos o número desejado:

210.526.315.789.473.684

Observe que se não limitarmos o número de algarismos, haverá outras 
soluções (é só continuar a brincadeira).


Abraços,
Nehab

PS: Onde você viu esta questão?


At 15:01 31/7/2007, you wrote:

Ola' pessoal,

Uma ajuda Considere um número N com n algarismos e na posição 
das unidades o número 2. Ao invertemos o 2, colocando-o na posição 
inicial, encontramos um novo número K, onde K=2N. Qual o valor de N?


Pensei em congruencia...seria uma boa


=
Instruções para entrar na lista, sair da lista e usar a lista em
http://www.mat.puc-rio.br/~nicolau/olimp/obm-l.html
=


Re: Res: [obm-l] Teoria Numeros

2007-07-30 Por tôpico Carlos Eddy Esaguy Nehab

Desculpe-me: engoli uma palavra no texto: Se você quiser ver...

Oi, Klaus,

Se você ver a utilidade do referido produto notável

(a^4 + 4b^4) = (a^4 + 4a^2b^2 + 4b^4) - 4a^2b^2 = (a2 + 2b^2)^2 - (2ab)^2
= (a^2 + 2b^2 + 2ab) * (a^2 + 2b^2 - 2ab)

dê uma paquerada neste interessante exercício de uma Lista do prof. 
Felipe Rodrigues :


Simplifique X = P/Q, onde

P = (10^4 + 324)(22^4 + 324)(34^4 + 324)(46^4 + 324)(58^4 + 324)  e
Q = (4^4 + 324)(16^4 + 324)(28^4 + 324)(40^4 + 324)(52^4 + 324).

Acho que este exercício caiu em alguma olimpíada brasileira, mas não 
consigo localizar em qual.


Abraços,
Nehab



At 10:09 30/7/2007, you wrote:

Valeu Leandro. Eu nunca tinha ouvido falar nessa fatoração de Sophie Germain.

- Mensagem original 
De: [EMAIL PROTECTED] [EMAIL PROTECTED]
Para: obm-l@mat.puc-rio.br
Enviadas: Domingo, 29 de Julho de 2007 17:37:11
Assunto: Re: [obm-l] Teoria Numeros

Olá Klaus,

Esse problema se resolve com uso da clássica fatoração de Sophie Germain:

a^4 + 4*b^4 = (a^2 + 2b^2 + 2ab) * (a^2 + 2b^2 - 2ab).

Trabalhando com a sua expressão,

545^4 + 4^545  =  545^4 + 4*(4^136)^4, que é da forma acima, ou 
seja, a^4 + 4*b^4,


para a = 545 e b = 4^136.

Resta cuidar para que nenhum dos parênteses acima seja 1;

mas isso é praticamente trivial, dado que a^2 + b^2  2ab, pois a e 
b são diferentes de zero,


e assim sobra um b^2 dentro de cada um.

Abraço,

- Leandro A. L.


Alertas do Yahoo! Mail em seu celular. 
http://br.mobile.yahoo.com/mailalertas/Saiba mais.


Re: Res: [obm-l] Teoria Numeros

2007-07-30 Por tôpico Carlos Eddy Esaguy Nehab

Oi, Klaus,

Se você ver a utilidade do referido produto notável

(a^4 + 4b^4) = (a^4 + 4a^2b^2 + 4b^4) - 4a^2b^2 = (a2 + 2b^2)^2 - (2ab)^2
= (a^2 + 2b^2 + 2ab) * (a^2 + 2b^2 - 2ab)

dê uma paquerada neste interessante exercício de uma Lista do prof. 
Felipe Rodrigues :


Simplifique X = P/Q, onde

P = (10^4 + 324)(22^4 + 324)(34^4 + 324)(46^4 + 324)(58^4 + 324)  e
Q = (4^4 + 324)(16^4 + 324)(28^4 + 324)(40^4 + 324)(52^4 + 324).

Acho que este exercício caiu em alguma olimpíada brasileira, mas não 
consigo localizar em qual.


Abraços,
Nehab



At 10:09 30/7/2007, you wrote:

Valeu Leandro. Eu nunca tinha ouvido falar nessa fatoração de Sophie Germain.

- Mensagem original 
De: [EMAIL PROTECTED] [EMAIL PROTECTED]
Para: obm-l@mat.puc-rio.br
Enviadas: Domingo, 29 de Julho de 2007 17:37:11
Assunto: Re: [obm-l] Teoria Numeros

Olá Klaus,

Esse problema se resolve com uso da clássica fatoração de Sophie Germain:

a^4 + 4*b^4 = (a^2 + 2b^2 + 2ab) * (a^2 + 2b^2 - 2ab).

Trabalhando com a sua expressão,

545^4 + 4^545  =  545^4 + 4*(4^136)^4, que é da forma acima, ou 
seja, a^4 + 4*b^4,


para a = 545 e b = 4^136.

Resta cuidar para que nenhum dos parênteses acima seja 1;

mas isso é praticamente trivial, dado que a^2 + b^2  2ab, pois a e 
b são diferentes de zero,


e assim sobra um b^2 dentro de cada um.

Abraço,

- Leandro A. L.


Alertas do Yahoo! Mail em seu celular. 
http://br.mobile.yahoo.com/mailalertas/Saiba mais.


Re: [obm-l] Uma boa de geometria - CALMA !!!

2007-07-30 Por tôpico Carlos Eddy Esaguy Nehab

Oi,  Douglas,

Muito legais suas idéias e sua solução.  Eu passei perto de sua 
expressão mas aqui vai uma modesta colaboração para você fechar SUA 
bonita solução do jeito que você queria... (é só um treinozinho nas 
nojentas expressões trigonométricas vestibulinas...):


Façamos
X  = (cosA)^2 + (cosB)^2 + (cosC)^2
Dai, como cos2x = 2(cosx)^2 -1, vem
X = (1 + cos2A)/2 + (1+cos2B)/2 + (cosC)^2
X =  1 + [cos(2A) +cos(2B) ]/2   + (cosC)^2

Mas
cos(2A) + cos(2B) = 2cos(A+B)cos(A-B) = -2cosC cos(A-B).

Substituindo em X:
X = 1 - cosC [ cos(A-B) - cosC]  = 1 - cosC [ cos(A-B) + cos(A+B) 
].   Dai acabou:

X= 1 - cosC. [2cosA.cosB]  = 1 - 2cosA.cosB.cosC

Substituindo este X na expressão que você obteve, você chega na 
desejada expressão do enunciado que o motivou.
7 - 4 [ (cosA)^2 + (cosB)^2 + (cosC)^2 ] =  7 - 4 [ 1 - 
2cosA.cosB.cosC ] = 3 - 8 cosA.cosB.cosC


Um grande abraço,
Nehab

PS: Nem ouse me incluir na sua linda construção.  O mérito é todo seu !


At 22:22 30/7/2007, you wrote:

Olá Nehab!

Primeiramente gostaria de expressar minha satisfação do problema ter
de fato chamado sua atenção e do Rogério Ponce. Já participo da
lista(não muito ativamente) há um bom tempo e percebo que assim como
eu, vocês gostam muito de geometria.

O problema na verdade veio da minha cabeça, mas foi inspirado em um
problema proposto na última(ou penúltima) Eureka. Originalmente o
problema pedia para mostrar que XYZ estão alinhados se e somente se
cosA*cosB*cosC = -3/8. Então pensei em me inspirar nos chineses, que
gostavam de resolver teoremas usando áreas, e pensei em zerar a área
do triangulo XYZ para chegar na tão esperada relação.

Por um lado estou satisfeito, pois consegui chegar em uma expressão
que relaciona as áreas corretamente, por outro estou frustrado pois
não consigo fazer a última passagem, que certamente exige uma
fatoração ou algo do tipo que não estou conseguindo enxergar.

A relação que eu cheguei foi S(XYZ) = S(ABC)*[7 - 4((cosA)^2 +
(cosB)^2 + (cosC)^2)].
Note que a relação é válida nos casos mais triviais em que o triangulo
é equilatero, retangulo(que, como o Rogerio falou, são respectivamente
1:4 e 1:3) ou isosceles com angulo de 120 graus(basta fazer um desenho
para ver que a área dá zero). Notem que a relação pedida no problema
da Eureka é satisfeita para este triangulo isosceles.

Aos curiosos que querem saber como eu cheguei nessa relação, segue a
idéia abaixo:

Construam o triangulo ABC e suas respectivas reflexões XYZ.
Observe que S(XYZ) = [S(ABC) + S(BCX) + S(ACY) + S(ABZ)] - S(AYZ) -
S(XBZ) - S(XYC)

S(ABC) = S(BCX) = S(ACY) = S(ABZ) por construção

As áreas de AYZ XBZ e XYC podem ser somadas ou subtraídas, dependendo
se os ângulos YAB = 3A, XBZ = 3B ou XCY = 3C forem maiores ou menores
que 180 graus. Para esses triangulos vou usar que S(AYZ) =
bc*sen(3A)/2, S(XBZ) = ac*sen(3B)/3 e S(XYC) = ab*sen(3C)/2.

Então a relação passa a ser S(XYZ) = 4S(ABC) - bc*sen(3A)/2 -
ac*sen(3B)/3 - ab*sen(3C)/2

Agora substituímos sen(3X) = -4*[sen(X)]^3 + 3*sen(X) em todos e
substituímos também bc/2 ac/2 e ab/2 respectivamente por
S(ABC)/sen(A), S(ABC)/sen(B) e S(ABC)/sen(C), devido à mesma fórmula
de área em função dos lados e do angulo para o triangulo original.

Fazendo as devidas substituições acima, simplificamos os senos e
ficamos com a relação da soma dos quadrados dos senos. Basta trocar
[sen(X)]^2 por 1 - [cos(X)]^2 e chegamos em S(XYZ) = S(ABC)*[7 -
4((cosA)^2 + (cosB)^2 + (cosC)^2)].

Quando eu enviei o problema ainda não tinha chegado nesse resultado e
achava que chegaria em uma expressão mais fácil de passar para o
produto de cossenos.

Qualquer ajuda para terminar o problema eu agradeço bastante e
certamente darei os devidos créditos quando enviar a solução para a
Eureka.

Abraços, Douglas




Em 30/07/07, Carlos Eddy Esaguy Nehab[EMAIL PROTECTED] escreveu:

  Oi, querido Ponce

  Naturalmente não se supunha (pelo menos eu) que a relação entre as áreas
 independesse do triângulo, mas mesmo assim, confesso que tentei vários
 caminhos e não encontrei uma solução simples para o problema.

  Eu esperava algo do tipo:  a razão entre as áreas é o quadrado do produto
 dos senos dos angulos, ou  coisa similar.  Embora tendo encontrado várias
 coisas curiosas sobre o maldito e interessante triângulo, tentando resolver
 o problema, não encontrei nada simples que merecesse ser publicado.

  E também confesso que imaginei que alguém mais inspirado 
conseguisse alguma

 expressão simples para a resposta.Resta aguardar  que quem propôs o
 problema informe se sabe alguma coisa (aliás hábito pouco 
praticado em nossa

 lista é informar a origem dos problemas propostos - e às vezes, a origem é
 bastante interessante).

  Eu realmente gosto desta informação pois tenho o hábito (e gosto) de
 mencionar a origem (e a solução) de qualquer problema que eu proponho, no
 mínimo para respeitar a história... e o trabalho alheio.

  Abraços,
  Nehab

  At 01:09 29/7/2007, you wrote:

 Ola' Douglas e colegas da lista

Re: [obm-l] Favor Responder...

2007-07-26 Por tôpico Carlos Eddy Esaguy Nehab

Ok, falei, mas não sei se você ouviu...

Nehab

At 19:14 26/7/2007, you wrote:

Obrigado pela atenção...
Enviei dois e-mails nessa conta na lista de discussão da obm...
Não recebi nenhuma resposta nem vejo meu e-mail na lista...
Portanto não sei se estes estão chegando...
Caso alguém receba este e-mail favor me responder falando...
Arigatô


Re: [obm-l] Uma boa de geometria

2007-07-25 Por tôpico Carlos Eddy Esaguy Nehab

Oi, João Carlos,

Tive uma idéia que não sei se frutifica, pois não estou muito 
inspirado, mas veja se ajuda.  (é uma solução não geométrica, no 
sentido usual):


Dada uma reta fixa, pense na transformação (não linear, é claro) que 
associa a cada ponto do plano o seu simétrico com relação a tal reta.


1) Observe agora que dado um triângulo, a transformação T1, digamos, 
que toma o simétrico de A, B e C com relação à reta suporte de AB 
deixa A e B no mesmo lugar e coloca o ponto C onde você quer (ou 
seja, no simétrico C' etc).


Como conseqüência, se você pensar nas 3 transformações T1, T2 e T3 
(respectivamente que tomam o simétrico com relação às retas suporte 
de AB, BC e CA), verá que o triângulo que você construiu tomando os 
três simétricos é a imagem do triângulo original pela composta das 3 
transformações T1, T2 e T3 (concorda?).   Logo, a questão é saber se 
a tal composta é fácil de analisar.


2) Vejamos: uma simetria com relação a uma reta pode ser decomposta 
como soma de uma simetria (transformação linear com determinante 
-1)  e uma translação.


Pense na T1, por exemplo, assim: se a reta suporte de AB é a reta y = 
px + q dá para calcular a imagem de um ponto (x; y) por T1:
T1( x, y) = T'(x, y)  + 1/(p^2+1) . (2q; -2pq),  onde a matriz da 
transformação T' (simetria) possui linha 1 igual a  [2p   p^2 -1]  e 
segunda linha [ 1 - p^2   2p], ambas divididas por (p^2 +1).  Dica: 
Imponha que o ponto médio de CC' está nesta reta e que o coeficiente 
angular da reta  CC' vale -1/p (são ortogonais).


3) Agora: será que é fácil analisar a composição de T1, T2  e T3  sem 
muito trabalho braçal?


Não me inpirei, mas fica aí a idéia.   Naturalmente que vou tentar 
uma solução geométrica mais tarde  :-).


Abraços,
Nehab

At 08:33 25/7/2007, you wrote:


Até o momento, não sei como resolver essa questão não.


Seja um triangulo ABC com lados a, b, c.

X eh a reflexao de A em relacao a reta que passa por BC
Y eh a reflexao de B em relacao a reta que passa por AC
Z eh a reflexao de C em relacao a reta que passa por AB

Qual a relacao entre as areas de ABC e XYZ?
=
Instruções para entrar na lista, sair da lista e usar a lista em
http://www.mat.puc-rio.br/~nicolau/olimp/obm-l.htmlhttp://www.mat.puc-rio.br/~nicolau/olimp/obm-l.html
=
Instruções para entrar na lista, sair da lista e usar a lista em 
http://www.mat.puc-rio.br/~nicolau/olimp/obm-l.html


Re: [obm-l] Conjectura - Teoria dos N�meros

2007-07-17 Por tôpico Carlos Eddy Esaguy Nehab

Oi, Yuri,

Cuidado, Yuri, só vale a ida...  Se n é primo então  a^n = a (mod n)...

Por exemplo,   3^91 = 3 (mod 91) mas  91 é composto.
Veja que 3^6 = 1 (mod 91), logo, 3^90 =1 (mod 91)...

Abraços,
Nehab


At 15:44 16/7/2007, you wrote:

Isso é um teorema do euler: a^n = a (mod n) se e somente se n eh primo.

Iuri



On 7/16/07, Angelo Schranko mailto:[EMAIL PROTECTED] 
[EMAIL PROTECTED] wrote:

Saudações Srs.

Sou novo na lista.
Por favor me ajudam a provar (ou encontrar um contra-exemplo)
para a seguinte conjectura :

(2^(n - 1) - 1)/n é inteiro = n primo

Obrigado,
[]´s
Angelo


Novo http://yahoo.com.br/oqueeuganhocomisso+Yahoo! Cadê? - 
Experimente uma nova busca.




Re: [obm-l] O sapo e agora, ent�o, o jornaleiro... ( off topic)

2007-07-16 Por tôpico Carlos Eddy Esaguy Nehab

Oi, Santa Rita,

E eu peço desculpas por colocá-lo ao lado dos coroas..., mas o erro 
foi a informação de que você é pai de ex-olímpico - alguém comentou 
isto.  Daí, algumas contas de somar malfeitas :-)  ... e o absurdo da 
comparação


Mas cá pra nós, você conhece ou não a Gladys e seus bichinhos...

Abraços,
Nehab

At 10:27 16/7/2007, you wrote:

Ola Alonso e demais
colegas desta lista ... OBM-L,

Agradeco a correcao que voce fez a mensagem do Nehab. Eu tambem acho
que idade, sexo, origem, formacao academica, titulos etc sao aspectos
totalmente irrelevantes no que concerne a real capacidade intelectual
das pessoas, nao obstante ja ter observado que muitas pessoas -
sobretudo os mais mediocres -  viverem bastante atentos a estas coisas
acidentais e secundarias  ...

Eu sei que o EU É ODIAVEL, como dizia o Gide ... Mas eu imagino que
quando eu for um quarentao, cinquentao, sessentao e mesmo um setentao
estarei com uma capacidade intelectual maior que a atual, pois procuro
me fixar nas minhas deficiencias e nao nas minhas qualidades, o que me
garante um crescimento constante.

Para que a mensagem nao fique totalmente off-topic aqui vai um
problema matematico simples , que eu acho bonitinho :

PROBLEMA : Prove que em qualquer sequencia de 39 inteiros positivos
existe ao menos um numero cuja soma dos algarismos e divisivel por 11.

Mais problemas deste nivel veja aqui :
http://www.mat.puc-rio.br/~nicolau/psr

Um Abraco a Todos
Paulo Santa Rita
2,0A1A,160707


Em 16/07/07, ralonso[EMAIL PROTECTED] escreveu:

 Olá Nehab, Ponce e Demais colegas:

Eu conheço o Paulo Santa Rita pessoalmente e ele não é tão velho assim
:)
Aliás, ele aparenta ter bem menos que 40 anos (eu tenho 32) ele me disse a
idade
dele, e é bem menos do que a mensagem abaixo sugere :).   Brincadeiras a
parte
acho que o importante não é a idade e sim a potência cerebral. Basta lembrar
que
Andrew Willes provou o teorema de Fermat quando já tinha 40 anos.  Essa
história de que
a matemática é coisa exclusiva de gente jovem não é muito verdade.   O que
ocorre é que com
a idade as pessoas começam a ganhar mais atribuições e responsabilidades  se
preocupar mais com
outros problemas de ordem prática e menos com problemas de ordem teórica
(como
a matemática). O brilhante Einstein já havia percebido isso logo com 21 anos
quando virou
funcionário público.  A frase que eu nunca me esqueço de seu livro Como
Vejo o Mundo
é  o homem possui um estômago e precisa de alimento para sustentar seu
cérebro.
   Sem isto não há intelecto que resista.  Nos casos mais extremos, o
capitalismo chega às vezes
a ser mais cruel, sacrificando, às vezes, o próprio intelecto 

   Abraços!
Ronaldo.



Carlos Eddy Esaguy Nehab wrote:
Ponce,

Este seu amigo sapo é muito metido e saudosista.   Quanto à  Tia Glads e...
seus bichinhos... francamente, só uns 3 caras aqui da lista sabem do que
você está falando.   Eu devo ser o segundo e acho que o Santa Rita é o
terceiro.   Bem, tu tá envelhecendo mesmo, hein.   Podia ter perguntado pela
Xuxa, já do passado, mas mais moderninha  Quanto ao Godofredo, não tenho
a menor idéia...!Espero que não seja um sapo...

Se você quer lembrar do tempo em que você ainda era bonitinho tá tudo em
http://www.infantv.com.br/alfab_prog.htm.

Carinhoso abraço,
Nehab

At 20:54 13/7/2007, you wrote:
Oi Nehab,
nao consegui vislumbrar uma forma elegante de resolver o problema, embora o
sapo tenha me assoprado que o jornaleiro deve comprar 90 jornais para
obter um lucro medio de R$361,80 .
[]'s
Rogerio Ponce

PS: Voce ainda se lembra da Tia Glads na televisao?  E do Godofredo ?
:-)



Carlos Eddy Esaguy Nehab [EMAIL PROTECTED] escreveu: Bem,
  Depois do sapo e das soluções interessantíssimas do Nicolau e do Ponce,
achei que procede colocar na lista um problema clássico (e extremamente
interessante para a área de logística - atualmente tenho dado alguns cursos
de Metodos Quantitativos aplicados à Logística - daí a motivação.
  Bem, o problema e o seguinte:
  Um jornaleiro compra de uma empresa uma certa quantidade de jornais por
dia (sua capacidade financeira limita esta quantidade a no máximo 110
jornais), para revendê-los. Ele paga R$ 3 e os vende a R$ 8. Os jornais que
ele comprou no dia e não consegue vendê-los são comprados pela empresa (de
volta) por R$ 1.
  O grande problema é que ele nao sabe quantos jornais deve comprar para
maximizar seu lucro, uma vez que a demanda diária é desconhecida. No entanto
a experiência mostra que a demanda pelos jornais, independente do dia,
supera 50 jornais e é distribuida da seguinte maneira:
  Probabilidade da demanda de jornais ser X jornais (em qualquer dia) vale
p%, onde: X p% 50 10% 60 12% 70 15% 80 20% 90 18% 100 15% 110 10%
  A pergunta é a esperada: quantos jornais o jornaleiro deve comprar para
maximizar seu lucro esperado?
  Abraços, Nehab


Novo Yahoo! Cadê? - Experimente uma nova busca.















=
Instruções

Re: [obm-l] O sapo e agora, ent�o, o jornaleiro... (off topic)

2007-07-14 Por tôpico Carlos Eddy Esaguy Nehab

Ponce,

Este seu amigo sapo é muito metido e saudosista.   Quanto à  Tia 
Glads e... seus bichinhos... francamente, só uns 3 caras aqui da 
lista sabem do que você está falando.   Eu devo ser o segundo e acho 
que o Santa Rita é o terceiro.   Bem, tu tá envelhecendo mesmo, 
hein.   Podia ter perguntado pela Xuxa, já do passado, mas mais 
moderninha  Quanto ao Godofredo, não tenho a menor 
idéia...!Espero que não seja um sapo...


Se você quer lembrar do tempo em que você ainda era bonitinho tá 
tudo em http://www.infantv.com.br/alfab_prog.htm.


Carinhoso abraço,
Nehab

At 20:54 13/7/2007, you wrote:

Oi Nehab,
nao consegui vislumbrar uma forma elegante de resolver o problema, 
embora o sapo tenha me assoprado que o jornaleiro deve comprar 90 
jornais para obter um lucro medio de R$361,80 .

[]'s
Rogerio Ponce

PS: Voce ainda se lembra da Tia Glads na televisao?  E do Godofredo ?
:-)



Carlos Eddy Esaguy Nehab [EMAIL PROTECTED] escreveu:
Bem,

Depois do sapo e das soluções interessantíssimas do Nicolau e do
Ponce, achei que procede colocar na lista um problema clássico (e
extremamente interessante para a área de logística - atualmente
tenho dado alguns cursos de Metodos Quantitativos aplicados à
Logística - daí a motivação.

Bem, o problema e o seguinte:

Um jornaleiro compra de uma empresa uma certa quantidade de jornais
por dia (sua capacidade financeira limita esta quantidade a no máximo
110 jornais), para revendê-los. Ele paga R$ 3 e os vende a R$
8. Os jornais que ele comprou no dia e não consegue vendê-los são
comprados pela empresa (de volta) por R$ 1.

O grande problema é que ele nao sabe quantos jornais deve comprar
para maximizar seu lucro, uma vez que a demanda diária é
desconhecida. No entanto a experiência mostra que a demanda pelos
jornais, independente do dia, supera 50 jornais e é distribuida da
seguinte maneira:

Probabilidade da demanda de jornais ser X jornais (em qualquer dia)
vale p%, onde:
X p%
50 10%
60 12%
70 15%
80 20%
90 18%
100 15%
110 10%

A pergunta é a esperada: quantos jornais o jornaleiro deve comprar
para maximizar seu lucro esperado?

Abraços,
Nehab



Novo http://yahoo.com.br/oqueeuganhocomissoYahoo! Cadê? - 
Experimente uma nova busca.


Re: [obm-l] PRODUTO DA UFPB

2007-07-14 Por tôpico Carlos Eddy Esaguy Nehab

Oi, Arkon

Multiplique sua expressão por 2.sen pi/65, notando que sucessivamente 
você poderá aplicar sen 2a = 2.cos a.sen a, ajustando o coeficiente 2...


Abraços,
Nehab

At 11:22 14/7/2007, you wrote:

OLÁ PESSOAL, ALGUÉM PODERIA ME PASSAR O MACETE PARA RESOLVER ESSA:


?xml:namespace prefix = v ns = urn:schemas-microsoft-com:vml 
/?xml:namespace prefix = o ns = 
urn:schemas-microsoft-com:office:office /(UFPB-90) O produto 
cospi/65.cos2pi/65.cos4pi/65.cos8pi/65.cos16pi/65.cos32pi/65 é igual a:




a) 1/64. b) 1/32. c) 
1/2.   d) 0. e) 1.


O GABARITO É LETRA A



DESDE JÁ MUITO OBRIGADO


Re: [obm-l] quest�o 4 de matem�tica - torneira corrupta !

2007-07-13 Por tôpico Carlos Eddy Esaguy Nehab

Oi Renan !

Ora esta !   É verdade !!!   Só com seu comentáro eu prestei atenção !!!

Parece uma torneira corrupta com vazão fácil de calcular !  Em uma 
hora o estrago que ela causa é
(A+B+C+D) - (A+B+C)  = 1/12 - 1/10 =  -1/60 ávos do orçamento - 
oppp, desculpe, do tanque, que corresponde a apenas 20% de 
corrupção com relação à vazão total incluindo a própria corrupta)...


Mais barato que outras torneiras que andam pelo noticiário 
ultimamente... (sorry pelo off topic - mas foi irresistível e 
incontrolável - é raro, mas às vezes acontece... :-)


Abraços humoristicos, que ninguém é de ferro...
Nehab

At 01:43 13/7/2007, you wrote:
Essa torneira D parece mais um ralo, apropósito. A vazão dela é 
negativa, não é?


Em 13/07/07, Carlos Eddy Esaguy Nehab mailto:[EMAIL PROTECTED] 
[EMAIL PROTECTED] escreveu:

Ai que saudades destes problemas !

Ai vai:

Em uma hora, temos:
A, B e C enchem 1/10 do tanque
B, C e D enchem 1/15 do tanque
A, B e D enchem 1/20 do tanque
A, C e D enchem 1/30 do tanque.

Logo em 1 hora 3 torneiras A, 3 B, 3 C 3 3 D  enchem 1/10 + 1/15 + 
1/20 + 1/30 =  1/4 do tanque, ou seja as torneiras A, B, C e D 
(apenas uma de cada tipo) enchem 1/12 do tanque.


Ou seja, sua ilógica resposta está lógica e certa :-)

Abraços,
Nehab

PS: só no gostei do segundo artigo indefinido um 
tanque  Ficaria melhor se fosse o tanque...


At 14:55 12/7/2007, you wrote:

nao sei se estou mandando a questão para o e-mail certo, mas lá vai:


Quatro torneiras A, B, C e D enchem um tanque. Sabendo que: as 
torneiras A, B e C enchem um tanque em 10 horas; B, C, e D em 15 
horas; A, B e D em 20 horas e A, C e D em 30 horas. Estando o 
tanque vazio, em quantas horas as quatro torneiras encheriam o 
tanque se funcionassem conjuntamente?




(suponho q as os dados da questão estejam errados, pois minhas 
resposta deu ilógicas 12 horas)


_
Seja um dos primeiros a testar o novo Windows Live Mail Beta- 
grátis. Acesse 
http://www.ideas.live.com/programpage.aspx?versionId=5d21c51a-b161-4314-9b0e-4911fb2b2e6dhttp://www.ideas.live.com/programpage.aspx?versionId=5d21c51a-b161-4314-9b0e-4911fb2b2e6d 



=
Instruções para entrar na lista, sair da lista e usar a lista em
http://www.mat.puc-rio.br/%7Enicolau/olimp/obm-l.htmlhttp://www.mat.puc-rio.br/~nicolau/olimp/obm-l.html
=





--
Abraços,
J.Renan


[obm-l] O sapo e agora, ent�o, o jornaleiro...

2007-07-13 Por tôpico Carlos Eddy Esaguy Nehab

Bem,

Depois do sapo e das soluções interessantíssimas do Nicolau e do 
Ponce, achei que procede colocar na lista um problema clássico (e 
extremamente interessante para a área de logística -  atualmente 
tenho dado alguns cursos de Metodos Quantitativos aplicados à 
Logística - daí a motivação.


Bem, o problema e o seguinte:

Um jornaleiro compra de uma empresa uma certa quantidade de jornais 
por dia (sua capacidade financeira limita esta quantidade a no máximo 
110 jornais), para revendê-los.   Ele paga  R$ 3  e os vende a R$ 
8.  Os jornais que ele comprou no dia e não consegue vendê-los são 
comprados pela empresa (de volta) por R$ 1.


O grande problema é que ele nao sabe quantos jornais deve comprar 
para maximizar seu lucro, uma vez que a demanda diária é 
desconhecida.  No entanto a experiência mostra que a demanda pelos 
jornais, independente do dia, supera 50 jornais e é distribuida da 
seguinte maneira:


Probabilidade da demanda de jornais ser  X  jornais (em qualquer dia) 
vale  p%, onde:

X   p%
50  10%
60  12%
70  15%
80  20%
90  18%
100 15%
110 10%

A pergunta é a esperada: quantos jornais o jornaleiro deve comprar 
para maximizar seu lucro esperado?


Abraços,
Nehab

=
Instruções para entrar na lista, sair da lista e usar a lista em
http://www.mat.puc-rio.br/~nicolau/olimp/obm-l.html
=


Re: [obm-l] quest�o 4 de matem�tica (il�gica resposta?)

2007-07-12 Por tôpico Carlos Eddy Esaguy Nehab

Ai que saudades destes problemas !

Ai vai:

Em uma hora, temos:
A, B e C enchem 1/10 do tanque
B, C e D enchem 1/15 do tanque
A, B e D enchem 1/20 do tanque
A, C e D enchem 1/30 do tanque.

Logo em 1 hora 3 torneiras A, 3 B, 3 C 3 3 D  enchem 1/10 + 1/15 + 
1/20 + 1/30 =  1/4 do tanque, ou seja as torneiras A, B, C e D 
(apenas uma de cada tipo) enchem 1/12 do tanque.


Ou seja, sua ilógica resposta está lógica e certa :-)

Abraços,
Nehab

PS: só no gostei do segundo artigo indefinido um 
tanque  Ficaria melhor se fosse o tanque...


At 14:55 12/7/2007, you wrote:

nao sei se estou mandando a questão para o e-mail certo, mas lá vai:


Quatro torneiras A, B, C e D enchem um tanque. Sabendo que: as 
torneiras A, B e C enchem um tanque em 10 horas; B, C, e D em 15 
horas; A, B e D em 20 horas e A, C e D em 30 horas. Estando o tanque 
vazio, em quantas horas as quatro torneiras encheriam o tanque se 
funcionassem conjuntamente?




(suponho q as os dados da questão estejam errados, pois minhas 
resposta deu ilógicas 12 horas)


_
Seja um dos primeiros a testar o novo Windows Live Mail Beta- 
grátis. Acesse 
http://www.ideas.live.com/programpage.aspx?versionId=5d21c51a-b161-4314-9b0e-4911fb2b2e6d


=
Instruções para entrar na lista, sair da lista e usar a lista em
http://www.mat.puc-rio.br/~nicolau/olimp/obm-l.html
=


Re: [obm-l] iberoamericana

2007-07-11 Por tôpico Carlos Eddy Esaguy Nehab

Oi, Salhab,

Sua enorme produção na lista e seu jeito alegre de narrar suas 
soluções e tentativas de solução são certamente um incentivo a todos 
que aqui participam da lista, especialmente os novos...  O saldo é 
infinitamente favorável a você.


Pode ter certeza de minha admiração pela sua imensa participação 
aqui.  Também me delicio com seu jeito divertido e leve de 
conversar com sua própria solução...   Quando você solta um hehe 
dou gostosas gargalhadas.  É uma delícia lê-lo.


Você faz matemática com um enorme prazer e alegria e é uma das raras 
pessoas me dá faz sentir uma saudade danada de quando eu era jovem 
...  Você nem imagina quanto (talvez alguns coroas da lista que 
foram meus alunos identifiquem tal semelhança).Também nunca tive 
medo de errar.  O importante sempre foi tentar... e preservar a 
alegria do embate em busca de soluções...


Você é um professor nato...

Enorme abraço
Nehab


At 00:22 11/7/2007, you wrote:

Olá Nehab,
eita eita.. obrigado novamente pela correcao :)
acho que é a 3a vez q erro seguido aqui na lista.. hehe

abracos,
Salhab

On 7/10/07, Carlos Eddy Esaguy Nehab [EMAIL PROTECTED] wrote:


 Oi, Marcelo Salhab,

 O centro do círculo circunscrito está no encontro das mediatrizes e não nas
medianas.

 Nehab


 At 04:23 10/7/2007, you wrote:

Ola novamente,
 fiz um programinha em MATLAB pra plotar todos esses pontos..
 e adivinha? uma reta mesmo!

 segue abaixo o programa, basta colocar num m-file.
 function teste()
 A = [ 10 10 0 ];
 r = 2;
 ang = linspace(0, 2*pi, 1000);
 k = [ 0 0 1 ];
 for i = 1:100
M = [ r*cos(ang(i)) r*sin(ang(i)) 0 ];
s = (dot(A, M) + dot(M, M))/(2*dot(cross(A, k), M));
X = (A+M)/2 + s*cross(M-A, k);
ptos(i) = X(1) + j*X(2);
 end
 plot(ptos, 'x');

 mas ainda nao achei meu erro nos calculos..
 abracos,
 Salhab

 On 7/10/07, Marcelo Salhab Brogliato [EMAIL PROTECTED] wrote:

bom...
 fazendo as contas, cheguei em:
 X(xm-xa) + Y(ym-ya) = [r^2 - ||A||^2]/2
 onde o centro da circunferencia pedida esta em (X, Y)

 isto é... nada! ehehe
 acho que com isso posso dizer que nao será uma reta..
 mas tb nao sei o que sera..
 [usei o matlab pra fazer o algebrismo por mim.. entao acredito q nao
 esta errado]
 [agora, ate pensei em pedir pra ele calcular X^2 e Y^2 e ver o que
 da... mas ja fechei..]

 abracos,
 Salhab



 On 7/10/07, Marcelo Salhab Brogliato [EMAIL PROTECTED] wrote:
  Olá,
  pensei em uma abordagem usando vetores..
  vamos dizer que nossa circunferencia esta na origem.. e conhecemos os
  vetores M e A..
  como sabemos, o centro da circunferencia que passa por M, N e A é o
  encontro das medianas dos segmentos de reta MN e MA..
  M, N e A sao vetores no plano XY (isto é, nao possuem componente em Z)..
  x = produto vetorial
  . = produto escalar
 
  V1 = (M-A) x k .. este é o vetor diretor da mediana de MA
  (A+M)/2.. este é um ponto da mediana de MA...
  portanto, esta reta já esta determinada..
 
  V2 = M x k ... este é o vetor diretor da mediana de MN
  0.. este é um ponto da demana de MN
  portanto, esta reta tambem já esta determinada..
 
  temos que encontrar X, tal que:
  X = (A+M)/2 + s*V1
  X = t*V2
 
  X é o centro da circunferencia pedida..
  (A+M)/2 + s*[(M-A)xk] = t*[Mxk]
  fazendo o produto escalar por M, temos:
  [(A+M)/2].M + s*[(Mxk).M - (Axk).M] = t*[(Mxk).M]
  [A.M + M.M]/2 - s*[(Axk).M] = 0
  s = [A.M + M.M]/{2*[(Axk).M]}
 
  assim: X = (A+M)/2 + s*[(M-A)xk], onde s esta acima..
  agora, temos que A = (xa, ya) ; M = (xm, ym) ... substituir..
 
  vou fazer aki mais tarde... dai eu mando
 
  abracos,
  Salhab
 
 
  On 7/9/07, Klaus Ferraz [EMAIL PROTECTED] wrote:
  
   (Iberoamericana-2004)-Considera-se no plano uma
   circunferência de centro O e raio r, e um ponto A exterior a ela. Seja
M um
   ponto da circunferência e N o ponto diametralmente oposto a M.
Determinar o
   lugar geométrico dos centros das  circunferências que passam por A, M e
N
   quando M varia.
  
   ps. Eu tenho quase que certeza que é uma reta. Tentei analiticamente,
porém
   deu muitas contas e acabou num dando em nada.
   Flickr agora em português. Você cria, todo mundo vê. Saiba mais.
 

=
 Instruções para entrar na lista, sair da lista e usar a lista em
 http://www.mat.puc-rio.br/~nicolau/olimp/obm-l.html
=


=
Instruções para entrar na lista, sair da lista e usar a lista em
http://www.mat.puc-rio.br/~nicolau/olimp/obm-l.html
=


Re: [obm-l] Material sobre �lgebra de proposi�

2007-07-11 Por tôpico Carlos Eddy Esaguy Nehab

Oi Rafael,

O Leonidas Hegenberg (ITA), possui vários livros sobre o asssunto que 
são excelentes para os iniciantes (desde 1960...).


Veja EPU (Editora Pedagógica e Universitária):

Simbolização e Dedução
Cálculo Sentencial,
Exercícios I (Simbolização),
Exercícios II (Dedução),
Cálculo de Predicados,
Exercícios III (Simbolização),
Exercícios IV (Dedução)  e
Dicionário de Lógica

Você pode procurar nos sebos também, edições antigas:

Procure Leônidas Hegenberg em
http://www.traca.com.br

Abraços,
Nehab

At 21:21 9/7/2007, you wrote:

Olá, pessoal !

Acabei de estudar a teoria sobre lógica de proposições, argumentos 
dedutivos, silogismos e por aí vai ... Gostaria, agora, de algum 
materia e/ou site com MUITOS exercícios resolvidos para eu fixar os 
conceitos. Alguém poderia me ajudar ?


Re: [obm-l] iberoamericana

2007-07-10 Por tôpico Carlos Eddy Esaguy Nehab

Oi, Marcelo Salhab,

O centro do círculo circunscrito está no encontro das mediatrizes e 
não nas medianas.


Nehab

At 04:23 10/7/2007, you wrote:

Ola novamente,
fiz um programinha em MATLAB pra plotar todos esses pontos..
e adivinha? uma reta mesmo!

segue abaixo o programa, basta colocar num m-file.
function teste()
A = [ 10 10 0 ];
r = 2;
ang = linspace(0, 2*pi, 1000);
k = [ 0 0 1 ];
for i = 1:100
   M = [ r*cos(ang(i)) r*sin(ang(i)) 0 ];
   s = (dot(A, M) + dot(M, M))/(2*dot(cross(A, k), M));
   X = (A+M)/2 + s*cross(M-A, k);
   ptos(i) = X(1) + j*X(2);
end
plot(ptos, 'x');

mas ainda nao achei meu erro nos calculos..
abracos,
Salhab

On 7/10/07, Marcelo Salhab Brogliato [EMAIL PROTECTED] wrote:

bom...
fazendo as contas, cheguei em:
X(xm-xa) + Y(ym-ya) = [r^2 - ||A||^2]/2
onde o centro da circunferencia pedida esta em (X, Y)

isto é... nada! ehehe
acho que com isso posso dizer que nao será uma reta..
mas tb nao sei o que sera..
[usei o matlab pra fazer o algebrismo por mim.. entao acredito q nao
esta errado]
[agora, ate pensei em pedir pra ele calcular X^2 e Y^2 e ver o que
da... mas ja fechei..]

abracos,
Salhab



On 7/10/07, Marcelo Salhab Brogliato [EMAIL PROTECTED] wrote:
 Olá,
 pensei em uma abordagem usando vetores..
 vamos dizer que nossa circunferencia esta na origem.. e conhecemos os
 vetores M e A..
 como sabemos, o centro da circunferencia que passa por M, N e A é o
 encontro das medianas dos segmentos de reta MN e MA..
 M, N e A sao vetores no plano XY (isto é, nao possuem componente em Z)..
 x = produto vetorial
 . = produto escalar

 V1 = (M-A) x k .. este é o vetor diretor da mediana de MA
 (A+M)/2.. este é um ponto da mediana de MA...
 portanto, esta reta já esta determinada..

 V2 = M x k ... este é o vetor diretor da mediana de MN
 0.. este é um ponto da demana de MN
 portanto, esta reta tambem já esta determinada..

 temos que encontrar X, tal que:
 X = (A+M)/2 + s*V1
 X = t*V2

 X é o centro da circunferencia pedida..
 (A+M)/2 + s*[(M-A)xk] = t*[Mxk]
 fazendo o produto escalar por M, temos:
 [(A+M)/2].M + s*[(Mxk).M - (Axk).M] = t*[(Mxk).M]
 [A.M + M.M]/2 - s*[(Axk).M] = 0
 s = [A.M + M.M]/{2*[(Axk).M]}

 assim: X = (A+M)/2 + s*[(M-A)xk], onde s esta acima..
 agora, temos que A = (xa, ya) ; M = (xm, ym) ... substituir..

 vou fazer aki mais tarde... dai eu mando

 abracos,
 Salhab


 On 7/9/07, Klaus Ferraz [EMAIL PROTECTED] wrote:
 
  (Iberoamericana-2004)-Considera-se no plano uma
  circunferência de centro O e raio r, e um ponto A exterior a 
ela. Seja M um
  ponto da circunferência e N o ponto diametralmente oposto a M. 
Determinar o
  lugar geométrico dos centros das  circunferências que passam 
por A, M e N

  quando M varia.
 
  ps. Eu tenho quase que certeza que é uma reta. Tentei 
analiticamente, porém

  deu muitas contas e acabou num dando em nada.
  Flickr agora em português. Você cria, todo mundo vê. Saiba mais.



=
Instruções para entrar na lista, sair da lista e usar a lista em
http://www.mat.puc-rio.br/~nicolau/olimp/obm-l.html
=


Re: [obm-l] Probabilidade

2007-07-05 Por tôpico Carlos Eddy Esaguy Nehab

Oi, Salhab,

Acho que as contas de suas derivadas o enganaram...:  Veja na linha 
onde você afirma que:

...
isto é, podemos dizer que, se f(p1, b1), entao: f(x, y)  f(x, 
z) para yz

...

Não é verdadeiro não. Fixando  p1 = 3, por exemplo, f(x,3) (como 
função de x) é crescente de x=1 a 17 e descrescente de x =17 a 50.


Eu estou tentando uma solução sem análise (só com algebra) e ainda 
não consegui.  Um possível argumento, na sua linha poderia ser o fato 
que sua função é contínua num fechado (x e y entre 1 e 50)  e então 
haverá máximo e mínimo nem que seja na fronteira..., que é o 
caso  da solução do problema.


Note que se você imaginar a sua função como uma função de duas 
variáveis no R3:  z = f(x, y),  para cada valor de x fixado, a 
interseção do gráfico de f (que é uma superfície no R3) com o plano x 
= cte é uma soma de hipérboles (ou dado que y é restrito, pedaços 
de hipérboles) ...  Idem fixando y.


Se você tiver algum software para exibir =gráficos de funções com 
duas variáveis ficará mais fácil...


Abraços,
Nehab

PS: será que o Arthur Steiner (tão criativo e competente) não se 
interessa pelo problema e nos ajuda ?



At 03:10 5/7/2007, you wrote:

Olá Nehab,
obrigado pela correcao.. :))

pensei no seguinte:
2P = b1/(p1+b1) + b2/(p2+b2)
2P = b1/(p1+b1) + (50-b1)/(100-b1-p1)

vamos analisar como a funcao se comporta com p1...
derivando em relacao a p1 (como se a funcao fosse continua),
conseguimos mostrar que a funcao é decrescente (com 0 = p1 = 50)...
isto é, podemos dizer que, se f(p1, b1), entao: f(x, y)  f(x, z) para yz
assim, para maximiza-lo, precisamos pegar o menor valor possivel de 
p1..portanto: p1 = 0...

logo: 2P = 1 + (50-b1)/(100-b1)
agora, derivando em relacao a b1, vamos que a funcao é decrescente
tambem.. isto é, b1 deve ser o menor possivel.. portanto: b1 = 1 (pois
nenhuma caixa pode estar vazia)..

sera q esta certo?

abracos,
Salhab

On 7/4/07, Carlos Eddy Esaguy Nehab [EMAIL PROTECTED] wrote:


 Oi Salhab,

 Você se distraiu:  sua P vale
 P = 1/2 + 1/2 * 49/99e não  P = 1/2 + 1/2 * 49/50

 Olha que coincidência.  Este problema foi apresentado por um economista no
processo de seleção de meu filho há alguns anos e é realmente muito
interessante (na verdade ele formulou supondo que eram dois candidatos ao
emprego e que meu filho era um deles...- muito divertido e criativo...)

 A solução é colocar apenas 1 bola branca em uma urna e na outra as 99 bolas
restantes...A probabilidade é máxima e igual a  1/2 . 1 +  1/2 . 49/99 =
74,7% que é quase 75%

 Não tô achando uma solução simples para justificar a resposta.

 Abraços,
 Nehab


 At 10:47 4/7/2007, you wrote:

Olá,

 p1, b1 = quantidade de bolas pretas e brancas (respectivamente) na urna 1
 p2, b2 =  na urna 2

 b1+b2 = 50
 p1+p2 = 50

 vamos calcular a probabilidade da bola ser branca:
 P = 1/2 * b1/(p1+b1) + 1/2 * b2/(p2+b2)
 2P = b1/(p1+b1) + b2/(p2+b2)

 agora, temos que maximizar essa funcao..
 ainda estou pensando em como fazer isso..
 mas veja que: se b1 = 1 e p1 = 0 ... temos: P = 1/2 + 1/2 * 49/50 = 0,99
 uma probabilidade um tanto quanto alta :)
 provavelmente a máxima...

 abracos,
 Salhab


 On 7/3/07, Graciliano Antonio Damazo [EMAIL PROTECTED] wrote:

galera estou com dificuldade em pór no papel os calculos desse exercicio,
 pois eu imagino a resposta por intuição mas nao consigo chegar nas
 contas.me ajudem


 1) um prisioneiro possui 50 bolas brancas e 50 bolas pretas e duas urnas.
O
 prisioneiro deve colocar do modo que preferir as bolas nas duas
 urnas(nehunma urna pode ficar vazia). As urnas serao embaralhadas e o
 prisioneiro deverá, de olhas fechados, escolher uma urna e , nesta urna,
uma
 bola. Se a bola for branca, ele será libertado e , caso contrario,
 condenado. Como deve proceder o prisioneiro para maximizar  a probabilidade
 de ser libertado?

 desde já agradeço. Abraços


  
 Novo Yahoo! Cadê? - Experimente uma nova busca.


=
 Instruções para entrar na lista, sair da lista e usar a lista em
 http://www.mat.puc-rio.br/~nicolau/olimp/obm-l.html
=


=
Instruções para entrar na lista, sair da lista e usar a lista em
http://www.mat.puc-rio.br/~nicolau/olimp/obm-l.html
=


Re: [obm-l] Probabilidade

2007-07-04 Por tôpico Carlos Eddy Esaguy Nehab

Oi Salhab,

Você se distraiu:  sua P vale
P = 1/2 + 1/2 * 49/99e não  P = 1/2 + 1/2 * 49/50

Olha que coincidência.  Este problema foi apresentado por um 
economista no processo de seleção de meu filho há alguns anos e é 
realmente muito interessante (na verdade ele formulou supondo que 
eram dois candidatos ao emprego e que meu filho era um deles...- 
muito divertido e criativo...)


A solução é colocar apenas 1 bola branca em uma urna e na outra as 99 
bolas restantes...A probabilidade é máxima e igual a  1/2 . 1 
+  1/2 . 49/99 = 74,7% que é quase 75%


Não tô achando uma solução simples para justificar a resposta.

Abraços,
Nehab

At 10:47 4/7/2007, you wrote:

Olá,

p1, b1 = quantidade de bolas pretas e brancas (respectivamente) na urna 1
p2, b2 =  na urna 2

b1+b2 = 50
p1+p2 = 50

vamos calcular a probabilidade da bola ser branca:
P = 1/2 * b1/(p1+b1) + 1/2 * b2/(p2+b2)
2P = b1/(p1+b1) + b2/(p2+b2)

agora, temos que maximizar essa funcao..
ainda estou pensando em como fazer isso..
mas veja que: se b1 = 1 e p1 = 0 ... temos: P = 1/2 + 1/2 * 49/50 = 0,99
uma probabilidade um tanto quanto alta :)
provavelmente a máxima...

abracos,
Salhab


On 7/3/07, Graciliano Antonio Damazo [EMAIL PROTECTED] wrote:

galera estou com dificuldade em pór no papel os calculos desse exercicio,
pois eu imagino a resposta por intuição mas nao consigo chegar nas
contas.me ajudem


1) um prisioneiro possui 50 bolas brancas e 50 bolas pretas e duas urnas.  O
prisioneiro deve colocar do modo que preferir as bolas nas duas
urnas(nehunma urna pode ficar vazia). As urnas serao embaralhadas e o
prisioneiro deverá, de olhas fechados, escolher uma urna e , nesta urna, uma
bola. Se a bola for branca, ele será libertado e , caso contrario,
condenado. Como deve proceder o prisioneiro para maximizar  a probabilidade
de ser libertado?

desde já agradeço. Abraços


 
Novo Yahoo! Cadê? - Experimente uma nova busca.



=
Instruções para entrar na lista, sair da lista e usar a lista em
http://www.mat.puc-rio.br/~nicolau/olimp/obm-l.html
=


Re: [obm-l] picaretagem no ensino da matem�tica

2007-07-04 Por tôpico Carlos Eddy Esaguy Nehab

Calma gente,

Se a gente levar para o lado da brincadeira, este maluco até fez uma 
positiva propaganda da Matemática, sugerindo que ela nem é tão 
difícil assim e, se bem ensinada, qualquer passante na rua pode 
aprender a fazer contas rapidinho...


Bem pior é o que eu vejo no dia a dia escrito em livros adotados por ai...

Abraços,
Nehab

At 11:09 4/7/2007, you wrote:
Meu, esse cara é um babaca. Como pode se dizer ser um professor? E 
acha a raiz das ponta...


Quero agora que a fantástica RedeTV, ainda mais fantástica pela 
publicação desta fabulosa reportagem, desta incrível revolução no 
ensino da matemática, publique a prisão deste charlatão imbecil.


Ou será que a RedeTV pouco se importa com a formação do brasileiro? 
Querem apenas audiência da grande massa desinformada que admira o 
sucesso do método, nas palavras da jornalista imbecil.


Ele causou uma revolução realmente, querida entrevistadora que não 
sei o nome, e vai acabar por estragar ainda mais nosso querido ensino.


O pior é ver o coitado do sujeito que dizia achar super difícil e 
agora achar super fácil...


E que beleza de coordenadores desta escola estadual que convidaram o 
Sr. Místico para ensinar seus super métodos para os vestibulandos... 
será que ninguém se dá conta de que vestibulando já tem muito com o 
que sofrer?


Desculpem a revolta.
Bruno



2007/7/4, Marcelo Salhab Brogliato 
mailto:[EMAIL PROTECTED][EMAIL PROTECTED]:

Realmente, assustador!

abracos,
Salhab

On 7/4/07, Emanuel Valente mailto:[EMAIL PROTECTED] 
[EMAIL PROTECTED] wrote:

 É incrível como uma emissora possa colocar uma matéria dessas. O
 cidadão diz que desenvolveu algorítmos para as 4 operações básicas e
 promete levar ao ensino público. E pasmem, não é piada:

 
http://www.youtube.com/watch?v=7-644rpNVT4http://www.youtube.com/watch?v=7-644rpNVT4


 =
 Instruções para entrar na lista, sair da lista e usar a lista em
 
http://www.mat.puc-rio.br/~nicolau/olimp/obm-l.htmlhttp://www.mat.puc-rio.br/~nicolau/olimp/obm-l.html

 =


=
Instruções para entrar na lista, sair da lista e usar a lista em
http://www.mat.puc-rio.br/~nicolau/olimp/obm-l.htmlhttp://www.mat.puc-rio.br/~nicolau/olimp/obm-l.html 


=




--
Bruno França dos Reis
email: bfreis - http://gmail.comgmail.com
gpg-key: 
http://planeta.terra.com.br/informatica/brunoreis/brunoreis.keyhttp://planeta.terra.com.br/informatica/brunoreis/brunoreis.key

icq: 12626000

e^(pi*i)+1=0


Re: [obm-l] Re: [obm-l] picaretagem no ensino da matem�tica

2007-07-04 Por tôpico Carlos Eddy Esaguy Nehab

Ora, João,

Não quero te amolar, mas... o que você entende por macete?  Esta 
palavra infelizmente tem uma conotação negativa, mas nós não fazemos 
outra coisa nesta lista que não seja usar procedimentos, regras, 
memória anterior de solução de outros problemas, analogias  etc, etc.


Você acha que é feio dizer que usamos macetes?  Não acho.  Os macetes 
nada mais são do que experiência acumulada...


E quer queiramos ou não, 99% do que fazemos aqui é encontrar pulos 
do gato para problemas curisos e muitas vezes malucos.


Tudo bem, gostamos disto, mas   haja macetes!

Que o tal professor é um idiota, que a escola que o chamou 
ridícula, não se discute, mas nós todos podemos odiar a palavra 
macete, mas não vivemos sem eles...


Caramba, não sei onde estava com a cabeça que escrevi tanta 
inutilidade por causa de um idiota engraçado 


Abraços,
Nehab

At 15:15 4/7/2007, you wrote:
É, mas ele está também passando a idéia de que a matemática é, na 
verdade, um conjunto de macetes...

- Original Message -
From: mailto:[EMAIL PROTECTED]Carlos Eddy Esaguy Nehab
To: mailto:obm-l@mat.puc-rio.brobm-l@mat.puc-rio.br
Sent: Wednesday, July 04, 2007 2:39 PM
Subject: Re: [obm-l] picaretagem no ensino da matemática

Calma gente,

Se a gente levar para o lado da brincadeira, este maluco até fez uma 
positiva propaganda da Matemática, sugerindo que ela nem é tão 
difícil assim e, se bem ensinada, qualquer passante na rua pode 
aprender a fazer contas rapidinho...


Bem pior é o que eu vejo no dia a dia escrito em livros adotados por ai...

Abraços,
Nehab

At 11:09 4/7/2007, you wrote:
Meu, esse cara é um babaca. Como pode se dizer ser um professor? E 
acha a raiz das ponta...


Quero agora que a fantástica RedeTV, ainda mais fantástica pela 
publicação desta fabulosa reportagem, desta incrível revolução no 
ensino da matemática, publique a prisão deste charlatão imbecil.


Ou será que a RedeTV pouco se importa com a formação do brasileiro? 
Querem apenas audiência da grande massa desinformada que admira o 
sucesso do método, nas palavras da jornalista imbecil.


Ele causou uma revolução realmente, querida entrevistadora que 
não sei o nome, e vai acabar por estragar ainda mais nosso querido ensino.


O pior é ver o coitado do sujeito que dizia achar super difícil e 
agora achar super fácil...


E que beleza de coordenadores desta escola estadual que convidaram 
o Sr. Místico para ensinar seus super métodos para os 
vestibulandos... será que ninguém se dá conta de que vestibulando 
já tem muito com o que sofrer?


Desculpem a revolta.
Bruno



2007/7/4, Marcelo Salhab Brogliato 
mailto:[EMAIL PROTECTED][EMAIL PROTECTED] :

Realmente, assustador!
abracos,
Salhab
On 7/4/07, Emanuel Valente mailto:[EMAIL PROTECTED] 
[EMAIL PROTECTED] wrote:

 É incrível como uma emissora possa colocar uma matéria dessas. O
 cidadão diz que desenvolveu algorítmos para as 4 operações básicas e
 promete levar ao ensino público. E pasmem, não é piada:

 
http://www.youtube.com/watch?v=7-644rpNVT4http://www.youtube.com/watch?v=7-644rpNVT4 



 =
 Instruções para entrar na lista, sair da lista e usar a lista em
 
http://www.mat.puc-rio.br/~nicolau/olimp/obm-l.htmlhttp://www.mat.puc-rio.br/~nicolau/olimp/obm-l.html 


 =

=
Instruções para entrar na lista, sair da lista e usar a lista em
http://www.mat.puc-rio.br/~nicolau/olimp/obm-l.htmlhttp://www.mat.puc-rio.br/~nicolau/olimp/obm-l.html 


=




--
Bruno França dos Reis
email: bfreis - http://gmail.comgmail.com
gpg-key: 
http://planeta.terra.com.br/informatica/brunoreis/brunoreis.keyhttp://planeta.terra.com.br/informatica/brunoreis/brunoreis.key

icq: 12626000

e^(pi*i)+1=0


Re: [obm-l] Re: [obm-l] picaretagem no ensino da matem�tica

2007-07-04 Por tôpico Carlos Eddy Esaguy Nehab

Sensato, Avelino

Nehab

At 20:25 4/7/2007, you wrote:
O esquema da raiz quadrada realmente funciona pessoal, mas só para 
as raízes exatas de três dígitos. Editaram a filmagem e, acredito 
eu, acabaram cortando a explicação.
Também acho que ele exagerou quando disse que é um método 
revolucionário tendo em vista que a maior parte dos seus 
algoritmos são extremamentes restritos, de forma que mais 
se  parecem com macetes. Agora, se os métodos não funcionaram em 
alguns casos, é bom ir com calma antes de chamar ele de babaca. 
Provavelmente deve estar faltando alguma explicação e/ou restrição 
que deve ser analisada com um maior rigor antes de fuzilar o pobre coitado.
Assim, ao contrário de babaca, o que me parece é que ele tem uma boa 
intenção na tentativa de levar, ao menos interesse  a uma população 
em que, na minha opinião, 90% não sabem fazer uma operação simples 
de matemática.


Felipe Avelino
2007/7/4, Fellipe Rossi 
mailto:[EMAIL PROTECTED][EMAIL PROTECTED]:

Lamentável

Em 04/07/07, Rogerio Ponce 
mailto:[EMAIL PROTECTED][EMAIL PROTECTED]  
 escreveu:

Ola' pessoal,
ainda bem que ele nao refinou os macetes:

Por exemplo, quanto vale 199 / 995?
Ora, basta cortar os 99 de cima e de baixo, de modo que o 
resultado e' 1/5 ( ( pode conferir ! ).


Da mesma forma, calcular 424/742 tambem e' muito facil: basta 
simplificar o 42 de cima com o 42 de baixo, e o resultado e' 4/7.


Simples, nao?

[]'s
Rogerio Ponce

João Luís Gomes Guimarães 
mailto:[EMAIL PROTECTED][EMAIL PROTECTED]  escreveu:


Ou então a raiz de 784, por exemplo... é só desprezar o 8, e fazer a raiz
de 7 (?) e a raiz de 4...

é muito pilantra que há nesse mundo, mesmo...

- Original Message -
From: Romildo Franco
To:
Sent: Wednesday, July 04, 2007 12:58 PM
Subject: Re: [obm-l] picaretagem no ensino da matemática


 Manda ele achar a raiz de 962 agora.
 =
 Instruções para entrar na lista, sair da lista e usar a lista em
 
http://www.mat.puc-rio.br/%7Enicolau/olimp/obm-l.htmlhttp://www.mat.puc-rio.br/~nicolau/olimp/obm-l.html

 =


=
Instruções para entrar na lista, sair da lista e usar a lista em
http://www.mat.puc-rio.br/%7Enicolau/olimp/obm-l.htmlhttp://www.mat.puc-rio.br/~nicolau/olimp/obm-l.html 


=



Novo http://yahoo.com.br/oqueeuganhocomisso+Yahoo! Cadê? - 
Experimente uma nova busca.





--
Fellipe Rossi



Re: [obm-l] Ajuda

2007-07-03 Por tôpico Carlos Eddy Esaguy Nehab

Oi,

Faça x = 1 e você verá que de fato, como o Rafael também mostrou, há 
algum erro no enunciado.

Nehab

At 02:07 3/7/2007, you wrote:
Será que pode fazer isso mesmo sabendo que a função é par ou ímpar, 
no caso por exemplo p(-2) = p(2)


Em 03/07/07, rgc 
mailto:[EMAIL PROTECTED][EMAIL PROTECTED] escreveu:

Veja se o enunciado está certo porque eu cheguei num absurdo:
seja x = -2 -- 2p(-2) - 2p(4) = 16
seja x = 4 -- 2p(4) - 2p(-2) = 34
Somando: 0=50 --absurdo!!!

- Original Message -
From: mailto:[EMAIL PROTECTED]Marcelo Costa
To: mailto:obm-l@mat.puc-rio.brobm-l@mat.puc-rio.br
Sent: Tuesday, July 03, 2007 12:24 AM
Subject: [obm-l] Ajuda

Alguém poderia me auxiliar nesta???
Dado o polinômio p(x) tal que 2p(x) - 2p(2 - x) = 3x^2 - 3x - 2 para 
todo x real, o valor de p( - 2) + p(4) é:

a) 4
b) 16
c) 34
d) 50
e) 66

Valeu, obrigado




--
[]'s


Re: [obm-l] Ajuda

2007-07-03 Por tôpico Carlos Eddy Esaguy Nehab

Oi, Marcelo

Você conseguiu mais uma vacuidade aqui da lista (este negócio andou 
na moda há algum tempo: rsrsrsr).

Conseguiu descobrir um valor sobre um polinômio que não existe...

Abraços,
Nehab

At 18:08 3/7/2007, you wrote:

Perdão, parece molecagem, mas acabei de tanto tentar, conseguir resolver.
x = - 2, 2p(-2) - p(4) = 16
x = 4, 2p(4) - p(-2) = 34
fazendo p(-2) = x e p(4) = y teremos:
2x - y = 16
- x + 2y = 34,
somando as expressões
x + y = 50 que é a resposta
obrigado e desculpem o incômodo

Em 03/07/07, Carlos Eddy Esaguy Nehab 
mailto:[EMAIL PROTECTED][EMAIL PROTECTED]  escreveu:

Oi,

Faça x = 1 e você verá que de fato, como o Rafael também mostrou, há 
algum erro no enunciado.

Nehab


At 02:07 3/7/2007, you wrote:

Será que pode fazer isso mesmo sabendo que a função é par ou ímpar, no caso

por exemplo p(-2) = p(2)

Em 03/07/07, rgc 
mailto:[EMAIL PROTECTED][EMAIL PROTECTED]  escreveu:

Veja se o enunciado está certo porque eu cheguei num absurdo:
seja x = -2 -- 2p(-2) - 2p(4) = 16
seja x = 4 -- 2p(4) - 2p(-2) = 34
Somando: 0=50 --absurdo!!!

- Original Message -
From: mailto:[EMAIL PROTECTED]Marcelo Costa
To: mailto:obm-l@mat.puc-rio.brobm-l@mat.puc-rio.br
Sent: Tuesday, July 03, 2007 12:24 AM
Subject: [obm-l] Ajuda
Alguém poderia me auxiliar nesta???
Dado o polinômio p(x) tal que 2p(x) - 2p(2 - x) = 3x^2 - 3x - 2 para 
todo x real, o valor de p( - 2) + p(4) é:

a) 4
b) 16
c) 34
d) 50
e) 66
Valeu, obrigado




--
[]'s



[obm-l] Sugest

2007-07-02 Por tôpico Carlos Eddy Esaguy Nehab

A todos,

Seria extremamente interessante se pudéssemos, quando fosse o caso, 
explicitar (com mais freqüência) a origem dos problemas que  postamos 
na nossa lista, especialmente quando é um teorema ou problema clássico.


É claro que também é possível que os colegas que ajudam com a solução 
possam conhecer tal origem e contribuir com esta informação.


Abraços,
Nehab

=
Instruções para entrar na lista, sair da lista e usar a lista em
http://www.mat.puc-rio.br/~nicolau/olimp/obm-l.html
=


Re: [obm-l] Conjunto

2007-06-29 Por tôpico Carlos Eddy Esaguy Nehab

Não reisti à tentação:

Faça um diagrama dos dois conjuntos (C e H) e imagine que na 
interseção haja  X  alunos.   Então há 4X  em C e 3X em H (pense no 
20% e no 25%) e a união conterá x +3x +4x = 8x.  Logo... x = 6 ...


Abraços,
Nehab


At 12:12 29/6/2007, you wrote:
Pessoal peço encarecidamente que me ajudem com essa questão pois 
estou conseguindo solucioná-la de jeito nenhum. Desde já agradeço! 
Abraço a todos!


Um grupo de alunos de uma escola deveria visitar o Museu de Ciência 
e o Museu de História da cidade. Quarenta e oito alunos foram 
visitar pelo menos um desses museus. 20% dos que foram ao de Ciência 
visitaram o de História e 25% dos que foram ao de História visitaram 
também o de Ciência. Calcule o número de alunos que visitaram os dois museus.




Novo http://yahoo.com.br/oqueeuganhocomissoYahoo! Cadê? - 
Experimente uma nova busca.


Re: [obm-l] D�vida

2007-06-21 Por tôpico Carlos Eddy Esaguy Nehab

Oi, Nicolau (e demais colegas envolvidos com este problema)...

Ah se eu tivesse como qualidade uma pequena dose que fosse do seu 
pragmatismo...!!!


Sua primeira solução (que eu havia conseguido fazer) e me lembra um 
exercício de 2005 do IME (que segue a mesma idéia da recorrência):


IME 2005: Sejam a, b e c as raízes do polinômio p(x) = x^3 + r x - 
t  onde r e s são números reais não nulos.

a) Determine a^3 + b^3 + c^3 em função de r e s;
b) Demostre que S^(n+1) + rS^(n-1)  -t S(n-2) = 0  para todo número 
natural n=2, onde S(k) = a^k + b^k +c^k

para qualquer número natural k.

Mas quando eu percebi que tinha que fazer aquelas contas desisti 
deste caminho, pois fui menos pragmático (um dos grandes defeitos que 
tenho) e pensei:  e se o enunciado pedisse  a^2001+b^2001+c^2001?   O 
que eu faria?   Certamente não seriam contas como 
aquelas.   Pensamento talvez romântico, mas ai fiquei tentando 
chegar no 21 sem passar pelas contas e confesso que não consegui...


Até usei o fato que p_(n+4) - p_(n+3) = p_(n+3) - p_(n)  para as 
contas ficarem mais rápidas (pelas diferenças), mas não me 
satisfiz...  Ah perfeccionismo...


Vivendo e aprendendo
Um grande abraço,
Nehab

At 11:15 21/6/2007, you wrote:

On Thu, Nov 01, 2001 at 02:02:41AM -0300, Pedro Costa wrote:
 Amigos da lista, me dê uma idéia resolver esta questão:

 Se a, b e c são números complexos tais que a+b+c = 1, a^2+b^2+c^2 = 3 e
 a^3+b^3+c^3 = 7, determine o valor de a^21+b^21+c^21.

Sejam X = ab+ac+bc, Y = a^2b + ab^2 + a^2c + ac^2 + b^2c + bc^2, Z = abc.
Temos
(a+b+c)^2 = (a^2+b^2+c^2) + 2(ab+ac+bc)
1 = 3 + 2X
X = -1

(ab+ac+bc)(a+b+c) = (a^2b+ab^2+a^2c+ac^2+b^2c+bc^2) + 3abc
-1 = Y + 3Z

(a+b+c)^3 = (a^3+b^3+c^3) + 3(a^2b+ab^2+a^2c+ac^2+b^2c+bc^2) + 6abc
-1 = 7 + 3Y + 6Z

Y = -4, Z = 1

Assim a, b, c são as raízes de x^3 - x^2 - x - 1 = 0.
Podemos observar que a seqüência p_n = a^n+b^n+c^n satisfaz
p_(n+3) = p_(n+2) + p_(n+1) + p_n donde obtemos os valores abaixo para p_n:

p_1  = 1
p_2  = 3
p_3  = 7
p_4  = 11
p_5  = 21
p_6  = 39
p_7  = 71
p_8  = 131
p_9  = 241
p_10 = 443
p_11 = 815
p_12 = 1499
p_13 = 2757
p_14 = 5071
p_15 = 9327
p_16 = 17155
p_17 = 31553
p_18 = 58035
p_19 = 106743
p_20 = 196331
p_21 = 361109

Assim a^21+b^21+c^21=p_21=361109.

Alternativamente, depois de encontrar o polinômio de raízes a,b,c
podemos considerar a matriz N = [[0,0,1],[1,0,1],[0,1,1]]
cujos autovalores são a,b,c.

[001]
N = [101]
[011]

Temos

  [011]
N^2 = [012]
  [112]

  [124]
N^4 = [236]
  [247]

  [24 7]
N^5 = [3611]
  [4713]

   [44 81149]
N^10 = [68125230]
   [81149274]

   [1951335890 66012]
N^20 = [3012255403101902]
   [3589066012121415]

   [35890 66012121415]
N^21 = [55403101902187427]
   [66012121415223317]

Observe que cada matriz pode ser calculada como um produto de duas das
matrizes anteriores.

Finalmente temos a^21+b^21+c^21=traço(N^21)=361109
(e chegamos na mesma resposta).

[]s, N.
=
Instruções para entrar na lista, sair da lista e usar a lista em
http://www.mat.puc-rio.br/~nicolau/olimp/obm-l.html
=


Re: [obm-l] D�vida

2007-06-20 Por tôpico Carlos Eddy Esaguy Nehab

Oi, gente,

Não acho que a solução por complexos dê frutos.  Mas a questão é 
mesmo nojentinha.  No sábado terei mais tempo e tentarei fechá-la 
como o Rennó comentou (ainda não consegui e não foi por preguiça, não).


Claro que se algum colega souber o pulo do gato não faça 
cerimônia...  Adorarei não ter que torrar parte dos poucos neurônios 
que ainda funcionam...


Abraços,
Nehab

At 19:23 20/6/2007, you wrote:

Olá Ronaldo,

Será que a solução do problema seguiria por esse caminho? Não seria 
possível utilizar apenas produtos notáveis para resolver? Assim como 
o Nehab e o Salhab estavam tentando?


On 6/20/07, ralonso 
mailto:[EMAIL PROTECTED][EMAIL PROTECTED] wrote:

Olá Henrique.  Você tem 3 equações e três incógnitas alfa, beta e gamma.
Resolva o sistema, ache alfa, beta e gamma.
   Escreva alfa como:
alfa = cos w + i sen w,
   alfa^21 = cos 21w + i sen 21w

  fazendo o mesmo para beta e gamma e some os três.

[]s
Ronaldo.

Henrique Rennó wrote:

Olá Pedro,

Você poderia dizer qual a fonte deste problema? De onde ele foi tirado?
 On 11/1/01, Pedro Costa mailto:[EMAIL PROTECTED] 
[EMAIL PROTECTED] wrote:


Amigos da lista, me dê uma idéia resolver esta questão:

Se
[]
e
[]
são números complexos tais que
[]
,
[]
e

[]
, determine o valor de
[]
.


--
Henrique




--
Henrique
Content-Type: image/gif; name=clip_image004.gif
Content-ID: [EMAIL PROTECTED]
X-Attachment-Id: 0.1.2
Content-Disposition: inline; filename=clip_image004.gif

Content-Type: image/gif; name=clip_image008.gif
Content-ID: [EMAIL PROTECTED]
X-Attachment-Id: 0.1.4
Content-Disposition: inline; filename=clip_image008.gif

Content-Type: image/gif; name=clip_image012.gif
Content-ID: [EMAIL PROTECTED]
X-Attachment-Id: 0.1.6
Content-Disposition: inline; filename=clip_image012.gif

Content-Type: image/gif; name=clip_image002.gif
Content-ID: [EMAIL PROTECTED]
X-Attachment-Id: 0.1.1
Content-Disposition: inline; filename=clip_image002.gif

Content-Type: image/gif; name=clip_image010.gif
Content-ID: [EMAIL PROTECTED]
X-Attachment-Id: 0.1.5
Content-Disposition: inline; filename=clip_image010.gif

Content-Type: image/gif; name=clip_image006.gif
Content-ID: [EMAIL PROTECTED]
X-Attachment-Id: 0.1.3
Content-Disposition: inline; filename=clip_image006.gif

inline: e963c.gifinline: e9646.gifinline: e9650.gifinline: e965a.gifinline: e9664.gifinline: e966e.gif

Re: [obm-l] D�vida

2007-06-18 Por tôpico Carlos Eddy Esaguy Nehab

Oi, Salhab,

Não consegui enxergar o enunciado do problema em meu Eudora, mas... 
acompanhando sua proposta de solução...
Desenvolvendo X = (a+b+c)(a^2+b^2+c^2) , conseguimos o valor de abc 
que você mencionou:


X = a^3 +b^3 + c^3  + ab(a+b) + bc(b+c) + ac(a+c)
X = a^3 +b^3 + c^3  + ab(1-c) + bc(1-a) + ac(1-b)
X = a^3 +b^3 + c^3  + (ab+bc+ac) - 3abc
Logo, temos:  1.3 =  7 + (-1) - 3abc  ou seja, abc = 1

Logo, seu polinomio  é
x3 - x2 -x -1 = 0.  Fazendo z = x +1/3 elimina-se o termo em 
x^2  obtendo-se (se eu na errei nas contas)

x^3 - 4/3 x  - 38/27 =0 que é uma cubica padrão (modelito Cardano).

Abraços,
Nehab

PS: Eu gosto de apresentar como produtos notáveis as relações que 
se seguem, muito uteis qdo rola cubo...

(a + b + c)3  = a3 +b3 +c3 + 3(a + b + c)(ab + bc + ac) – 3abc
(a + b + c)3  = a3 +b3 +c3 + 3(a + b)(b + c)(a + c)

At 21:51 17/6/2007, you wrote:

Ola,

Vamos dizer que alfa = a, beta = b, gamma = c... entao:

a + b + c = 1
a^2 + b^2 + c^2 = 3
a^3 + b^3 + c^3 = 7

(a+b+c)^2 = a^2 + b^2 + c^2 + 2(ab + bc + ac) = 1^2
assim: 3 + 2(ab + bc + ac) = 1  ab + bc + ac = -1

(a+b+c)^3 = a^3 + b^3 + c^3 + 3(a^2b + a^2c + ab^2 + ac^2 + bc^2) + 6abc = 1^3
7 + 3(a^2b + a^2c + ab^2 + ac^2 + bc^2) + 6abc = 1
3(a^2b + a^2c + ab^2 + ac^2 + bc^2) + 6abc = -6
(a^2b + a^2c + ab^2 + ac^2 + bc^2) + 2abc = -2

bom, a ideia eh achar o valor de abc.. dai montamos um polinomio do 
3o. grau...
ja sabemos que ele eh a da forma: x^3 - x^2 - x - (abc)... assim, 
basta acharmos as raizes..


abraços,
Salhab





On 11/1/01, Pedro Costa mailto:[EMAIL PROTECTED] [EMAIL PROTECTED] wrote:

Amigos da lista, me dê uma idéia resolver esta questão:

See são números complexos tais que ,  e

, determine o valor de .

Internal Virus Database is out-of-date.
Checked by AVG Anti-Virus.
Version: 7.0.289 / Virus Database: 0.0.0 - Release Date: unknown



Re: [obm-l] D�vida

2007-06-18 Por tôpico Carlos Eddy Esaguy Nehab

Salhab

Você poderia me mandar o enunciado do problema, pois não consegui 
lê-lo.   O que se pede é a^7+b^7+c^7 ?


Não vai dar para fazer a forma polar nao, pois não é nada fácil 
encarar isto no Cardano.  De qualquer forma se você puder me mandar o 
enunciado, tentarei alguma solução mais acessível.


Abraços,
Nehab


At 13:53 18/6/2007, you wrote:

Olá Nehab,

obrigado por continuar minha solucao.. e gostei dos produtos
notaveis.. nao conhecia! mas já estao anotados! :)

agora, Pedro, basta encontrar as raizes do polinomio e fazer: a^7 + 
b^7 + c^7..

hmm uma sugestao eh trabalhar na forma polar :)

abraços,
Salhab

On 6/18/07, Carlos Eddy Esaguy Nehab [EMAIL PROTECTED] wrote:


 Oi, Salhab,

 Não consegui enxergar o enunciado do problema em meu Eudora, mas...
acompanhando sua proposta de solução...
 Desenvolvendo X = (a+b+c)(a^2+b^2+c^2) , conseguimos o valor de abc que
você mencionou:

 X = a^3 +b^3 + c^3  + ab(a+b) + bc(b+c) + ac(a+c)
 X = a^3 +b^3 + c^3  + ab(1-c) + bc(1-a) + ac(1-b)
 X = a^3 +b^3 + c^3  + (ab+bc+ac) - 3abc
 Logo, temos:  1.3 =  7 + (-1) - 3abc  ou seja, abc = 1

 Logo, seu polinomio  é
 x3 - x2 -x -1 = 0.  Fazendo z = x +1/3 elimina-se o termo em x^2
obtendo-se (se eu na errei nas contas)
 x^3 - 4/3 x  - 38/27 =0 que é uma cubica padrão (modelito Cardano).

 Abraços,
 Nehab

 PS: Eu gosto de apresentar como produtos notáveis as relações que se
seguem, muito uteis qdo rola cubo...
 (a + b + c)3  = a3 +b3 +c3 + 3(a + b + c)(ab + bc + ac) ­ 3abc
 (a + b + c)3  = a3 +b3 +c3 + 3(a + b)(b + c)(a + c)


 At 21:51 17/6/2007, you wrote:

Ola,

 Vamos dizer que alfa = a, beta = b, gamma = c... entao:

 a + b + c = 1
 a^2 + b^2 + c^2 = 3
 a^3 + b^3 + c^3 = 7

 (a+b+c)^2 = a^2 + b^2 + c^2 + 2(ab + bc + ac) = 1^2
 assim: 3 + 2(ab + bc + ac) = 1  ab + bc + ac = -1

 (a+b+c)^3 = a^3 + b^3 + c^3 + 3(a^2b + a^2c + ab^2 + ac^2 + bc^2) + 6abc =
1^3
 7 + 3(a^2b + a^2c + ab^2 + ac^2 + bc^2) + 6abc = 1
 3(a^2b + a^2c + ab^2 + ac^2 + bc^2) + 6abc = -6
 (a^2b + a^2c + ab^2 + ac^2 + bc^2) + 2abc = -2

 bom, a ideia eh achar o valor de abc.. dai montamos um polinomio do 3o.
grau...
 ja sabemos que ele eh a da forma: x^3 - x^2 - x - (abc)... assim, basta
acharmos as raizes..

 abraços,
 Salhab





 On 11/1/01, Pedro Costa  [EMAIL PROTECTED]
 wrote:

 Amigos da lista, me dê uma idéia resolver esta questão:

 See são números complexos tais que ,  e

 , determine o valor de .

 Internal Virus Database is out-of-date.
 Checked by AVG Anti-Virus.
 Version: 7.0.289 / Virus Database: 0.0.0 - Release Date: unknown



=
Instruções para entrar na lista, sair da lista e usar a lista em
http://www.mat.puc-rio.br/~nicolau/olimp/obm-l.html
=


Re: RES: [obm-l] 2^x = x^2

2007-06-16 Por tôpico Carlos Eddy Esaguy Nehab

Oi, Ponce,

Adorei, mas depois neguinho te chama de velho e você não pode 
reclamar.  Mas que tavam complicando ah... lá isto isto 
tavam  Método de Newton (acho que tinha um tal de Raphson 
também, ou tô caduco?), ora pipocas, como diria minha velha...


Grandes abraços,
Nehab

At 11:30 16/6/2007, you wrote:

Ola' pessoal,
queria lembrar que nao e' necessario software especial, e nem 
computador para calcularmos as raizes de 2^x = x^2 . Qualquer 
calculadora cientifica da' conta do recado, antes que o XP entre no ar...:-)


Brincadeiras 'a parte, vamos ao trabalho !

Relembrando o metodo de Newton : para uma funcao bem comportada 
y=f(x) , a aplicacao sucessiva de

  x2 = x1 - y1/ f'(x1)
a partir de um ponto x1 , que esteja na vizinhanca de uma raiz 
de f(x) , nos leva 'a propria raiz.

Considerando nossa funcao
 y=2^x - x^2
temos que
 x2 = x1 - (2^x - x^2) / ( 2^x * ln2 - 2*x )

Conforme o Nehab havia dito, vemos que uma das raizes esta' entre 0 e -1.
Tomando-se x1 = -0.5 , obtemos x2=-0.8067565 .
Reintroduzindo esse valor em x1, obtemos o proximo x2=-0.7673536
Na terceira iteracao, obtemos x2= -0.7491 , e na quarta iteracao 
x2=-0.74696 .
Nada mal, para quem dispuser de apenas 5 minutos, lapis, papel e uma 
calculadora barata...


[]'s
Rogerio Ponce


Érica Gualberto Pongelupe [EMAIL PROTECTED] escreveu:
Oi Todo mundo
use um software de geometria dinâmica, por exemplo, o Cabri, ou 
mesmo um software do tipo Graphmatica que vc verá claramente as três raizes.

Abração
Érica
Oi, Arthur (e Julio),

Você esqueceu que x pode ser negativo.  Para x positivo, ok. Mas, 
faça um grafiquinho simples de y = x^2 e y = 2^x  e você veráque 
obviamente há uma raiz negativa (entre -1 e 0).


Abraços,
Nehab

At 11:08 15/6/2007, you wrote:
Por inspecao, vemos que 2 e 4 sao raizesdesta equacao. Resta agora 
analisar se hah outras raizes. Temos 2^x = x^2se, e somente se, x 
ln(2) = 2 ln(x), ou seja, sse ln(x)/x = ln(2)/2. Sejaa funcao 
definida em (0, oo) por f(x) = ln(x)/x. Temos que f'(x) = (1 
-ln(x))/x^2, do que concluimos que f' se anula em x* = e.  A 
esquerdade e, f' eh positiva e, aa direita, eh negativa, o que nos 
mostra que fpassa por um maximo global em x* = e, para o quel f(x*) 
= 1/e. Destaforma, f eh estritamente crecente m (0, e) e 
estritamente decrescente em(e, oo). Temos ainda que f eh continua, 
que lim x - 0+ f(x) = -oo eque lim x - oo f(x) = 0. Isso implica 
que, em (0, e) f assuma umaunica vez todos os reais em (-oo, 1/e) e 
que, em (e,oo) , assuma umaunica vez todos os reais em (1/e, 
0).  Concluimos assim que , paraa0, a1/e, a equacao ln(x)/x = a 
tem exatamente duas raizes emR. Como ln(2)/2  1/e, ha exatamente 
2 reais satisfazendo ln(x)/x= ln(2)/2. Logo, 2 e 4 sao as duas 
unicas raizes reais de 2^x = x^2.


Serah que hah outras raizes complexasnao reais?

Artur






 -Mensagem original-
De: [EMAIL PROTECTED]:[EMAIL PROTECTED] 
nome de Julio Sousa

Enviada em: quinta-feira, 14 de junho de 2007 19:38
Para: obm-l@mat.puc-rio.br
Assunto: [obm-l] 2^x = x^2

achar as raízes de 2^x = x^2

--
Atenciosamente
Home Page: http://www.rumoaoita.com/rumoaoita.com
Júlio Sousa

--



Novo http://yahoo.com.br/oqueeuganhocomissoYahoo! Cadê? - 
Experimente uma nova busca.


Re: RES: [obm-l] 2^x = x^2

2007-06-15 Por tôpico Carlos Eddy Esaguy Nehab

Oi, Arthur (e Julio),

Você esqueceu que x pode ser negativo.  Para x positivo, ok.  Mas, 
faça um grafiquinho simples de y = x^2 e y = 2^x  e você verá que 
obviamente há uma raiz negativa (entre -1 e 0).


Abraços,
Nehab

At 11:08 15/6/2007, you wrote:
Por inspecao, vemos que 2 e 4 sao raizes desta equacao. Resta agora 
analisar se hah outras raizes. Temos 2^x = x^2 se, e somente se, x 
ln(2) = 2 ln(x), ou seja, sse ln(x)/x = ln(2)/2. Seja a funcao 
definida em (0, oo) por f(x) = ln(x)/x. Temos que f'(x) = (1 - 
ln(x))/x^2, do que concluimos que f' se anula em x* = e.  A esquerda 
de e, f' eh positiva e, aa direita, eh negativa, o que nos mostra 
que f passa por um maximo global em x* = e, para o quel f(x*) = 1/e. 
Desta forma, f eh estritamente crecente m (0, e) e estritamente 
decrescente em (e, oo). Temos ainda que f eh continua, que lim x - 
0+ f(x) = -oo e que lim x - oo f(x) = 0. Isso implica que, em (0, 
e) f assuma uma unica vez todos os reais em (-oo, 1/e) e que, em 
(e,oo) , assuma uma unica vez todos os reais em (1/e, 
0).  Concluimos assim que , para a0, a1/e, a equacao ln(x)/x = a 
tem exatamente duas raizes em R. Como ln(2)/2  1/e, ha exatamente 
2 reais satisfazendo ln(x)/x = ln(2)/2. Logo, 2 e 4 sao as duas 
unicas raizes reais de 2^x = x^2.


Serah que hah outras raizes complexas nao reais?

Artur






 -Mensagem original-
De: [EMAIL PROTECTED] [mailto:[EMAIL PROTECTED] 
nome de Julio Sousa

Enviada em: quinta-feira, 14 de junho de 2007 19:38
Para: obm-l@mat.puc-rio.br
Assunto: [obm-l] 2^x = x^2

achar as raízes de 2^x = x^2


--
Atenciosamente
Home Page: http://www.rumoaoita.comrumoaoita.com
Júlio Sousa


Re: [obm-l] Treinando pra Olimpiada

2007-05-20 Por tôpico Carlos Eddy Esaguy Nehab

Oi, Rivaldo,

Há alguns dias postei uma mensagem dando a dica de um link sobre 
cúbicas e raízes de equações do terceiro grau que certamente o 
interessarão, pois abordam exatamente o que você procura (e de uma 
forma muito interessante).  Para você não ter trabalho, ai vão os links:


http://www.m-a.org.uk/docs/library/2059.pdf
http://www.m-a.org.uk/docs/library/2060.pdf

Abraços,
Nehab
=
At 17:11 20/5/2007, you wrote:


Suponha que a equação de coeficientes reais X^3+cx+d=0,
admita 3 raizes reais. Mostrar que uma das raizes dessa equação
é dada pela formula

x= (-3d/2c)-(M)raiz(L)/(6ci), onde:

L=12c^3+81d^2   M=senp/(1-cosp)   i=raiz(-1)


   p=(1/3)arccos(H) H=(54d^2+4c^3)/(-4c^3)



Obs1_  Na formula acima estamos supondo c e p diferentes de zero. No
caso em que c=0 ou p=0, a equação acima tem solução trivial.

Obs2_ A hipotese da equação ter 3 raizes reais é equivalente a afirmar
que o numero L é menor ou igual a zero.

Obs3_ A formula acima não vale quando L 0, isto é , quando a equação
não admite 3 raizes reais.

Abs.

Rivaldo.

=
Instruções para entrar na lista, sair da lista e usar a lista em
http://www.mat.puc-rio.br/~nicolau/olimp/obm-l.html
=


[obm-l] Equa��es do terceiro grau e c�bicas

2007-05-16 Por tôpico Carlos Eddy Esaguy Nehab

Colegas da Lista

Como são recorrentes na Lista as equações do terceiro grau segue a 
referência a um artigo extremamente criativo sobre cúbicas que, 
numa abordagem interessantíssima, analisa aspectos (invariantes) de 
sua geometria.   Eu gostei muito.


http://www.m-a.org.uk/docs/library/2059.pdf

Este artigo gerou dezenas de outros: basta procurar no google por a 
new approach to solving the cubic para encontrá-los.


O próprio autor tem um outro artigo (aplicado a um problema específico) em
http://www.m-a.org.uk/docs/library/2060.pdf

Abraços,
Nehab

At 13:58 15/5/2007, you wrote:


De: [EMAIL PROTECTED]
Para: obm-l@mat.puc-rio.br
Cópia:
Data: Mon, 14 May 2007 19:25:33 -0300 (BRT)
Assunto: [obm-l] equação do terceiro grau
 Resolver a equação 8x^3 - 6x - 1 = 0


Seja f(x) = 8x^3 - 6x - 1

f(-1) = -3  0
f(-1/2) = 1  0 == tem uma raiz entre -1 e -1/2
f(0) = -1  0 == tem uma raiz entre -1/2 e 0
f(1) = 1  0  == tem uma raiz entre 0 e 1

Ou seja, f tem 3 raízes reais, todas de módulo  1.
Logo, podemos expressá-las na forma x = cos(t).

Sabemos que cos(3t) é um polinômio de 3o. grau em cos(t).
Especificamente,
cos(3t) = cos(2t+t) =
cos(2t)cos(t) - sen(2t)sen(t) =
(2*cos^2(t) - 1)*cos(t) - 2*sen^2(t)*cos(t) =
2*cos^3(t) - cos(t) - 2*cos(t) + 2*cos^3(t) =
4*cos^3(t) - 3*cos(t)   (que sorte...)

x = cos(t) é raiz da equação ==
8*cos^3(t) - 6*cos(t) - 1 = 0 ==
2*cos(3t) = 1 ==
cos(3t) = 1/2.

Se quisermos t no intervalo [0,2pi), teremos:
3t = pi/3 ou 5pi/3 ou 7pi/3 ou 11pi/3 ou 13pi/3 ou 17pi/3 ==
t = pi/9 ou 5pi/9 ou 7pi/9 ou 11pi/9 ou 13pi/9 ou 17pi/9 ==
cos(t) = cos(pi/9) ou cos(5pi/9) ou cos(7pi/9)
(pois cos(11pi/9) = cos(7pi/9), cos(13pi/9) = cos(5pi/9) e 
cos(17pi/9) = cos(pi/9))


Logo, as raízes da equação são:
cos(pi/9), cos(5pi/9) e cos(7pi/9).

[]s,
Claudio.


Re: [obm-l] Multiplos de 5 , 8 e 11

2007-05-16 Por tôpico Carlos Eddy Esaguy Nehab

Oi, Rafael,

Você não deveria ter aberto mão de ir da esquerda para a direita para 
tentar preservar ao máximo os algarismos em ordem decrescente.   Note 
que é possível você ter 98765abcd0  e ajeitar  o 4, 3, 2 e 1 para 
obter o múltiplo de 8 e 11.  Assim, se eu também não me distrai, 
9.876.513.240 deve ser o maior.


Abraços,
Nehab

PS: Este problema é mais simples que outro que já caiu Olimpíada de 
Maio de 2003 (e não me engano) e cuja solução também usa uma certa 
força bruta:  qual o menor múltiplo (naturalmente positivo) de 56 
cuja soma dos algarismos é 56 e que termina em 56...


At 18:44 16/5/2007, you wrote:

Oi
Não ficou muito formal mas acho que consegui achar o numero. Deem 
uma confirida pra ver se está tudo certo.
Seja n o numero: n deve ser divisivel por 5, 8 e 11. Pra ser por 5 
deve acabar em 5 ou 0. Mas pra ser por 8 deve ser par. Logo acaba em 0.
Como os algarismos devem ser distintos vamos tentar colocar os 
menores algarismos perto do 0 pra n ser o maior possivel.
Fazendo n terminar em 210 não é divisivel por 8. Invertendo a ordem 
temos: n termina em 120. Isso faz n ser divisivel por 8.
Vamos tentar colocar os outros algarismos: tentando o maior possivel 
temos n = 9.876.543.120. Mas pra ser divisivel por 11 a soma dos
algarismos de ordem impar subtraida da soma dos algarismos de ordem 
par deve ser divisivel por 11. Nesse caso: 26-19=7 e n não é o procurado.
Vamos tentar fazer essa subtração ser divisivel por 11. Veja que o 
modo mais facil é igualar ao proprio 11. Pra isso precisamos trocar 
alguns algarismos
para mudar suas ordens. Pra achar o maior numero começamos a fazer 
isso pela direita: invertendo o 4 com o 3. Assim aumentamos pra 9 a diferença.
Trocamos mais uma vez, agora 6 com 5. Devemos ter o numero desejado: 
n = 9.875.634.120

- Original Message -
From: mailto:[EMAIL PROTECTED]Artur Costa Steiner
To: mailto:obm-l@mat.puc-rio.brobm-l@mat.puc-rio.br
Sent: Wednesday, May 16, 2007 2:39 PM
Subject: RES: [obm-l] Multiplos de 5 , 8 e 11

Também naop consegui achar o máximo de A
-Mensagem original-
De: mailto:[EMAIL PROTECTED][EMAIL PROTECTED] 
[mailto:[EMAIL PROTECTED] nome de Bruno França dos Reis

Enviada em: quarta-feira, 16 de maio de 2007 13:39
Para: mailto:obm-l@mat.puc-rio.brobm-l@mat.puc-rio.br
Assunto: Re: [obm-l] Multiplos de 5 , 8 e 11

0 pertence a A, logo A é não vazio
9876543210 é cota superior de A.
A é fechado (pois é formado apenas por pontos isolados).
Assim temos que A admite máximo.

Agora, como achar esse máximo, sem usar força bruta, não estou conseguindo!

Podemos determinar o maior múltiplo comum de 5, 8 e 11 menor do que 
9876543210 facilmente, chamêmo-lo k. Se k pertence a A, acabou. 
Senão, queremos então achar n tal que:

(5*8*11) * n = 440n = k
e
440n possua apenas algarismos distintos

Dá pra fazer certas observações (imediatas) que reduzem em muito o 
número de testes que
teríamos que fazer, mas deve ter alguma solução não braçal que nao 
consigo encontrar!


Abraço
Bruno

2007/5/16, Artur Costa Steiner 
mailto:[EMAIL PROTECTED][EMAIL PROTECTED]:

Gostaria de uma sugestão neste problema de teoria dos numeros

Seja A o conjunto dos multiplos comuns de 5, 8, 11 compostos por 
algarismos distintos (base 10, conforme usual). A tem um elemento 
máximo? Se tiver, qual?


Artur

=
Instruções para entrar na lista, sair da lista e usar a lista em
http://www.mat.puc-rio.br/~nicolau/olimp/obm-l.htmlhttp://www.mat.puc-rio.br/~nicolau/olimp/obm-l.html
=




--
Bruno França dos Reis
email: bfreis - http://gmail.com/gmail.com
gpg-key: 
http://planeta.terra.com.br/informatica/brunoreis/brunoreis.keyhttp://planeta.terra.com.br/informatica/brunoreis/brunoreis.key

icq: 12626000

e^(pi*i)+1=0


[obm-l] Garrafa de Klein

2007-05-08 Por tôpico Carlos Eddy Esaguy Nehab

Oi, gente,

Procurando (para meus alunos) algum mpeg ou java com a geração da 
garrafa de Klein dei de cara com 3 referências interessantes (dentre 
as zilhões):

http://alem3d.obidos.org/pt/struik/kbottle/mov
http://www.mat.ufpb.br/~lenimar/cgraf/inters/2sup13.htmço.

e http://www.kleinbottle.com/classicalklein.htm, um fabricante 
(mesmo) de garrafas de Klein (dica do site 
http://inexo.com.br/~danton/blog/index.shtml?P=2003-07).  Engraçado, não?


Se alguém conhecer alguma dica de uma simulação em java agradeço...

Abraços,
Nehab

=
Instruções para entrar na lista, sair da lista e usar a lista em
http://www.mat.puc-rio.br/~nicolau/olimp/obm-l.html
=


Re: [obm-l] Livros d matematica

2007-05-05 Por tôpico Carlos Eddy Esaguy Nehab

Oi,

Poupando trabalho ao Nicolau...

Olha os direitos autorais.
Atualmente há edição em português e custa  R$ 60,00.Compre-o !!!

Veja em  http://www.vestseller.com.br/

Nehab


At 19:54 4/5/2007, you wrote:

alguem tem um link pra baxa o livro lidski to q nem loko atras
e alguem pode me indica outros pra olimpiadas talz
abraço

_
O Windows Live Spaces é seu espaço na internet com fotos (500 por 
mês), blog e agora com rede social http://spaces.live.com/


=
Instruções para entrar na lista, sair da lista e usar a lista em
http://www.mat.puc-rio.br/~nicolau/olimp/obm-l.html
=


Re: [obm-l] Malba Tahan

2007-04-26 Por tôpico Carlos Eddy Esaguy Nehab

Queridos colegas,

Não resistí  à tentação de falar sobre o problema dos quatro 
quatros  Fui meio que pupilo aluno do Malba Tahan no Pedro II em 
1959 (calma, só tenho 61 aninhos) numa época e que a gente também 
estudava Latim, Grego, Geometria Descritiva, Perspectiva, Francês, 
etc) e na época sua irmã (que era uma simpática gorducha) era a 
bibliotecária da unidade do Humaitá.  Saudades da inconseqüência 
daquela época...


Então, gostaria de lembrar que este problema era proposto para a 
garotada de 10 a 12 anos.  Logo, sem teorizar, a graça está é em usar 
as 4 operações e (talvez) o fatorial que era explanado localmente.


E se não me falha o raio da memória, havia um ou dois números (entre 
1 e 100) que não conseguíamos representar.  E ponto final.  E 
conviver com esta impossibilidade era também um aprendizado !  A vida 
é assim... (desculpem a filosofada...).


É claro que se a gente começar a usar um monte de recursos 
adicionais, o problema perde um pouco a função original (que era 
desafiar a garotada e motivá-los para as curiosidades da Matemática), 
a simplicidade e a graça.


Carlos Nehab

PS: Não sei se vocês sabem mas o Homem que Calculava já existe há 
algum tempo em Inglês.  Tive oportunidade de comprá-lo para filhos de 
alguns amigos estrangeiros... (vide amazon)


At 14:38 25/4/2007, you wrote:

Ola'  Claudio e colegas da lista,

com termiais fica uma covardia:
termial(4) * termial(4) - termial(4) / termial(4) = 10*10 - 1 = 99

E se puder usar exponenciacao e raiz quadrada, o ceu e' o limite:
termial( termial( raiz(4) )) * 4^raiz(4) + termial(raiz(4)) = 6*16 + 3 = 99

Usando apenas raiz, fatorial e logaritmo, cheguei a pensar em:
4! * [ 4 + log(sqrt(sqrt(sqrt(4 / log(4) ] = 24 * [ 4 + 1/8 ] = 99

Mas confesso que somente com as 4 operacoes basicas, e sem usar 
dizimas, nao consegui vislumbrar coisa alguma...


[]'s
Rogerio Ponce


Re: [obm-l] Malba Tahan
claudio\.buffara
Wed, 25 Apr 2007 08:58:24 -0700
Nesse caso, Termial(4)^raiz(4) - 4/4 = 99.

Maurício Collares [EMAIL PROTECTED] escreveu:
Se me lembroe bem, também é permitido usar termiais (Termial de i =
Somatorio de 1 a i de i... É como fatorial, só que usa soma ao invés
de produto)

--
Abraços,
Maurício

__
Fale com seus amigos de graça com o novo Yahoo! Messenger
http://br.messenger.yahoo.com/


Re: [obm-l] Re:[obm-l] Teoria dos n�meros 56....56 etc

2007-04-26 Por tôpico Carlos Eddy Esaguy Nehab

Oi, gente (e Claudio),

Eu achei este problema um pouquinho pesado para o Nivel 1 (Olimpíada 
de Maio de 2003, acho) e até comentei isto com o  Nicolau (em off) há 
algum tempo.


Eis uma solução mais com cabeça de Nivel 1 (eu acho) e com um 
pouquinho de força bruta...   Embora eu a tenha feito, o Nicolau a completou.


O número termina em 56 e 5+6 = 11; logo, como a soma de seus 
algarismos é 56 a soma dos demais é 45.  Então ele tem que ter no 
mínimo 5 algarismos (pois 5 x 9 = 45).Além disso é divisível por 
8 (e 7 pois é divisível por 56).   Então os três últimos algarismos 
formam um número divisível por 8...  Logo, devem ser 256, 456, 656 ou 
856.  Então nosso N = M56, onde M é tal que:

- a soma dos algarismos deve ser 45 (portanto no mínimo 5 algarismos);
- o último algarismo deve ser 2, 4, 6 ou 8 (então tem que ter mais do 
que 5 algarismos)

- deve ser divisível por 7
- deve ser o menor possível.

Tentemos M's:
O menor possível, soma dos algarimos igual a 45, último algarismo 
8,6,4,ou 2 e divisível por 7...:

6 algarismos (do menor pro maior)
18 - não é divisível por 7
289998 - não é divisível por 7
298998 - é divisível por 7 (é o cara!)

Logo o N procurado é 29899856

Nehab


At 16:21 25/4/2007, you wrote:

O enunciado implica que:
N == 56 (mod 100) == N == 56 (mod 4*25)
N == 0 (mod 56) == N == 0 (mod 8*7)
N == 56 (mod 9) == N == 2 (mod 9)

Ou seja:
N == 6 (mod 25)
N == 0 (mod 8)
N == 0 (mod 7)
N == 2 (mod 9)

n == 6 (mod 25) ==
N = 6 + 25*a == 2 (mod 9) ==
a == 2 (mod 9) ==
a = 2 + 9*b ==
N = 56 + 225*b == 0 (mod 8) ==
b == 0 (mod 8) ==
b = 8c ==
N = 56 + 1800*c == 0 (mod 7) ==
c == 0 (mod 7) ==
c = 7d ==
N = 56 + 12600*d

Agora, resta achar d de modo que a soma dos algarismos de N seja 56, 
ou equivalentemente, que a soma dos algarismos de 126*d seja 45.


Um pouco de reflexão mostra que d não deve ser muito pequeno, pois 
se o algarismo médio é 4,5 (=(0+1+2+...+9)/10), então 126*d deve ter 
cerca de 45/4,5 = 10 algarismos (é claro que tem que ter, no mínimo, 
6 algarismos, pois o maior número de 5 (ou menos) algarismos com 
soma 45 é 99.999, que não é múltiplo de 126).


Mas, por sorte, 88.888*126 = 11.199.888, cuja soma dos algarismos é 45.
O N correspondente é 1.119.988.856.
Falta provar que este é, de fato, o menor N que satisfaz ao enunciado.

Por enquanto, estou sem idéias.

[]s,
Claudio.



De: [EMAIL PROTECTED]
Para: obm-l@mat.puc-rio.br
Cópia:
Data: Mon, 23 Apr 2007 16:46:56 -0300
Assunto: [obm-l] Teoria dos números

Amigos, ajude-me nessas questões:

 1) Ache o menor número natural terminado em 56, divisível por 56, 
e com a soma dos seus algarismos igual a 56.


 2) Quantas soluções inteiras tem a equação x^1995 + y^1996 = z^1997


Re: [obm-l] Maximiza�

2007-03-30 Por tôpico Carlos Eddy Esaguy Nehab

Oi, Vinícius,

Como é meu hábito, ao invés de resolver  problema básicos postados, 
vou dar o caminho das pedras, propondo outro problema simples para 
você ter uma percepção geométrica dos problemas propostos:


Imagine que você queira obter o maior valor possível para Z = x + 2y, 
sabendo que:


(1)  x = 0
(2)  y = 0
(3)  x + y = 5
(4)  3x + 2y = 6

Note que todas as restrições são lineares e se você pensar no plano 
xy perceberá que cada restrição define uma região do plano.

(1) região do 1 e 4 quadrantes;
(2) região do 1 e 2 quadrantes;
(3) região abaixo da reta que passa pelos pontos (0;5) e (5;0);
(4) região acima da reta que passa pelos pontos (2;0) e (0;3).

A interseção destas regiões é um quadrilátero de vértices nos pontos 
(2;0); (5;0); (0;3) e (0;5).


Agora imagine que você faça Z = 2 e Z = 4 na função objetivo que você 
quer maximizar...


Veja que as reta  4 = x + 2y  e 6 = x + 2y são paralelas e quanto 
maior o valor de Z, mais alto essas estão no plano (ou seja, se você 
vai aumentando z, o gráfico da reta  z = x + 2y vai subindo...


Ora, desejamos um par (x;y) que esteja na região delimitada pelo 
quadrilátero  e que torne a expressão z = x + 2y máxima, certo?


Se você concorda que o valor de z procurado deva corresponder a uma 
reta que encoste na regão do quadrilátero e que esteja o mais alto 
possível, você entendeu a interpretação geométrica do problema de 
programação linear.


E então a solução corresponde ao par (x; y) que é a interseção das 
retas  x + y = 5   e  3x + 2y = 6   (veja as restrições 3 e 4).  Daí 
basta calcular o valor de z para este par.


Espero ter ajudado.

Abraços,
Nehab

At 07:25 30/3/2007, you wrote:

Bom dia.
Gostaria de obter de vocês uma opinião a respeito de dois problemas 
de maximização:


Uma empresa de artigos de couro fabrica dois tipos de produtos: 
malas e mochilas. A empresa tem quatro departamentos para 
fabricação. As malas são vendidas com lucro de R$ 50 / un e o lucro 
por unidade da mochila é R$ 40. As quantidades de horas necessárias 
para confeccionar cada produto, assim como o número total de horas 
disponíveis em cada departamento, são apresentados a seguir:

Departamento 1
Horas / dia: 300
Horas necessárias (mala): 2
Horas necessárias (mochila): 0 (não produz)
Departamento 2
Horas / dia: 540
Horas necessárias (mala): 0 (não produz)
Horas necessárias (mochila): 3
Departamento 3
Horas / dia: 440
Horas necessárias (mala): 2
Horas necessárias (mochila): 2
Departamento 4
Horas / dia: 300
Horas necessárias (mala): 6/5
Horas necessárias (mochila): 3/2

Maximizar o lucro da empresa.

Uma empresa fabrica três tipos de madeira compensadas (placas de 
aglomerados) e possui três departamentos de produção: 1, 2 e 3. Os 
dados abaixo resumem a produção em horas por unidade de cada um dos 
três departamentos de produção, o tempo máximo disponível em cada 
departamento e o lucro unitário de cada placa:


Departamento I: Tempo disponível: 900h
Departamento II: Tempo disponível: 400h
Departamento III: Tempo disponível: 600h

Placa A (lucro por unidade fabricada: R$ 40):
Operações em horas (departamento I): 2h
Operações em horas (departamento II): 2h
Operações em horas (departamento III): 4h

Placa B (lucro por unidade fabricada: R$ 30):
Operações em horas (departamento I): 5h
Operações em horas (departamento II): 5h
Operações em horas (departamento III): 2h

Placa C (lucro por unidade fabricada: R$ 20):
Operações em horas (departamento I): 10h
Operações em horas (departamento II): 3h
Operações em horas (departamento III): 2h

Maximizar o lucro da empresa.

Equação e inequações do primeiro problema (mala = x; mochila = y):
Função lucro: 50(x1+x2+x3+x4) + 40(y1+y2+y3+y4)
Restrições de cada departamento:
2x1 + 0y1 = 300
3y2 + 0x2 = 540
2x3 + 2y3 = 440
(6/5)x4 + (3/2)y4 = 300

Equação e inequações do segundo problema:
Função lucro: 40a + 30b + 20c
Restrições de cada departamento:
2a+5b+10c=900
2a+5b+3c=400
4a+2b+2c=600

Minha dúvida é: a soma da maximização de cada uma das partes é igual 
à maximização do todo? Ou eu devo considerar essas restrições 
interdependentes e fazer um sistema linear de quatro (no primeiro 
problema) ou três (no segundo problema) inequações?
Se a soma da maximização de cada uma das partes puder ser 
considerada a maximização do todo, qual deveria ser o enunciado para 
que as restrições pudessem, nos dois problemas, ser interdependentes?


Obg,
Vinícius



Re: [obm-l] o menor valor

2007-03-30 Por tôpico Carlos Eddy Esaguy Nehab

Oi, Ronaldo,

Complementando a dica do Claudio, veja que a uma interpretação 
geométrica ajuda...


x2 + y2 = 1 é uma circunferência de centro na origem e raio 1.
Considere que você deseja minimizar a expressão z = 2y -6x +1 (vide 
Claudio, abaixo) que, para cada valor de z,  corresponde a uma reta 
paralela à reta  y = 3x.


Logo, você deseja a reta mais alta que tangencia a circunferência 
(deu para sacar?).  Pense em vários valores de z e no gráfico das 
retas correspondentes.


Com o par (x;y) procurado é esta interseção, por uma simples 
semelhança de triângulos (imagine que a reta tangente está traçada) , 
tal par (x; y) é tal que y = -x/3 (pois o ponto está no segundo 
quadrante) .  Logo, substituindo na circunferência, chegamos ao 
resultado já fornecido pelo Claudio.


Obs: enviei estas observações pois hoje mesmo  postei um comentário 
sobre programação linear  bem geométrico e acho que a interpretação 
geométrica é muito útil em problemas simples como os postados para, 
depois, entendermos os realmente complicados...


Abraços,
Nehab


At 19:11 30/3/2007, you wrote:

Olá Cláudio. Obrigado pela referência, vou dar uma olhada.
Eu mesmo confesso que não sei porque o método funciona.

On 3/28/07, claudio.buffara [EMAIL PROTECTED] wrote:

Infelizmente, a maioria das pessoas que usa multiplicadores de Lagrange
segue apenas uma receita de bolo, sem ter a menor
ideia de por que o metodo funciona. Uma boa explicacao encontra-se no cap. 4
do livro Analise Real - vol.2 do Elon Lages Lima,
publicado pelo Impa.

No entanto, nesse caso, dah pra fazer com matematica do ensino medio:

Como x^2+y^2=1, o problema eh minimizar 2y-6x+1 sujeita a x^2+y^2=1.
Uma ideia razoavel eh fazer x = cos(t), y = sen(t) e cair no problema:
Minimizar f(t) = 2*sen(t) - 6*cos(t) + 1
raiz(40)*(sen(t)*(2/raiz(40)) - cos(t)*(6/raiz(40))) + 1 =
raiz(40)*sen(t-a) + 1, onde cos(a) = 2/raiz(40) e sen(a) = 6/raiz(40).

O valor minimo de f(t) ocorre quando sen(t-a) = -1 ==
f(t) = 1 - raiz(40) = 1 - 2*raiz(10).
Nesse caso, t - a = -pi/2 + 2kpi == t = a - pi/2 + 2kpi ==
x = cos(t) = cos(a - pi/2) = sen(a) = 3/raiz(10)
y = sen(t) = sen(a - pi/2) = -cos(a) = -1/raiz(10)

[]s,
Claudio.


-- Cabeçalho original ---

De: [EMAIL PROTECTED]
Para: obm-l@mat.puc-rio.br
Cópia:
Data: Wed, 28 Mar 2007 13:43:52 -0300
Assunto: Re: [obm-l] o menor valor

 Ah... só mais uma coisa... esqueci o link:

http://en.wikipedia.org/wiki/Lagrange_multipliers


 On 3/28/07, Ronaldo Alonso [EMAIL PROTECTED] wrote:
 
  Só pra complicar um pouco, essa dá para resolver com cálculo
  usando multiplicadores de Lagrange, isto é minimizar o valor
  de uma função sujeita a uma restrição.
 No caso a função é  f(x,y) = x^2 + y^2 - 6x + 2y  e a restrição é
  g(x,y) = x^2 + y^2 = 1
 
Vc forma uma função auxiliar h(x,y) = f(x,y) - lambda * g(x,y)
  Faz as derivadas parciais de h(x,y) iguais a zero, calcula lambda usando
o
  vínculo
  e substitui os valores de x e y que fazem com que tornam h mínimo (para
  isso vc tem
  que resolver um sisteminha.
 
 Alguém se habilita a usar esse esquema para conferir a resposta?
 
  []s a todos.
 
 
 
 
  On 3/26/07, vitoriogauss [EMAIL PROTECTED] wrote:
  
   legal essa maneira ...gostei
  
  
Já que vc. gosta de G.A. (brincadeira) pode considerar a primeira
   equação como a de uma circunferência centrada em O, de raio unitátio,
e
   procurar o raio de outra com centro em (3,-1) que tangencia a
primeira.
   
Deve obter o menor valor como 1 - sqrt10
   
[]s
   
vitoriogauss [EMAIL PROTECTED] escreveu:
se x^2 + y^2 = 1, o menor valor de x^2 + y^2 - 6x + 2y é
   
Vitório Gauss
   
   
   
  
=
Instruções para entrar na lista, sair da lista e usar a lista em
http://www.mat.puc-rio.br/~nicolau/olimp/obm-l.html
   
  
=
   
   
 __
Fale com seus amigos  de graça com o novo Yahoo! Messenger
http://br.messenger.yahoo.com/
  
   Vitório Gauss
  
  
  
  
=
   Instruções para entrar na lista, sair da lista e usar a lista em
   http://www.mat.puc-rio.br/~nicolau/olimp/obm-l.html
  
  
=
  
 
 
 
  --
  -
  Analista de Desenvolvimento
  Conselho Regional de Engenharia, Arquitetura e Agronomia de SP.




 --
 -
 Analista de Desenvolvimento
 Conselho Regional de Engenharia, Arquitetura e Agronomia de SP.




=
Instruções para entrar na lista, sair da lista e usar a lista em
http://www.mat.puc-rio.br/~nicolau/olimp/obm-l.html

Re:[obm-l] Casais

2007-03-16 Por tôpico Carlos Eddy Esaguy Nehab

Oi, Claudio,

Você foi mais corajoso do que eu.  Ei ia perguntar se o problema era 
politicamente correto, mas...  você comentou...


Abraços,
Nehab

At 09:07 16/3/2007, you wrote:

-- Cabeçalho original ---

De: [EMAIL PROTECTED]
Para: obm-l@mat.puc-rio.br
Cópia:
Data: Thu, 15 Mar 2007 15:18:35 -0300
Assunto: [obm-l] Casais

 Problema
 No início de uma festa há  6  rapazes desacompanhados 
e  10  garotas desacompanhadas.

 Quantos são os estados possíveis no fim da festa?

 Benedito


Voce perguntou o numero de estados possiveis. Supondo que voce se 
refira ao estado de agregacao no fim da festa (ou seja, quem
vai estar com quem num dado instante escolhido arbitrariamente como 
sendo o fim da festa), eu diria que esse eh igual ao
numero de particoes de um conjunto com 16 elementos em subconjuntos 
nao vazios, ou seja 10480142147 (=Bell(16)).


Se a sua resposta foi SOMA(k=0...6) Binom(6,k)*Binom(10,k)*k!, voce 
eh decididamente careta...


[]s,
Claudio.




=
Instruções para entrar na lista, sair da lista e usar a lista em
http://www.mat.puc-rio.br/~nicolau/olimp/obm-l.html
=


=
Instruções para entrar na lista, sair da lista e usar a lista em
http://www.mat.puc-rio.br/~nicolau/olimp/obm-l.html
=


Re: [obm-l] Quest�o de Complexos

2007-03-15 Por tôpico Carlos Eddy Esaguy Nehab

Oi, Júlio.

Pense assim: sua expressão é da forma

X = (1+a)(1+a^2)(1+a^4)

concorda?  Então multiplique por   1 - a   ambos os lados e você chega lá...

Abraços,
Nehab


At 00:16 15/3/2007, you wrote:
Amigos, estou estudando pro ITA e não tô conseguindo resolver essa 
questão. Obrigado!


{1 + [(1 + i)/2]}*{1 + [(1 + i)/2]^2}*{1 + [(1 + i)/2]^4}*{1 + [(1 + 
i)/2]^8}*...*{1 + [(1 + i)/2]^2^n}


Espero que dê pra entender...


Isso é o produtório de 1 + [(1 + i)]^2^k (Com K variando de 0 a n)




--
Atenciosamente
Júlio Sousa


=
Instruções para entrar na lista, sair da lista e usar a lista em
http://www.mat.puc-rio.br/~nicolau/olimp/obm-l.html
=


Re: [obm-l] Quest�o Trigonom�trica

2007-03-03 Por tôpico Carlos Eddy Esaguy Nehab

Oi, Anderson,

Um possível pulo do gato é notar que o lado esquerdo da equação é do 
tipo y + 1/y, logo, seu valor está fora do intervalor (-2; 2).  Daí, 
olhando pro lado direito... dá para seguir em frente.  Percebeu?


Abraços,
Nehab

At 13:17 3/3/2007, you wrote:

Oi, segue a questão!


 As soluções da equação: cossec(13x) + sen(13x) = 2cos(3x)



Até!


=
Instruções para entrar na lista, sair da lista e usar a lista em
http://www.mat.puc-rio.br/~nicolau/olimp/obm-l.html
=


=
Instruções para entrar na lista, sair da lista e usar a lista em
http://www.mat.puc-rio.br/~nicolau/olimp/obm-l.html
=


Re: Res: [obm-l] Trigonometria

2007-02-21 Por tôpico Carlos Eddy Esaguy Nehab

Oi,

A dica do Danilo que sugere usar a 
relação  tan(60-x)*tanx*tan(60+x)=tan3x  (embora meio mágica para o 
propósito do exercício proposto, simples de demonstrar)  me lembrou 
outra relação semelhante, qual seja,


4.sen(60-x).senx.sen(60+x) = sen 3x

utilizada em uma das demonstrações do interessante (e nada simples) 
teorema de Morley, que se segue:


Os três pontos de interseção das trissetrizes adjacentes dos 
ângulos de um triângulo qualquer formam um triângulo equilátero


Vejam uma ótima discussão 
em  http://www.cut-the-knot.org/triangle/Morley/index.shtml


Abraços,
Nehab

At 19:56 19/2/2007, you wrote:

Use que tan(60-x)*tanx*tan(60+x)=tan3x.
faça x=10 e use que tanx=1/tan(90-x).
[]'s

- Mensagem original 
De: Graciliano Antonio Damazo [EMAIL PROTECTED]
Para: obm-l@mat.puc-rio.br
Enviadas: Segunda-feira, 19 de Fevereiro de 2007 3:21:37
Assunto: [obm-l] Trigonometria

Alguem poderia me ajudar?

prove que: tg20º.tg30º.tg40º = tg10º

Obrigado por enquanto...

__
Fale com seus amigos de graça com o novo Yahoo! Messenger
http://br.messenger.yahoo.com/


__
Fale com seus amigos de graça com o novo Yahoo! Messenger
http://br.messenger.yahoo.com/


=
Instruções para entrar na lista, sair da lista e usar a lista em
http://www.mat.puc-rio.br/~nicolau/olimp/obm-l.html
=


Re: [obm-l] Equa��es ITA

2007-02-17 Por tôpico Carlos Eddy Esaguy Nehab

Oi, Bruna,

Renan já mandou a solução, mas gostaria de lembrar que já foi 
abordada nesta lista outra forma de resolver este tipo de questão (em 
vários emails), que pode ser útil para você:


Se dois polinômios possuem raízes em comum, elas coincidem com as 
raízes de seu mdc.


Assim se você souber calcular o mdc entre dois polinômios (que segue 
a mesma estratégia de mdc entre inteiros)... é um caminho 
interessante e  mais geral.


Abraços,
Nehab

At 20:56 16/2/2007, you wrote:
As equações x³ + ax² + 18 = 0 e x³ + nbx + 12 = 0, onde a e b são 
constantes reais e n um inteiro têm duas raízes comuns. Determine nb.

--
Bjos,
Bruna


=
Instruções para entrar na lista, sair da lista e usar a lista em
http://www.mat.puc-rio.br/~nicolau/olimp/obm-l.html
=


Re: [obm-l] Desenho Geom�trico [Complexos em Geometria e Napoleao]

2007-02-15 Por tôpico Carlos Eddy Esaguy Nehab

Oi, Luís

Embora sem poder dedicar muito tempo para isto, estou mapeando os 
exercícios de construções geométricas no triângulo em níveis de 
aprendizagem (em 3 niveis) pois de fato, uns são imediatos mas outros 
ainda são problemas em aberto.


Minhas referências centrais continuam sendo os artigos publicados no 
Mathematics Magazin - do William Wernick  (1982), que listou 139 
problemas e do Leroy F. Meyers (1996), que complementou algumas 
soluções (não sei quantos dos problemas estão em aberto - parece que 
são uns 20).


Mas meu interesse é desenvolver nos alunos o olhar geométrico, e 
então prefiro enfatizar os problemas com solução geométrica pura, 
evitando os que usam soluções por Geometria Analítica ou usando 
provas indiretas de não construtividade (usando Gauss, ou suas 
conseqüências)


Quanto ao prof Astyages Brasil (acho difícil haver duas pessoas com 
este nome)  não o conheço pessoalmente, mas há pouco tempo dei de 
cara numa livraria com um livro (possivelmente o que você soube que 
ele publicou), com no máximo umas 100 páginas em que o referido 
professor apresenta 3 demonstrações do último teorema de 
Fermat.  Logo... (é raro de acontecer, mas este é um livro que não eu 
li e não gostei...).  Vide 
http://www.papelvirtual.com.br/sitenovo/detalhes_produto2.asp?IDProduto=1058


Quanto ao problema que você propôs (o problema 25 do Wernick) - que 
prometeu a solução ...


Construir com régua e compasso um  triângulo dados o lado a ; a 
mediana m relativa ao lado a  e a bisetriz interna d relativa ao lado


não consegui uma solução simples e fui atrás de sua dica (lá vi a 
solução) e de fato é muito engenhosa e dificilmente eu a encontraria.


Abraços,
Nehab

At 14:30 14/2/2007, you wrote:

Sauda,c~oes,

Oi Nehab,

Este teu email é o gancho pra mandar o problema e
a solução abaixo.

==
rhombus (losange) construction
Posted by: Lu?s Lopes [EMAIL PROTECTED] qedtexte
Date: Wed Feb 14, 2007 4:03 am ((PST))

Dear Hyacinthists,

Construct a rhombus given a line and any four points
so that a diagonal is parallel to the line and each side
goes through one of those four points.

Mr. Smith presented me this problem yesterday and
told me it has been given as an assignment in 1963.
And that he is still looking for a solution!

As his memory may fail and I don't want to lose time
in an ill problem I would like to have your opinion
about it.

Best regards,
Luis

Dear Luis,

Let A,B,C,D be the given points, where A and C are supposed to lie on
opposite sides of the rhombus.

Reflect the vector BD in the given line to obtain B'D' and draw the latter
from A to obtain vec. AM= vec. B'D'. Then point M must lie on the same side
line of the rhombus as C. This defines (unless M=C, of course) the side
line and hence the directions of all the sides.

Best regards,
Vladimir

O Vladimir é da Rússia e lá eu acho que o DG faz parte
do currículo.

O professor do teste em 1963 era o Astyages Brasil.
Só conheci o Brasil recentemente, mas já ouvi dizer
que ele foi um excelente professor de geometria e
afins. Talvez você possa falar um pouco a respeito dele.

Soube que ele publicou um livro recentemente.

Quem me propôs o problema ontem foi o xxx
(encontrei-me ontem com ele pela primeira vez).
Ele era estudante da PUC e o Brasil passou o problema
num teste. Ele viu na tela do meu computador a figura
da solução do problema a,h_a,m_b e se deteve perto
de mim (assim do nada) pra me dizer que recentemente
tinha resolvido um problena de DG. A conversa avançou e
ele quer dizer pro Brasil que conseguiu resolvê-lo.

Por essas e outras não consigo entender por que o DG
foi retirado do currículo. E agora com os programas de
desenho deveria voltar.

O problema a,h_a,m_b de construir o triângulo com
estes dados é fácil. Um outro A,m_a,d_a d_a=bissetriz
interna é bem interessante e legal.

Conheço umas 4 soluções para ele. A solução sintética que
apareceu num periódico é muito elegante. Recai no
problema A,a,d_a, um clássico. A solução com GA
(do A,m_a,d_a) permite o uso de diversos conceitos,
a começar pela dedução do lugar geométrico dos pontos
médios dos segmentos determinados pelas interseções das
retas que passam pelo pé (D_a) da bissetriz com os lados
do triângulo.
Num sistema conveniente isto dá uma hipérbole (cônicas,
outro assunto que poderia reaparecer num tratamento
geométrico como o da apostila do Célio Pinto) de vértices
A e D_a e assíntotas paralelas aos lados do ângulo no
vértice A. A interseção com o círculo (A,m_a) resolve o
problema.

Conheço tb a solução sintética de um livro alemão que
vou mostrar num livro que estou escrevendo.

Podemos pensar no problema com a bissetriz externa
também.

Outro problema interessante é a,m_a,d_a. Vou
colocar a solução sintética (a essência da geometria)
do prof. Paul Yiu que apareceu num jornal eletrônico
(ForumGeometricorum) recentemente.

Caraca, não quero ganhar o concurso de quem faz o
mais longo email.

[]'s
Luís


From: Carlos Eddy Esaguy Nehab [EMAIL PROTECTED]
Reply

Re: [obm-l] Ajuda urgente

2007-02-15 Por tôpico Carlos Eddy Esaguy Nehab

Oi, Marcus Aurélio

Calcule a seqüência das diferenças b(n) = a(n+1) - a(n)  novamente a 
seqüência das diferenças c(n) = b(n+1) - b(n). Você obtém a PG  8, 24, 72, 216.


Abraços,
Nehab

Oi, At 11:57 15/2/2007, you wrote:

Alguem poderia me ajudar nessa questão?

Determine o termo geral da seqüência {3, 0, 5, 34, 135, 452, ...} e calcule
em seguida a soma dos seus n primeiros termos.



=
Instruções para entrar na lista, sair da lista e usar a lista em
http://www.mat.puc-rio.br/~nicolau/olimp/obm-l.html
=


=
Instruções para entrar na lista, sair da lista e usar a lista em
http://www.mat.puc-rio.br/~nicolau/olimp/obm-l.html
=


Re: [obm-l] lema de gauss

2007-02-15 Por tôpico Carlos Eddy Esaguy Nehab

Oi, Rafaek,

Conheço alguns resultados de Gauss mas não consigo perceber a qual o 
citado participante quis se referir.   De qualquer forma, até que 
algum outro colega consiga esclarecer esta dúvida, vamos a uma dica 
para seu problema.


Um amigo e colega que frequenta esta lista passou para  seus alunos 
um exercício muito semelhante a este, qual seja:

sqrt( 1 +  sqrt (1 + sqrt (1 + x) ) )  = x

A solução que encontrei (que vale para o seu, também, e o deixarei 
para você, mas apenas dizendo que a solução  vale  [sqr(21) - 
1]/2  )  foi a seguinte:


Observe que x  é igual ao radical horroroso da esquerda, 
concorda?  Então imagine que você substitua o x do radical pelo 
próprio radical:

Ficaria assim:

sqrt( 1 +  sqrt (1 + sqrt (1 + [  sqrt( 1 +  sqrt (1 + sqrt (1 + x) ) 
)   ]) ) )  = x


Se você fizer isto indefinidamente, obterá o seguinte:

x = sqr( 1 + sqr ( 1+ ..   +  sqr ( 1 + x) .)[com 
infinitos radicais]


Elevando ao quadrado, vem:

x^2 = 1 +   sqr ( 1+ ..   +  sqr ( 1 + x) .)  ou 
seja, do lado direito você continua com a mesma infinidade de 
radicais, ou seja, x:


x^2 = 1 + x  , cuja solução que convém (0)  dá  o número 
áureo   [sqr(5) + 1 ]  / 2


Há um detalhe nesta demonstração que precisa de um cuidado maior para 
sua justificativa.  Você percebe onde houve um certo abuso?


Abraços,
Nehab

At 18:27 15/2/2007, you wrote:

Ha um tempo atras apareceu na lista um problema do ime mais ou menos assim:

sqrt(5-sqrt(5-x)) = x

Um dos participantes da lista sugeriu o lema de gauss para resolver a questao.

 O que seria exatamente esse lema de gauss e mais importante ainda:
Como ele pode me ajudar a resolver essa questao  ( ja que pelo pouco
que entendi ele nao é um algoritmo para fatorar polinomios)   ???

Obrigado
--
Rafael

=
Instruções para entrar na lista, sair da lista e usar a lista em
http://www.mat.puc-rio.br/~nicolau/olimp/obm-l.html
=


=
Instruções para entrar na lista, sair da lista e usar a lista em
http://www.mat.puc-rio.br/~nicolau/olimp/obm-l.html
=


Re: RES: [obm-l] sequencias

2007-02-15 Por tôpico Carlos Eddy Esaguy Nehab

Oi, Arthur,

Acho que analisando as subseqüências de sen(Ln(n)) a seguir dá para 
provar que sen(Ln(n)) não converge.


Chamando log2 de logaritmo na base 2 e fazendo log2(e) = p, temos: 
x(n) = sen (Ln(n) )  =  sen [  log2(n) / p ].


Se n  = 2^k , 2^(2k)  e 2^(3k), obtemos 3 subseqüências de x(n) que 
teriam que convergir para L, quais sejam:


a_k = sen k/p
b_k = sen 2k/p
c_k = sen 3k/p = sen(k/p) [ 3 - (4sen(k/p))^2 ]

Mas
De (sen2k/p) ^2 =  4.(senk/p)^2 .  [1 - (senk/p)^2 ]   teríamos   L = 
0, 1/2 ou - 1/2.
De  sen 3k/p = senk/p [ 3 - (4senk/p)^2]  teríamos  L =  L ( 3 - 
4L^2]  ou seja,  L = 0 ou sqr(3)/2   ou  - sqr(3)/2 .


Logo se provarmos que x(n) não pode convergir para L = 0, estará 
provado que x(n) não é convergente.


Amanhã vejo como fechar isto (que acho que deve ser fácil) pois já 
está na hora das corujas...

Até amanhã
Abraços,
Nehab.

At 15:11 1/2/2007, you wrote:

Outro contra exemplo talvez seja sen(ln(n)), mas embora pareca intuitivo que
esta sequencia divirja, ainda nao a consegui uma prova matematicamente
valida
Artur

-Mensagem original-
De: Artur Costa Steiner
Enviada em: quinta-feira, 1 de fevereiro de 2007 13:56
Para: obm-l@mat.puc-rio.br
Assunto: RES: [obm-l] sequencias


No caso (i), a seq. não tem que ser convergente. Um contra-exemplo é a seq.
cujos termos são 0, 1, 1/2, 0, 1/3, 2/3, 1, 3/4, 2/4, 1/4, 0, 1/5, ...

A lei de formação é um vai vem em [0,1] em que vc vai dividindo o intervalo
em subintervalos com comprimentos dados pelos  inversos dos inteiros
positivos. Vamos direto de 0 a 1, depois voltamos a 0 passando pelo 1/2,
depois vamos de novo para 1 passando agora por 1/3 e 2/3, aí voltamos para 0
por 3/4/, 2/4, 1/4 e assim sucessivamente. Esta seq. satisfaz aa condicoes
dadas mas não converge.

Artur



-Mensagem original-
De: carlos martins martins [mailto:[EMAIL PROTECTED]
Enviada em: terça-feira, 30 de janeiro de 2007 22:34
Para: obm-l@mat.puc-rio.br
Assunto: [obm-l] sequencias


sou novo na lista e estou com um problema, na verdade dois, com sequências,

i) Seja (x_n) uma sequência tq se n tende a oo |x_(n+1) - x_(n)|=0 e  que
(x_n) é limitada.
  Mostre ou dê contra-exemplo que (x_n) é convergente.

ii) Se (a_n)  é uma sequência de números reais definida por
  a_1 = 1 e  a_(n+1)=a_n * (2 - a_(n)/2 ).
  Mostre que 1 = a_n = 2.

Na primeira não tive muito progresso.

Na segunda consegui mostrar por indução que 1 = a_n . Que a_n = 2, não
consegui, cheguei
a_n = 3.

_
Insta-le já o Windows Live Messenger. A nova geração do messenger.
http://get.live.com/messenger/overview

=
Instruções para entrar na lista, sair da lista e usar a lista em
http://www.mat.puc-rio.br/~nicolau/olimp/obm-l.html
=

=
Instruções para entrar na lista, sair da lista e usar a lista em
http://www.mat.puc-rio.br/~nicolau/olimp/obm-l.html
=

=
Instruções para entrar na lista, sair da lista e usar a lista em
http://www.mat.puc-rio.br/~nicolau/olimp/obm-l.html
=


=
Instruções para entrar na lista, sair da lista e usar a lista em
http://www.mat.puc-rio.br/~nicolau/olimp/obm-l.html
=


  1   2   >